寄托家园留学论坛

标题: 1010G零散版友作文互改帖(Argument) [打印本页]

作者: 江雪    时间: 2010-7-6 18:07:31     标题: 1010G零散版友作文互改帖(Argument)

本帖最后由 江雪 于 2010-7-6 18:10 编辑

最近版面零散版友习作增多但很多都是零回复,特开此贴,方便零散版友互改。

保证本贴的秩序,请大家仔细阅读以下的条款,只需花
2分钟即可。望大家积极配合,严格遵守互改规则,这样最终大家每一个人都是受益者。


参与对象:任何寄托版友,主要针对
尚未加入任何小组的零散版友

目标:互改作文。
原则:帮别人改一篇可获得被别人改一篇的资格,即权力与义务对等

规则如下:
·选择楼层原则:接龙——按楼层由近及远顺序选择最近一楼的未被修改且具备被修改资格(点击每层楼左上角的只看该作者可知)的版友习作进行修改,一般是你修改贴的上一楼。不允许随意挑楼层的作文改,否则扣1声望。

·互改具体操作分三步
1、首先跟帖占一层楼写上占位改**的字样,但是不急着修改,以防被人占楼;

2
、然后立马将自己的一篇习作跟贴发上来;
3
、第三步才是返回编辑第一个帖子修改别人的文章。

·特殊情况:如果目前没有可供修改的文章,请等上一楼贴出来修改。


·违规将严惩:
A.
只允许接龙,不允许随意选择楼层修改,违者删习作(楼下尚未修改)或警告1次(楼下已修改)!
B.
占楼须立马修改,虽完成修改但未在24小时内按时完成者,扣1声望!

C.
只发习作或48小时内仍未修改他人习作者,警告1次!(累计3次警告将被自动禁止发帖30天!)
D.
修改他人习作应付差事者,视情况只加0-2声望!
E.
提倡发全文,对修改习作仅上传附件的版友,加声望会有所保留!
F.
互改两帖之外的帖子直接删除,双方有问题交流可发站内短信!

·批改要求:认真、仔细、负责
在文中进行具体的修改,在最后提出文字上、结构上、立意上等不同方面的较为完整的评论,以及,更为重要的是,你的建议

·习作要求:提交前首先自己word检查语法错误并自己改到满意为止,尊重别人的时间,别人也会尊重你的时间。为方便大家阅读,可以附上自己的中文提纲。认真改作文的人可以加3-5声望。

·如何编辑?点击帖子右下方的编辑按钮即可对已经发上的帖子进行反复编辑。

·如何举报他人违规?请去专用的投诉+处理贴,不要在本帖发布举报信息,以免破坏接龙规则。

再一次重申总的原则:
只有你认真修改了别人的习作,别人才会修改你的习作!
作者: cynthiasmile    时间: 2010-7-6 18:17:11

TOPIC: ARGUMENT209 - The following recommendation was made by the Human Resources Manager to the board of directors of the Fancy Toy Company.

"In the last three quarters of this year, under the leadership of our president, Pat Salvo, our profits have fallen considerably. Thus, we should ask for her resignation in return for a generous severance package. In Pat's place, we should appoint Rosa Winnings. Rosa is currently president of Starlight Jewelry, a company whose profits have increased dramatically over the past several years. Although we will have to pay Rosa twice the salary that Pat has been receiving, it will be well worth it because we can soon expect our profits to increase considerably."
WORDS: 503          TIME: 00:40:27          DATE: 2010/7/4 21:50:18

In this memo,the author recommends replacing Pat Salvo,the current president in Fancy Toy Company,with Rosa Winnings,the president of Starlight Jewelry with twice the salary.To support his recommendation,the author cites that the Fancy Toy Company`s profits have declined during the last three quarters ,while Srarlight Jewelry`s profits have increased dramatically over the past several years.Close scrutiny of each of these facts, however, reveals that none of them lend credible support to the recommendation.
A threshold problem with the argument incolces that the author fails to provide any evidence to convince me Pat was the cause of Fancy Toy`s declining profits. Perhaps all the toy companies in this country suffered from formidable doldrums due to the economic depression.Or the costs of materials or labor have increased beyong control.Or more possiblly,perhaps other new toy companies were plunged in  vicious circle for the seriously violent competition , which invited the local depression. In other words, problems such as these might present insurmountable obstacles that prevented Fancy Toy Company's economy developing.
Yet another problem with the argument invovles that the mere fact that Starlight Jewelry`s profits have increased dramatically over the past several years hardly suffices to prove that it`s because of Rosa Winning`s leadership.It`s entirely possible that all jewelry businesses have prospered recently,regardless of the abilities of the president.Or perhaps Starlight Jewelry's competitive opponent firms and companies moved to other cities or even went bankrupt.In short, without accounting for other factors that might contribute to the increase in profits,the author can`t justify the claim .
Finally, even if we admit Rosa Winnings`  abilities in leading the Starlight Jewelry Company,the argument still fails to convince us by resting on a poor assumption that Rosa`s abilities are suitable for the Fancy Toy Company as well.Commen sense tells us that these two companies aims at different kind of consumer groups results in big difference in the distribution of products, and in the management expertise.Additionally, no evidence demonstrated that whether the ability of Rosa is worthy the salary she receives, perhaps the extra money will become the burden of Fancy Toy Company for it has already suffered a decline in profits.What`s more, it always take much time for a new leader to know about the company ,the staff,to enhance the prestige and to decide a new way to devolpe the company.This all may  distract attention from the coming chances and competitions for Fancy Toy Company.
In sum, the editorial relies on a series of dubious assumptions, which render it wholly unpersuasive. To further bolster the author's claims he or she must provide clear evidence such as comparing to another company with equal economic strength in the same period of time,the Fancy Toy Company really did a bad job,and Rosa Winning`s abilities will transfer to the toy business.In order to better evaluate the argument,we would need more information about how Pat and Rosa played the leadership role in each of the two conpanies and the objective and just judgements on each of the two conpanies` performance in the period under their leadership.
作者: 方方土    时间: 2010-7-6 23:39:18

本帖最后由 方方土 于 2010-7-7 19:42 编辑

此前粘贴作文为错误发帖,请版主及各位板油谅解,请先批改cythiasmile的arguement209
作者: 江雪    时间: 2010-7-7 18:24:39

占一楼作文
TOPIC: ARGUMENT26 - The following appeared in a memo from the chairperson of the school board in the town of Saluda.

"For the past five years, Mr. Charles Schade has been the music dir ...
方方土 发表于 2010-7-6 23:39


是先修改你楼上的作文,再贴你自己的作文。

楼下注意,请从二楼的argu改起。
作者: loujing001    时间: 2010-7-7 23:06:46

本帖最后由 loujing001 于 2010-7-7 23:12 编辑

改二楼

TOPIC: ARGUMENT209 - The following recommendation was made by the Human Resources Manager to the board of directors of the Fancy Toy Company.

"In the last three quarters of this year, under the leadership of our president, Pat Salvo, our profits have fallen considerably. Thus, we should ask for her resignation in return for a generous severance package. In Pat's place, we should appoint Rosa Winnings. Rosa is currently president of Starlight Jewelry, a company whose profits have increased dramatically over the past several years. Although we will have to pay Rosa twice the salary that Pat has been receiving, it will be well worth it because we can soon expect our profits to increase considerably."
WORDS: 503          TIME: 00:40:27          DATE: 2010/7/4 21:50:
18

In this memo, the author recommends replacing Pat Salvo, the current president in Fancy Toy Company, with Rosa Winnings, the president of Starlight Jewelry with twice the salary.
感觉这句应该点明用Rosa替换Pat的目的,是为了增加公司利润。因为全文都是讨论Rosa/Pat的领导与公司利润的关系。 To support his recommendation, the author cites that the Fancy Toy Company’s profits have declined during the last three quarters, while Starlight Jewelry’s profits have increased dramatically over the past several years. Close scrutiny of each of these facts, however, reveals that none of them lend credible support to the recommendation.

A threshold problem with the argument includes?that the author fails to provide any evidence to convince me Pat was the cause of Fancy Toy’s declining profits. Perhaps all the toy companies in this country suffered from formidable doldrums due to the economic depression. Or the costs of materials or labor have increased beyond control. Or more possibly, perhaps other new toy companies were plunged in vicious circle for the seriously violent competition, which invited the local depression. In other words, problems such as these might present insurmountable obstacles that prevented Fancy Toy Company's economy developing. 个人感觉economy developing是国家、社会范围的,用在单个公司上好像不太合适。 另外,Rat只在任三个季度,未满一年,而玩具销售可能会受到季度的影响,比如玩具在圣诞节期间很可能就比其他时间卖得好,而这一季度的利润可能就会比其他季度高。

Yet another problem with the argument involves that the mere fact that Starlight Jewelry’s profits have increased dramatically over the past several years hardly suffices to prove that it’s because of Rosa Winning’s leadership. It’s entirely possible that all jewelry businesses have prospered recently, regardless of the abilities of the president. Or perhaps Starlight Jewelry's competitive opponent firms and companies moved to other cities or even went bankrupt. In short, without accounting for other factors that might contribute to the increase in profits, the author can’t justify the claim. 这段和上一段在思路上相似,有点重复的感觉。我觉得可以这么写:同理,Starlight Jewelry的利润增长也不一定是由于Rosa的领导。而且,Rose是现任总裁,但并不意味着她在过去几年Starlight Jewelry利润增长的时候就当上了总裁。

Finally, even if we admit Rosa Winnings’ abilities in leading the Starlight Jewelry Company, the argument still fails to convince us by resting on a poor assumption that Rosa’s abilities are suitable for the Fancy Toy Company as well. Common sense tells us that these two companies aim at different kind of consumer groups results in big difference in the distribution of products, and in the management expertise. Additionally, no evidence demonstrated that whether the ability of Rosa is worthy the salary she receives, perhaps the extra money will become the burden of Fancy Toy Company for it has already suffered a decline in profits. What’s more, it always takes much time for a new leader to know about the company, the staff, to enhance the prestige and to decide a new way to develop the company. 这点我没有想到。This all may distract attention from the coming chances and competitions for Fancy Toy Company. 最后这句好像离主题有点远,或者在后面加一句:给他们公司带来其他的,甚至可能是更严重的问题。

In sum, the editorial relies on a series of dubious assumptions, which render it wholly unpersuasive. To further bolster the author's claims he or she must provide clear evidence such as comparing to another company with equal economic strength in the same period of time, the Fancy Toy Company really did a bad job, and Rosa Winning’s abilities will transfer to the toy business. In order to better evaluate the argument, we would need more information about how Pat and Rosa played the leadership role in each of the two companies and the objective and just judgments on each of the two companies` performance in the period under their leadership.最后一句,and the objective这里有点难懂。


文章结构很清晰,先分别论述两个公司利润变化的原因不一定是总裁造成的,然后退一步说即使是由总裁造成,换人也不一定能解决问题,最后提出改进的建议。语言方面,作者的水平比我高,很多单词是我不会用甚至不认识的。不过,除了少数拼写错误(用红色标出),文章里还有一些标点错误:缺少逗号、句号后面的空格;撇号有误,应该用引号键。最后,我的建议,也是我最近看范文的体会,从题目中寻找证据推翻题目的结论是一种很高级的办法。
这是我第一次改作文,不足之处还请版主版友指出。

作者: loujing001    时间: 2010-7-7 23:11:36

TOPIC: ARGUMENT51 - The following appeared in a medical newsletter.

"Doctors have long suspected that secondary infections may keep some patients from healing quickly after severe muscle strain. This hypothesis has now been proved by preliminary results of a study of two groups of patients. The first group of patients, all being treated for muscle injuries by Dr. Newland, a doctor who specializes in sports medicine, took antibiotics regularly throughout their treatment. Their recuperation time was, on average, 40 percent quicker than typically expected. Patients in the second group, all being treated by Dr. Alton, a general physician, were given sugar pills, although the patients believed they were taking antibiotics. Their average recuperation time was not significantly reduced. Therefore, all patients who are diagnosed with muscle strain would be well advised to take antibiotics as part of their treatment."
WORDS: 427          TIME: 01:26:05          DATE: 2010-7-5 23:06:30

The notion that antibiotics cure secondary infections after severe muscle strain so as to shorten the recuperation time seems an obvious conclusion at the first glance. After all, antibiotics are intended to kill bacteria to threat infections. However, the conclusion that taking antibiotics as part of the treatment shorten the healing time of patients with muscle strain may mask other, and potentially more significant causes of the different healing times, and may inspire doctors and patients' abuse of antibiotics.

First of all, as mentioned in the argument, there are two groups of patients anticipated in the study: the first group took antibiotics throughout their treatment; the second group was given sugar pills instead of antibiotics. The arguer presumes that sugar pills have no effect at all, so antibiotics may have some beneficial effect to infections as the first group recovered quicker than the second one on average.  However, there is absolutely no information provided about the effect of sugar pills. It is at least likely that sugar pills do harm to the healing from infections or/and muscle strain, and prevented patients in the second group from healing quickly.

Second, the argument is weakened by the fact that it does not concern about the differences between the two doctors -- Dr. Newland and Dr. Alton in the study. Dr. Newland is more professional on curing muscle strain whereas Dr. Alton has broader medicine knowledge. As we know that some psychological effect may be significant in treatment, it is possible that a patient recovers fast if he trusts in his doctor and believes the doctor's treatment is effective. So, maybe a specialized doctor, such as Dr. Newland, is more trustable to patients, especially those who are severely injured and recover quickly after his treatment. Also, an ineffective treatment, such as sugar pills, may reduce patients' trust in the doctor so as to lengthen the recuperation time.

The study does not take into account the different severity of muscle strain. Neither does it consider whether the anticipants were suffered from secondary infections. So it is not clear that whether the antibiotics help curing secondary infections after severe muscle strain or the muscle strain itself.

Finally, there is absolutely no evidence demonstrated that antibiotics have beneficial effects to all patients who are diagnosed with muscle strain. For example, a simple rest and less movement may have the same benefits as an expensive treatment of antibiotics. Before patients who suffer from muscle strain are recommended to use antibiotics as part of their treatment, a clear understanding of the beneficial effect provided by antibiotics is needed.

先谢谢后面的同学了!
作者: 奔跑的阿甘    时间: 2010-7-8 12:57:38

issue  165                    In any given field, the leading voices come from people who are motivated not by conviction but by the desire to respect opinions and ideas that differ from those held by majority.


The speaker asserts that the leading voices are from individuals who are motivated by the desire to respect opinions and ideas which are greatly different from those held by majority in any given field.It has merit from a normative standpoint, and I agree insofar as unique ideas are the root cause of innovations. However, this assertion is indisfensible since certain areas inherently call for convinction when it comes to others’ standpoints.
It is trule that the speaker’s assertion that the motivation for leaders is the desire to pesent unique opinions and ideas which the majority do not share in common is quite compelling. We cannot deny the fact that numerous great contributions to science and technology arise from ideas or opinions that sharply differ from the views held by the majority. The advances in this field only lead to the challenge to conventional theories. For example, both Newton and Einstain refused to accept certain physics laws of the day undiscrimately. To the contrary, they redefined these laws according to their new findings and understantding, which the majority of people did not share in common at that time. As it turns out, it is the Newton’s motivation law that contributes to the development of mechanics; it is the Einstain’s relative theory that makes great contributions to the progress of society. So does the field of arts. Those who eventually become the leading artists are those who departed from established rules of composition, such as Dali, Picasso and so forth. Besides, our most influential musicians seem to be different from the majority as well. Consider, for example, jazz pioneers—lonius monk and miles davis, who gave up all the harmonic laws, as well as folk musician poet—bob dalan, who built a new criteria for lyricism.

Beyond the concession, nevertheless, I disagree with the statement because ceratain other fields inherently require conviction in terms of the views held by the majority. Consider, for example, the politic area. The consequse will be entirely opposite when politicians present opinions that greatly differ from those held by general populace. After all, all citizens wish to lead a desirable life, opposing the average people’s position would to be opposite of happy life which will be surely rejected. The leading voices will come from people who can ensure every individual to enjoy better living standards.

Yet, important as originality and individuality are, to be different from the majority should not be a goal for its own sake, the goal should be the search for truth itself. Otherwise, it will be pretty dangerous. People will create bogus things without reasonable knowledge and theory, which are not innovations that make breakthrough compared with the past. Sometimes innovations might not be accepted by the people of the day, nonetheless, they are of potential values. Return to the example mentioned above, although einstain’s relative theories denied cerain conventional physics laws, they were created on the basis of the physics system. Einstain discovered the truth and thus gave birth to relative theory rather than the purpose for difference own sake. Eventually, as it turns out, scientists have proven their value.

In sum, I concede that the speaker’s assertion that leading voices often come from people who have unique standpoints is right, after all, original ideas might result in innovations. Nevertheless, the speaker unnecessarily extents its range to embrace all fields. In fact, it does not work in certain areas like politic. Moreover, we should not be different for the purse of difference own sake. Original views are based on truth.
作者: 奔跑的阿甘    时间: 2010-7-8 13:18:37

本帖最后由 奔跑的阿甘 于 2010-7-8 14:01 编辑

TOPIC: ARGUMENT51 - The following appeared in a medical newsletter.

"Doctors have long suspected that secondary infections may keep some patients from healing quickly after severe muscle strain. This hypothesis has now been proved by preliminary results of a study of two groups of patients. The first group of patients, all being treated for muscle injuries by Dr. Newland, a doctor who specializes in sports medicine, took antibiotics regularly throughout their treatment. Their recuperation time was, on average, 40 percent quicker than typically expected. Patients in the second group, all being treated by Dr. Alton, a general physician, were given sugar pills, although the patients believed they were taking antibiotics. Their average recuperation time was not significantly reduced. Therefore, all patients who are diagnosed with muscle strain would be well advised to take antibiotics as part of their treatment."
WORDS: 427          TIME: 01:26:05          DATE: 2010-7-5 23:06:30

The notion that antibiotics cure secondary infections after severe muscle strain so as to shorten the recuperation time seems an obvious conclusion at the first glance. After all, antibiotics are intended to kill bacteria to threat/prevent infections. However, the conclusion that taking antibiotics as part of the treatment shorten the healing time of patients with muscle strain may mask other, and potentially more significant causes of/for the different healing times/convalescence, and may(even) inspire/arouse doctors and patients' abuse of antibiotics.

First of all, as mentioned in the argument, there are two groups of patients anticipated/participated in the study: the first group took antibiotics throughout their treatment; the second group was given sugar pills instead of antibiotics. The arguer presumes that sugar pills have no effect at all, so antibiotics may have some beneficial effect to infections as the first group recovered quicker than the second one on average.  However, there is absolutely no information provided about the effect of sugar pills. It is at least likely that sugar pills do harm to the healing from infections or/and muscle strain, and prevented patients in the second group from healing quickly.

Second, the argument is weakened by the fact that it does not concern about the differences between the two doctors -- Dr. Newland and Dr. Alton in the study. Dr. Newland is more professional on/in curing muscle strain whereas Dr. Alton has broader medicine knowledge. As we know that some psychological effect may be significant in treatment, it is possible that a patient recovers fast if he trusts in his doctor and believes the doctor's treatment is effective. So, maybe a specialized doctor, such as Dr. Newland, is more trustable to patients, especially those who are severely injured and recover quickly after his treatment. Also, an ineffective treatment, such as sugar pills, may reduce patients' trust in the doctor so as to lengthen the recuperation time. (文中已经说了病人以为自己在服用抗生素)

The study does not take into account the different severity of muscle strain. Neither does it consider whether  anticipants/patients were suffered/suffer from secondary infections. So it is not clear that whether the antibiotics help curing/cure secondary infections after severe muscle strain or the muscle strain itself.

Finally, there is absolutely no evidence demonstrated that antibiotics have beneficial effects to all patients who are diagnosed with muscle strain. For example, a simple rest and/or less movement may have the same benefits as an expensive treatment of antibiotics. Before patients who suffer from muscle strain are recommended to use antibiotics as part of their treatment, a clear understanding of the beneficial effect provided by antibiotics is needed.

错误逻辑基本找到,还有一个是,作者认为二次感染延缓了康复,所以前提是大家都遭受了二次感染。另一个是病人的资料没有,包括性别、年龄、体质,也许抗生素不起作用,而仅仅是因为年轻以及身体棒而提前康复。
你的措词造句是否应当再提高点呢,整篇读下来不是很舒服。(恕我直言)
作者: 追梦小木耳    时间: 2010-7-8 18:02:52

本帖最后由 追梦小木耳 于 2010-7-8 18:07 编辑

站位改8楼

~~晕,8楼写的是issue~~
怎么办??
作者: 追梦小木耳    时间: 2010-7-8 18:04:08

我的14
The following appeared in a memo from the owner of Green Thumb Gardening Center, a small business serving a suburban town.

“There is evidence that consumers are becoming more and more interested in growing their own vegetables. A national survey conducted last month indicated that many consumers were dissatisfied with the quality of fresh vegetables available in supermarkets. And locally, the gardening magazine Great Gardens has sold out at the Village News stand three months in a row. Thus, we at Green Thumb Gardening Center can increase our profits by greatly expanding the variety of vegetable seeds we stock for gardeners this coming spring.”

Word:421

The argument appears to be logical to conclude that consumers are becoming interested in growing vegetables based on the evidence. However,the proofs provided are not sufficient and convincing. Therefore, the decision made by Green Thumb Gardening Center of expanding the variety of vegetable seeds in order to increase profits is hasty.

First, the evidence given in the argument are not warranted. There exist other explanations for the increasing number of complains and the popularity of the gardening magazines. A percentage of individuals’ dissatisfaction with the quality of fresh vegetables does not indicate their growing interest in growing vegetables by themselves. Perhaps, the purpose of the survey is to demonstrate the consumers’ desire toward the supermarkets to supply quality vegetables. Moreover, the selling out of the gardening magazine may be contributed by other factors rather than residens’ increasing interest in growing vegetables. Such factors include the excellent works of some new editors who made the magazine more attractive, or some promoting measures have been taken by the sellers in order to raise the sails.

Second, the way in which the survey was conducted is questionable. It is largely possible that most people involved in the survey were from big cities with large population. Most of them were bothered by the quality of vegetables in supermarkets, while in the suburban town, the case is totally different. Consequently, before making decisions ,  the gardening center should evaluate local situations and make surveys among the inhabitants rather than according to the nationwide survy.

Finally, it is hasty for the gardening center to make the decision to expand the variety of vegetables in order to enhance profits based on the conclusion. Even if it is true that larger proportion of residents are spending their spare time in growing vegetables, the gardening center needs to know which kinds of vegetables are most popular with their customers. Are they trying to fill their gardens with various vegetables or only crazy about one specific kind? Are there any other companies in the region who have been supplying vegetable seeds to the local people? Before any decisions are made, the gardening center must consider these factors. Otherwise, it would be difficult to make any profits.

To sum up, the argument fails to provide sufficient evidence to support the conclusion that a growing number of people are taking interest in growing vegetables. And the decision to increase profit lacks careful consideration. To make the argument forceful, the arguer needs to study the market deeply and include all possible alternatives
作者: 紫陌纤尘o0    时间: 2010-7-8 18:48:16

为了督促这个小朋友参加互改,那个我改了一篇可不可以发到这里呢?我已经PM他让他来这里改文了。
原帖链接https://bbs.gter.net/thread-1119673-1-2.html

ARGUMENT 181
181. From aletter to the editor of a city newspaper.
"Onerecent research study has indicated that many adolescents need more sleep thanthey are getting, and another study has shown that many high school students inour city are actually dissatisfied with their own academic performance. As away of combating these problems, the high schools in our city should beginclasses at 8:30 A.M.instead of 7:30 A.M.,and end the school day an hour later. This arrangement will give students anextra hour of sleep in the morning, thereby making them more alert and moreproductive. Consequently, the students will perform better on tests and otherassignments, and their academic skills will improve significantly."

The authorof the letter cites one research indicated that adolescents need more hours tosleep, and one study in high school of our city which showed that the studentsare not satisfied with their own academic performance. Hence the editorsuggested delaying one hour to start school and one hour to end. With this newarrangement, the editor claimed that the student will have an extra hour ofsleep, thereby are more productive during their study time. Therefore, the authorasserted the students will performance better on tests and significantlyimproves their academic skills. Carefully examination the supporting evidence,however, reveals that it tends little credible support to the author.



Firstly, accordingto the study, the students are not satisfied with their performance at school,but the author overlooks to provide us information of the students, who thoughtthey could perform better. For example whether they have sufficient hours ofsleep or not, or whether they complain about hours of sleep they have now andwant to have more time for sleeping. Moreover it is also entirely possible thatthey are strict with themselves and eager to make progress everyday. Withoutproviding the sleep information about the students who are not satisfied withthemselves and rolling out other possible reasons why they are not content withtheir performance, the author is hard to convince us.



Secondly, theauthor has confidence that the high school students will have one more hour tosleep by postponing one hour later to get to school. However, one hour’s delayto go to school can not make sure of one hour more sleep to students. Due to thedelay of end time of the school, the time for sleep, perhaps, is postponed aswell. For example, they spend the same time on finishing their assignments andthen go to bed, so they still can not obtain an extra hour of sleep. If theirwaking up time in the morning is significantly influenced by their biologicalclocks, which will be even worse, their sleeping time will be shorted by thenew arrangement. Hence, it is hard for us to accept that with the newarrangement, the students will have an extra hour of sleeping in the morning.

Finally, theauthor could not say that the student will dramatically increase their academicskills, although we conceded that students, with more hours of sleep, are morealert and more productive than before. It is widely known that the academicskills including how to find problems, using what kind of method to analyze andfinally solve the problems. Only more alert and productive is not enough to ensurestudents will improve their academic skills. Without providing that studentswill improve their capacity of finding, analyzing and solving the problems, itcan not convince us that more alert and productive will improve their abilityof academic skills.



In sum, theconclusion reached in this letter is invalid and misleading. To make itlogically acceptable, the author should provide us the evidence that thearrangement ensures one hour more sleeping time indeed. Moreover, the authorshould provide numerical result of the students' performance of academic skillscompared with old time arrangement. In addition, it, in this letter, shouldcontain sleeping information of students who are not pleased with their ownperformance, and the real reason of their discontent.



181. From a letter to the editor of a city newspaper.

"One recent research study has indicated that many adolescents need more sleep than they are getting, and another study has shown that many high school students in our city are actually dissatisfied with their own academic performance. As away of combating these problems, the high schools in our city should begin classes at 8:30 A.M. instead of 7:30 A.M., and end the school day an hour later. This arrangement will give students an extra hour of sleep in the morning, thereby making them more alert and more productive. Consequently, the students will perform better on tests and other assignments, and their academic skills will improve significantly."

蓝色为我做的结构标志词
绿色为我的建议
红色为有问题的地方
原则上语法句式错误,只做部分标记,主要看逻辑!
The author of the letter cites one research indicated that adolescents need more hours to sleep, and one study in high school of our city which showed that the students are not satisfied with their own academic performance.[建议表达方式改变一下] Hence the editor suggested delaying one hour to start school and one hour to end. With this new arrangement, the editor claimed that the student will have an extra hour of sleep, thereby are more productive during their study time. Therefore, the author asserted the students will performance better on tests and significantly improves their academic skills. Carefully examination the supporting evidence, however, reveals that it tends little credible support to the author.
开头写得很普通,基本是对题目的复述。给你推荐一个个人认为不错的开头方式,仅供参考。
https://bbs.gter.net/viewthread.php?tid=920961&highlight=


Firstly, according to the study, the students are not satisfied with their performance at school, but the author overlooks to provide us information of the students, who thought they could perform better. For example whether they have sufficient hour’s of sleep or not, or whether they complain about hours of sleep[这是要表达什么?] they have now and want to have more time for sleeping[这里最好是用sleep, for sleeping多是形容一个物体的,比如Only use your bed for sleeping.]. Moreover it is also entirely possible that they are strict with themselves and eager to make progress everyday. Without providing the sleep information about the students who are not satisfied with themselves and rolling out other possible reasons why they are not content with their performance, the author is hard to convince us.
[你看我蓝色标出的两个关于information的内容,这其实是两码事,要一致。
这段来看,你是要表达学生对成绩不满意或许与睡眠不足无关。论述说实话不是太有力,比如两句and前后表达的意思基本一样,这是写作为不是做填空,两边最好突出并列的关系。对于alternative explains的阐述还可以再深入些。
这里其实还有一个重要的前提,作者就认为学生成绩确实不好,而且与睡眠不足有关。你的文章中提到可能是学生对自己要求高,这也就是说未必真的成绩不好。这个点不该忽略。
语言方面还待提高,这是个慢功夫,要先避免中式英语。
]

Secondly, the author has confidence that the high school students will have one more hour to sleep by postponing one hour later to get to school. However, one hour’s delay to go to school can not[避免太绝对化的语言] make sure of one hour more sleep to students. Due to the delay of end time of the school, the time for sleep, perhaps, is postponed as well. [这里表达不清楚啊]For example, they spend the same time on finishing their assignments and then go to bed, so they still can not obtain an extra hour of sleep. If their waking up time in the morning is significantly influenced by their biological clocks, which will be even worse, their sleeping time will be shorted by the new arrangement.[表达不清楚] Hence, it is hard for us to accept that with the new arrangement, the students will have an extra hour of sleeping in the morning.
[这段是要说,改变上学的时间未必能延长睡眠。这个攻击点本是没有问题的,但是,论述有些问题。首先是逻辑连词的用的不好,一层层的关系没有体现出来;还有就是表达不清楚,让人不知所云;再者,论据要充实,这种题目的alternative explains要丰富,你想想不能延长睡眠难道只能有做作业这一个原因么?其实,这个原因也不好。这只能说是符合中国基本国情,美国的青少年没这么痛苦吧?你可以考虑下诸如,晚到学校未必晚起床啊,或许起来多玩一会儿,那么上课更累了,从而影响学习。我只是简单举个例子,你还可以多想想。]

Finally, the author could not say that the student will dramatically increase their academic skills, although we conceded that students, with more hours of sleep, are more alert and more productive than before. It is widely known that the academic skills including how to find problems, using what kind of method to analyze and finally solve the problems. Only more alert and productive is not enough to ensure students will improve their academic skills. Without providing that students will improve their capacity of finding, analyzing and solving the problems, it can not convince us that more alert and productive will improve their ability of academic skills.
[这里似乎有些跑题,这个题目的关键还是睡眠对其它方面的影响。你这里单是谈alert and more productive而抛开睡眠是不合适的,有些钻牛角尖的意味。抓住核心问题!]
In sum, the conclusion reached in this letter is invalid and misleading. To make illogically acceptable, the author should provide us the evidence that the arrangement ensures one hour more sleeping time indeed. Moreover, the author should provide numerical result of the students' performance of academic skills compared with old time arrangement. In addition, it, in this letter, should contain sleeping information of students who are not pleased with their own performance, and the real reason of their discontent.
[结尾就不看了哦]

LZ对题目的分析把握还是比较到位的。此题是由两个study结果的结合来产生出作者的猜想,从而引出解决方案。那么你的入手点应该是先把作者认为的两个study的必然联系解除,这点你在第二段做到了,但是要更加有说服力!这样它的前提是攻破了。接着就说它的应对方案的问题,你在这里的论述还要加强!然后就其方案的预期结果来说,它的预期结果是,睡的多了will perform better on tests and other assignments, and their academic skills will improve significantly,那你就说它睡得多也未必能达到这样的效果。
整体看作为新手写得还是蛮不错的,在语言方面要加强,多看看范文,看人家是如何展开论述的。加油!

作者: tyarel    时间: 2010-7-8 21:17:50

本帖最后由 tyarel 于 2010-7-9 20:58 编辑

改11L的Argument 14


The argument appears to be logical to conclude that consumers are becoming interested in growing vegetables based on the evidence. Howeverthe proofs provided are not sufficient and convincing. Therefore, the decision made by Green Thumb Gardening Center of expanding the variety of vegetable seeds in order to increase profits is hasty.

First, the evidence given in the argument
are(is) not warranted. There exist other explanations for the increasing number of complains and the popularity of the gardening magazines. A percentage of individuals’ dissatisfaction with the quality of fresh vegetables does not indicate their growing interest in growing vegetables by themselves. Perhaps, the purpose of the survey is to demonstrate the consumers’ desire toward the supermarkets to supply quality vegetables. Moreover, the selling out of the gardening magazine may be contributed by(to) other factors rather than residents’ increasing interest in growing vegetables. Such factors include the excellent works of some new editors who made the magazine more attractive, or some promoting measures have been taken by the sellers in order to raise the sails(sales)
.

Second, the way in which the survey was conducted is questionable. It is largely possible that most people involved in the survey were from big cities with large population.
Most of them were bothered by the quality of vegetables in supermarkets(
这句感觉有点奇怪,是想表达“他们确实对质量不满”?), while in the suburban town, the case is totally different. Consequently, before making decisions ,  the gardening center should evaluate local situations and make surveys among the inhabitants rather than according to the nationwide survy(survey).

Finally, it is hasty for the gardening center to make the decision to expand the variety of vegetables in order to enhance profits based on the conclusion. Even if it is true that larger proportion of residents are spending their spare time in growing vegetables, the gardening center needs to know which kinds of vegetables are most popular with their customers. Are they trying to fill their gardens with various vegetables or only crazy about one specific kind? Are there any other companies in the region who have been supplying vegetable seeds to the local people? Before any decisions are made, the gardening center must consider these factors. Otherwise, it would be difficult to make any profits.

To sum up, the argument fails to provide sufficient evidence to support the conclusion that a growing number of people are taking interest in growing vegetables. And the decision
to increase profit(the variety of vegetable seeds
这个是decision的内容吧?这里to后面的内容理解上有可能产生歧义) lacks careful consideration. To make the argument forceful, the arguer needs to study the market deeply and include all possible alternatives.

总的来说,点抓得还是蛮准的,思路很清晰。个人感觉第一段可以拆分开来,原文中survey和杂志本来就是作为2个不同的论据的,所以还是分开分析比较恰当,并一块感觉有点混乱。
然后稍有些拼写的小错误,当心一点就是了,比我好多了=.=
最后对本人的分析提一点我自己的看法

对于survey,即使确实可以表达居民对于超市蔬菜的不满,也不能说明他们会自己种菜。可能选择别的途径,市场?或者还是在超市买,因为比自己种方便?
对于杂志脱销,也可以有别的原因,比如当地杂志市场竞争不强?该类杂志只有这么一种?或者杂志本身进货量就非常非常小?脱销完全不说明它在该地居民中广受欢迎。即使受欢迎也可能是别的原因造成的。
对于survey的代表性问题,还可以质疑结果当中many的概念,太模糊?然后就是对于自己种蔬菜是什么态度调查也没提及?另外就是菜价调查的人数,普遍性的问题了。


这些就是我在看到这篇argument的题干时另外想到的反驳内容,权当小小的补充希望有用
作者: tyarel    时间: 2010-7-8 21:19:09

TOPIC: ARGUMENT26 - The following appeared in a memo from the chairperson of the school board in the town of Saluda.



"For the past five years, Mr. Charles Schade has been the music director at Steel City High School, and during that time the school band from Steel City High has won three regional band competitions. In addition, the quality of the music rehearsal facilities and musical instruments at Steel City High has improved markedly over the past five years. Because of such successes at Steel City High, the Saluda school board should hire Mr. Schade to plan and direct the general music education programs for the entire Saluda school system."

WORDS: 460         TIME: 00:46:23          DATE: 2010/7/8 20:28:10



Based on the success of the school band and the improvement of music facilities quality during Mr. Charles 5 years' duration in Steel City High School (SCHS), the author recommended Mr. Schade to the Saluda school as the director of the general music education programs. Yet, close scrutiny all the evidence mentioned in the argument reveals that it is not as convincing as it seems at first glance.



Foremost, the author attributed the school band's success to Mr. Schade, which is not persuasive actually. Maybe the band is so excellent that it is always one of the best bands in the area and Mr. Charles did little to the band, which cannot show his talent and capacity to lead and direct the band. What's more, the author failed to illustrate the equal level of the bands in the area. It is entirely likely that all the bands are terrible, or at least not good enough, and that's why the band of SCHS can win so many times. With so many potential contributors, it is tough to assess what Mr. Schade contributed to the school band of SCHS.



Moreover, another fact the author stated as evidence to support his suggestion is also not that potent. Admittedly, the quality of music facilities has been improved. However, the author didn't exclude other factors which may result in the development either. Perhaps the school board decided to develop the school band and it is them, not Mr. Schade, who made the decision and spent a great amount of money to better the instruments. Besides that, all these new instrument may be donated by the musical instrument company trying to advertise through the band. That’s why it is too rashly to conclude that the improvement is Mr. Charles's contribution.



Even if all addressed above are what Mr. Schade did for the SCHS, it is not reasonable to draw the conclusion that he will fit the position in the Saluda school. No evidence has been shown to point out that Mr. Schade has experience of planning and directing general music education programs for the entire school system. The job is harder and more complicated, compared with what his former one. The problems the author pointed out that Mr. Schade handled perfectly is quite different from what he will face in the new position. He probably just can tackle the problems of a school band and have no energy to direct the entire music education program. Hence, lacking the evidence that Mr. Schade did the same work great, I cannot be persuaded that he will be proper to the position.



All in all, the author didn't list the enough evidence to bolster his recommendation and the argument will be strengthened if he can offer more facts to demonstrate that Mr. Schade is talented and experienced enough to direct the music education program for Saluda school.
作者: yanii    时间: 2010-7-9 06:45:12

改14楼
Based on the success of the school band and the improvement of music facilities quality during Mr. Charles 5 years' duration in Steel City High School (SCHS), the author recommended Mr. Schade to the Saluda school as the director of the general music education programs. Yet, close scrutiny(scrutinizing) all the evidence mentioned in the argument reveals that it is not as convincing as it seems at first glance.

Foremost, the author attributed the school band's success to Mr. Schade, which is not persuasive actually.(知道你想说什么,但此处用which不好,指代不明确) Maybe the band is so excellent that it is always one of the best bands in the area and Mr. Charles did little to the band, which cannot show his talent and capacity to lead and direct the band. What's more, the author failed to illustrate the equal level of the bands in the area. (?) It is entirely likely that all the bands are terrible(their skill), or at least not good enough, and that's why the band of SCHS can win so many times. With so many potential contributors, it is tough to assess what (whether) Mr. Schade contributed to the school band of SCHS.

Moreover, another fact the author stated as evidence to support his suggestion is also not that potent. Admittedly, the quality of music facilities has been improved. However, the author didn't exclude other factors which may result in the development either. Perhaps the school board decided to develop the school band and it is them, not Mr. Schade, who made the decision(是不是想说不是Mr.Schade 的功劳) and spent a great amount of money to better the instruments. Besides that, all these new instrument may be donated by the musical instrument company trying to advertise through the band. That’s why(怎么和前面的连接) it is too rashly to conclude that the improvement is Mr. Charles's contribution.

Even if all addressed above are what Mr. Schade did for the SCHS, it is not reasonable to draw the conclusion that he will fit the position in the Saluda school. No evidence has been shown to point out that Mr. Schade has experience of planning and directing general music education programs for the entire school system. The job is harder and more complicated, compared with what (删)his former one. The problems the author pointed out that Mr. Schade handled perfectly is quite different from what he will face in the new position(the problem 那儿前半句看不明白). He probably just can tackle the problems of a school band and have no energy to direct the entire music education program. Hence, lacking the evidence that Mr. Schade did the same work great(could do a tougher work equally perfectly), I cannot be persuaded that he will be proper to the position.

All in all, the author didn't list the enough evidence to bolster his recommendation and the argument will be strengthened if he can offer more facts to demonstrate that Mr. Schade is talented and experienced enough to direct the music education program for Saluda school.
(思路清晰,论证角度找得较全面,只是个别地方语言表达有些问题)
作者: yanii    时间: 2010-7-9 06:46:25

TOPIC: ARGUMENT237 - The following appeared as part of an article in a local Beauville newspaper.

"According to a government report, last year the city of Dillton reduced its corporate tax rate by 15 percent; at the same time, it began offering relocation grants and favorable rates on city utilities to any company that would relocate to Dillton. Within 18 months, two manufacturing companies moved to Dillton, where they employ a total of 300 people. Therefore, the fastest way for Beauville to stimulate economic development and hence reduce unemployment is to provide tax incentives and other financial inducements that encourage private companies to relocate here."
WORDS: 411
TIME: 01:00:00
DATE: 2010-7-8 11:29:04

The enthusiastic author recommends a fastest way for Beauville (Be) to stimulate economy and reduce unemployment. His supporting evidences, the data during the past 18 months, an analogy between Dillton (Di) and Be, and a assertive assumption, is susceptible to query, rendering the conclusion based on them stands scant closer examination.

To begin with, is the Di model a real success? Merely the data collected within 18 months doesn't necessarily indicate an increase after the relocating private companies. Maybe there are other factors in Di rather than the new policies that appeal to them, say available raw materials, potential market shares for them, convenient transportation to other markets, the particularly cheap labor and so forth. As to the 300 employees in the two companies, are they jobless local residents or immigrants along with the companies, or native skill workers that leap from the other local plants? If it is in the last two events, the relocation of companies contributes little to reducing local unemployment.

Even if the new measures function well in Di, it is likely not the case in Be. What a company emphasizes when choose location is not only some new policies trying to allure them, but also other in-depth factors, such as the development space for it, the number of potential consumers, the cost of local labor, the prosperity of local market and the completeness of its infrastruction. All those should be given equal priority. In other words, Be may not be the first choose regardless its attractive tax incentives and other financial inducements.

Finally, the author assumes that there is not other way to boost economy other than attracting private companies to Be. For example, aimed at developing local companies, expanding their market shares and therefore recruit more employees, may works equally well, if not better. In addition, if Be is a beautiful city, or be famous for its unique culture, why not enhance the relevant infrastruction to allure more tourists, who may bring about piles of money?

Though well-intentioned, the author provides insufficient evidences to support his specious recommendation and cannot convince even me. To appeal to the masses, he should collect more statistics showing that a number of private companies managers will prefer Be if it adopt positive economic polices towards the relocation of companies. Additionally, he can survey the local tourist market and its appeal to non-residents, to conclude whether this can be a green way to stimulate local economy.
作者: 费话先生    时间: 2010-7-9 10:02:24

本帖最后由 费话先生 于 2010-7-9 10:50 编辑

占楼 改16楼的
TOPIC: ARGUMENT237 - The following appeared as part of an article in a local Beauville newspaper.
"According to a government report, last year the city of Dillton reduced its corporate tax rate by 15 percent; at the same time, it began offering relocation grants and favorable rates on city utilities to any company that would relocate to Dillton. Within 18 months, two manufacturing companies moved to Dillton, where they employ a total of 300 people. Therefore, the fastest way for Beauville to
stimulate economic development and hence reduce unemployment is to provide tax incentives and other financial inducements that encourage private companies to relocate here."
WORDS: 411
TIME: 01:00:00
DATE: 2010-7-8 11:29:04


The enthusiastic author recommends a fastest way for Beauville (Be) to stimulate economy and reduce unemployment. His supporting evidences, the data during the past 18 months, an analogy between Dillton (Di) and Be, and a assertive assumption,
is (
应该改成are )susceptible to query, rendering the conclusion based on them stands scant closer examination.

To begin with, is the Di model a real success?
(嗯,问句开头做中心句我觉得有点风险,毕竟第一句得把之后的观点都有个大览的作用,比如提下这个错误是啥呀,to begin with, the assumption that di is successful in stimulating economic development is un substantiated Merely the data collected (这个语序貌似不太对呀,若是我会写
the mere data collected during 18 mouths….within 18 months doesn't necessarily indicate an increase after the relocating(用名词会好一点,relocation动词Ing放在前面不对啊,要么放后面) private companies. Maybe there are other factors in Di rather than the new policies that appeal to them, say available raw materials, potential market shares for them, convenient transportation to other markets, the particularly cheap labor and so forth. (我总觉得有点怪怪的,感觉不太正规呀,最后一个并列短语得用and吧,而且这种全部并列式的他因看上去给人印象不深刻呀,拆成两句话会好一些,毕竟这一句才是具体的展开重点呀)As to the 300 employees in the two companies, are they jobless local residents or immigrants along with the companies, or native skill workers that leap from the other local plants? If it isit is what
TRUE 或者 the case 之类的吧,而且it代指不明呀) in the last two events(不应该用event吧,situation我比较倾向用), the relocation of companies contributes little to reducing local unemployment.

Even if the new measures function well in Di, it is likely not the case in Be. What a company emphasizes when choose location is not only some new policies trying to allure them, but also other in-depth factors, such as the development space for it, the number of potential consumers, the cost of local labor, the prosperity of local market and the completeness of its infrastruction. All those should be given
equal priority
(为啥,我觉得这就是一个gratuitous assertion equal priority从何而来,都要考虑跟同等地位的优先程度不一样吧~这句话有逻辑漏洞呀,只能说should not ignore the above factors . In other wordsin other Words 个人觉得不适合用作小结尾,因为本身它是对上面话的进一步解释或paraphrase,没有总结的意义在), Be may not be the first choose regardless its attractive tax incentives and other financial inducements.

Finally, the author assumes that there is not
other
another way to boost economy otherrather than attracting private companies to Be. For example(这个词不是列举他因的标志诶,for example 后面跟的一定是具体事例,是真的是一个正儿八经的例子。), aimed at developing local companies, expanding their market shares and therefore recruit more employees, may works equally well, if not better. In addition, if Be is a beautiful city, or be famous for its unique culture, why not enhance the relevant infrastruction(这个词。。。好像没有。。。) to allure more tourists, who may bring about piles of money?

Though well-intentioned, the author provides insufficient evidences to support his specious recommendation and cannot convince even me. To appeal to the masses, he should collect more statistics showing that a number of private companies managers will prefer Be if it adopt positive economic polices towards the relocation of companies. Additionally, he can survey the local tourist market and its appeal to non-residents, to conclude whether this can be a green way to stimulate local economy.

总评:我觉得逻辑和攻击点还是很清晰的,有些表达也让我眼前一新,没有很模板化。但是美中不足的是,他因总是喜欢一溜就列举完了,一句话就搞定了,让人有点欲罢不能呀,不够well-developed.语言流畅度方面还是要注意一下,有些句子有点绕,不太顺流,我可能语言比较小白,所以语言方面我无法提供更好的建议,只觉得流畅是第一位的,我初读笔者的文章,有些迟滞~我想这一点可能会影响到笔者的分析吧。另外,我觉得他因列举得很详实,结构亦很清晰,不错。
作者: 费话先生    时间: 2010-7-9 10:06:04

先谢谢帮我认真修改的版友~第一篇argument,希望多提缺点~
TOPIC: ARGUMENT14 - The following appeared in a memo from the owner of Green Thumb Gardening Center, a small business serving a suburban town.
"There is evidence that consumers are becoming more and more interested in growing their own vegetables. A national survey conducted last month indicated that many consumers were dissatisfied with the quality of fresh vegetables available in supermarkets. And locally, the gardening magazine Great Gardens has sold out at the Village News stand three months in a row. Thus, we at Green Thumb Gardening Center can increase our profits by greatly expanding the variety of vegetable seeds we stock for gardeners this coming spring."
WORDS: 533
TIME: 00:45:00
DATE: 2010-7-9 9:17:58


The author predicts than Green Thumb Gardening Center can increase profits by greatly expanding the variety of vegetable seeds stocked for gardeners. To demonstrate his prediction, the author cites a national survey and good sales of a local gardening magazine. However, either the national survey or the good sales of magazine can lead little support to his prediction.

To begin with, it's unreliable to deduce that consumers are becoming more interested in growing vegetables themselves from the national survey. First, those who are dissatisfied with the quality of fresh vegetables are only a small part of consumers and their comments can be negligible to represent the majority of consumers. Second, even if most of consumers are really dissatisfied with the vegetables, it is not safe to say that the consumers are interested in growing their own vegetables. The purpose of crying out may be attracting the managers' attention, so they can improve their services. It’s like consumers aren't satisfied with some services doesn't mean they want serve themselves. Third, the author's assumption that the results of national survey can also manifest the trend in local is unwarranted. Perhaps the local residents are very satisfied with the vegetables in supermarkets because the supermarket's supplier is quality guaranteed.

The author's further assumes that the good sales of gardening magazine can indicate the popularity of residents’ self-growing vegetables. However, this may be not the case. It's possible that those who buy the magazine are interested in growing flowers rather than vegetables. Anyway, there is no evidence that the magazine is mainly about vegetables growing. The assumption will be poor-reasoned if there is a little information about how to grow vegetables. Besides, the author doesn't provide any statistics on the specific volume of the magazine. If the circulation is rather small, the magazine is also possible to be sold out even if there are only a few people who like it. Again, three mouths data may be not sufficient to draw any conclusions from it. Perhaps during the three mouths there appears an influx of tourists who buy the magazine to kill time and hence increase the sales of the magazine. Without ruling out other possible factors which may influence the sales of magazine, the author cannot draw the conclusion that local residents are more likely to buy the magazine, let alone grow vegetables themselves.
Suppose that the above evidence can convincingly prove that local people do much prefer to grow vegetables than before, the suggestion that Green Thumb Gardening Center can increase profits by greatly expanding the variety of vegetable seeds stocked may turn out to be ineffective. The climate in the town may be only suitable for a few certain sorts of vegetables. So if the owner doesn’t take the viability into account, the decision to expand the variety of seeds may lead nobody to buy.

In conclusion, the argument is not cogent enough to give the prediction. If the author want to demonstrate the future market of his company are really with wide prospect, he should provide evidence that the local people are clearly more desirable to grow vegetables and the suggestion to expand the variety of seeds are effective in attracting the consumers.

作者: yanii    时间: 2010-7-9 18:26:42     标题: 占18楼的argument修改

本帖最后由 yanii 于 2010-7-9 19:14 编辑


The author predicts thanthat Green Thumb Gardening Center can increase profits by greatly expanding the variety of vegetable seeds stocked for gardeners. To demonstrate his prediction, the author cites a national survey and good sales of a local gardening magazine. However, either the national survey or the good sales of magazine can lead little support to his prediction.

To begin with, it's unreliable to deduce that consumers are
becoming more interested in growing vegetables themselves from the national survey. First, those who are dissatisfied with the quality of fresh vegetables are only a small part of consumers and their comments can be negligible to represent the majority of consumers. Second, even if most of consumers
most consumersmost of the consumers are really dissatisfied with the vegetables, it is not safe to say that the consumers are interested in growing their own vegetables. The purpose of crying out may be attracting the managers' attention, so they can improve their services. (该句太突兀)It’s likely consumers aren't satisfied with some services doesn't mean they want serve themselves. Thirdly, the author's assumption that the results of national survey can also manifest the trend in local is unwarranted.Nice!) Perhaps the local residents are very satisfied with the vegetables in supermarkets because the supermarket's supplier is quality guaranteed.

The author's further assumes that the good
high sales of gardening magazine can indicate the popularity of residents’ self-growing vegetables. However, this may be not the case. It's possible that those who buy the magazine are interested in growing flowers rather than vegetables. Anyway, there is no evidence that the magazine is mainly about vegetables growing. The assumption will be poor-reasoned if there is a little information about how to grow vegetables. Besides, the author doesn't provide any statistics on the specific volume of the magazine. If the circulation is rather small, the magazine is also possible to be sold out even if there are only a few people who like it. Again, three mouths data may
be not sufficient (insufficient) to draw any conclusions from it. Perhaps during the three mouths there appears an influx of tourists who buy the magazine to kill time and hence increase the sales of the magazine. Without ruling out other possible factors which may influence the sales of magazine, the author cannot draw the conclusion that local residents are more likely to buy the magazine, let alone grow vegetables themselves.


Suppose that the above evidence can convincingly prove that local people do much prefer to grow vegetables than before(
不是不能推出吗Even suppose that local people do prefer to grow vegetable themselves more than before) , the suggestion that Green Thumb Gardening Center can increase profits by greatly expanding the variety of vegetable seeds stocked may turn out to be ineffective. The climate in the town may be only suitable for a few certain sorts of vegetables. So if the owner doesn’t take the viability into account, the decision to expand the variety of seeds may lead nobody to buy. (是该决定导致没有人买吗?)

In conclusion, the argument is not cogent enough to
give the prediction. If the author want to demonstrate the future market of his company are really with wide prospect,
(有点怪,不太明白)he should provide evidence that the local people are clearly more desirable to grow vegetables and the suggestion to expand the variety of seeds are effective in attracting the consumers.

总结:思路很清晰,结构、文笔都不错,词汇量也很大;对他因的思考也比较合理且全面,赞!
      个别地方选词不太准确,有些表达也小有问题。
      再改改就成型了,再接再厉!

PS:谢谢对我的修改呀!很有启发。
作者: yanii    时间: 2010-7-9 19:16:35

先谢谢将要辛苦修改的楼友们:)
TOPIC: ARGUMENT163 - The following is taken from the editorial section of the local newspaper in Rockingham.

"In order to save a considerable amount of money, Rockingham's century-old town hall should be torn down and replaced by the larger and more energy-efficient building that some citizens have proposed. The old town hall is too small to comfortably accommodate the number of people who are employed by the town. In addition, it is very costly to heat the old hall in winter and cool it in summer. The new, larger building would be more energy efficient, costing less per square foot to heat and cool than the old hall. Furthermore, it would be possible to rent out some of the space in the new building, thereby generating income for the town of Rockingham."

The editorial recommends the Rockingham's (Rh's) old town hall be replaced by a larger new one for saving money. However, its supporting evidences - several plausible assumptions and an assertive suggestion - are open to query, rendering it amount to a poor advice.

Turn to the first argument that the old town hall is too small to comfortably accommodate the town’s servants. What is the argument based on? Especially the vague word "comfortably"? Is it on complaints of some particular employees, whose concern is inexorable on being comfortable? Or the announcement of the authorities
without sufficient investigation on the hall and just taking it as an opportunity to improve their office conditions? If so, the civilian of Rh must be on guard not be deceived. On the other side, if the old hall is really not large enough to hold so many employees, purchasing some floors of established apt building or just constructing a small one to enlarge the gross office space both may be good choices. On the contrary, building a completely new one will inevitably waste a considerable amount of money rather save any.


Moreover, the editor assumes that the heat and cold fee is inefficient and costly
in the old town, and only by building a new one can they by solved. Yet, he overlooks or ignores intentionally the alternative solution. Through redecoration and replacing the old inefficient appliance, it may be equally efficient in the old hall. Besides, even new building may be more energy efficient in per square, the total fee of it may offset, if not exceed its benefits.


As for the editor's suggestion, renting out some of the space, in the would-be new building, thus generating income for the town is particularly ridiculous. Will the government build an excessive large one to spare space for rent or will it build a sizeable one to accommodate its employees? In the first situation, a higher front-end investment would be extremely high and may beyond the government can afford; If the government really want to expand income by renting rooms, why don’t they just build some? Does this consideration have any bearing with replacing the old town hall? On the second situation, there would be little space, if any, to rent, and the expanding income would be too small to mention. Additionally, what about the local rent market?
Is it prosperous or floundering? Absent such information, it's likely that the extra space will be vacant for a long time, without efficient utility, let alone expanding income.


As long as there is a common sense, all know that this editorial may be nothing but a propaganda for some intentional people, and the ignorant resident should be wary of being hoaxed. To rebut my conclusion and bolster the original one, the editor needs to provide more information about how come the old town hall is too small, and that the redecoration and replace of old heat and cold system may cost as much, or even more. As to the space of the new building, the government should consult with the taxpayers.
作者: koyoi    时间: 2010-7-9 20:48:09

本帖最后由 koyoi 于 2010-7-10 00:09 编辑

占20楼修改

这个是语法问题

这个是句子修善问题

这个是内容相关问题

TOPIC: ARGUMENT163 - The following is taken from the editorial section of the local newspaper in Rockingham.

"In order to save a considerable amount of money, Rockingham's century-old town hall should be torn down and replaced by the larger and more energy-efficient building that some citizens have proposed. The old town hall is too small to comfortably accommodate the number of people who are employed by the town. In addition, it is very costly to heat the old hall in winter and cool it in summer. The new, larger building would be more energy efficient, costing less per square foot to heat and cool than the old hall. Furthermore, it would be possible to rent out some of the space in the new building, thereby generating income for the town of Rockingham."

The editorial recommends the Rockingham's (Rh's) old town hall be replaced by a larger new one for saving money. However, its supporting evidences - several plausible assumptions and an assertive suggestion - are open to query, rendering it amount to a poor advice.

(To) Turn to the first argument that the old town hall is too small to comfortably accommodate the town’s servants (开头用不定式,因此这不是一个完整的句子). What is the argument based on? Especially the vague word "comfortably"?(这两句虽然语气比较强烈但是语法不那么规范尤其后一句只是一个句子成分但是被单独成句显得比较口语化)  Is it on complaints of some particular employees whose concern is (are) inexorable on being comfortable? Or (不应断句) the announcement of the authorities without sufficient investigation on the hall and just taking it as an opportunity to improve their office conditions? If so, the civilian of Rh must be on guard not be deceived (against being deceived). On the other side, if (建议直接使用even through之类的转折替换on the other side, ifthe old hall is really not large enough to hold so many employees, purchasing some floors of established apt buildings or just constructing a small one to enlarge the gross office space both (both 提到purchasing前或者放到be后) may be good choices. On the contrary, building a completely new one will inevitably waste a considerable amount of money rather than saving any (any的感觉稍微有点怪).

Moreover, the editor assumes that the heat and cold fee is inefficient (用词不当,费用不存在有无效率,而应该是费用的使用有无效率) and costly in the old town, and only by building a new one can they be solved. Yet, he overlooks or ignores intentionally the alternative solutions (或许复数会更好?以及你如何确定他是intentionally?). Through redecoration and replacing (建议词性统一 redecorating and replacing 或者redecoration and replacement of) the old inefficient appliance(稍微觉得redecoration和appliance的对应不太妥当), it may be equally efficient in the old hall(可以明白equally指代的比较双方是Old hall和new hall,但是整句是在围绕old,所以我觉得这里用become会更好). Besides, even the new building may be more energy efficient in per square, the total fee of it may offset (by what? Give examples) , if not exceed its benefits.

As for the editor's suggestion, renting out some of the space in the would-be new building, thus generating income for the town is particularly ridiculous (用ridiculous过于尖锐,我们仍然不能否认有这种可能性,我们要否认的应该是editor这种过于乐观理想的想法). Will the government build an excessively large one to spare space for rent or will it build a sizeable one to accommodate its employees? In the first situation, a higher front-end investment would be extremely high and may beyond the government can afford(觉得语法上或许有问题?我不确定); If the government really want to expand income by renting rooms, why don’t they just build some? Does this consideration have any bearing with replacing the old town hall? On the second situation, there would be little space, if any, to rent, and the expanding income would be too small to mention. Additionally, what about the local rent market? Is it prosperous or floundering (not academic) ? Absent such information, it's likely that the extra space will be vacant for a long time (我们的确是缺乏当地租房市场的信息,但这不等于当地租房市场不景气,因此用vacant是不恰当的), without efficient utility, let alone expanding income (口语化).

As long as there is a common sense (太绝对,如何确定这是common sense), all know that this editorial may be nothing but a propaganda for some intentional people, and the ignorant resident should be wary of being hoaxed. To rebut my conclusion and bolster the original one, the editor needs to provide more information about how come the old town hall is too small, and that the redecoration and replace of old heat and cold system may cost as much, or even more. As to the space of the new building, the government should consult with the taxpayers.

优:用transition words用得都蛮好蛮到位的~读起来比较流畅,而且没有模板化的感觉和什么严重的语法错误,找到的漏洞也都蛮准~(鉴于我自己水平也不高咳……所以如果有其他的我也木有看出来……抱歉~)

缺:关于攻击的第一点,觉得有点罗嗦,尤其是在2种具体情况的列举上花费了较多心思,其实可以考虑更精简的表达方式。关于第二点,我觉得后面的一点(也就是new hall的花费甚至会超过old hall)不够具体,是蛮重要的一个点,但被一笔带过了。第三点的问题我已经在文中标出。


感觉文章叙述比较激烈,用了许多反问句,但我个人觉得ARGU应该是比较冷静的分析逻辑漏洞并且驳斥,而不是总是用反问再去抛出问题,这里除了加强语气对分析问题似乎没有很大帮助,因为反问句的低效率,使得需要用更多的句子去驳斥观点,就会显得文章有点罗嗦。另外在一些地方过于主观,比如intentionally和有关第三点的驳论。

同时感觉童鞋有点先入为主地觉得这是propaganda,所以就觉得是cheat,在这点上强调得有点过多了,我觉得它可以作为一种猜测的可能,但不应成为一种比较强烈的观点倾向。作为最后一段,一般是如何改善避免这种逻辑漏洞(即让陈述变得合逻辑),但是开头两行变成了一种类似呼吁广大民众不要受骗上当的感性建议(即攻击陈述本身道义情理层面上的东西),换句话说,这其实也就是你把他的陈述假想为无漏洞的(因为如果没有漏洞,它就不会被认为是propaganda),而这本身与我们要找其逻辑漏洞的任务是相反的。

因为我也是初手所以可能在改的时候有些抓不到重点,在一些不必要纠结的地方吹毛求疵还请见谅~^^||

作者: koyoi    时间: 2010-7-11 21:26:04

7.The following appeared in a letter to the editor of the Clearview newspaper.

"In the next mayoral election, residents of Clearview should vote for Ann Green, who is a member of the Good Earth Coalition, rather than for Frank Braun, a member of the Clearview town council, because the current members are not protecting our environment. For example, during the past year the number of factories in Clearview has doubled, air pollution levels have increased, and the local hospital has treated 25 percent more patients with respiratory illnesses. If we elect Ann Green, the environmental problems in Clearview will certainly be solved."


The argument claims that residents of Clearview should support the member of Good Earth Coalition, Ann Green, to be the next mayor for the purpose of preventing the further deterioration of the environment. To support this recommendation, the editorial enumerated three phenomena: the double-increasing amount of the factories, an increased degree of the air pollution and an increase in the number of patients with respiratory illnesses by 25 percent which are all due to the current council’s negative attitude to the environment protection. However, the editorial suffers from several problems which make it less credible.
  To begin with, the figure of a member of Good Earth Coalition (GEC) do not necessarily evince Green will take measures to solve environment pollution problems. Even he exactly takes measures, whether the measures perform effective cannot be ensured. Similarly, the identity of Braun, a member of the town council, does not infer he support current council’s action and will do nothing to solve the problem of pollution after he takes the power. It is possible that his proposal of taking more care of the environment was not adopted by the council.

Additionally, the examples stated actually serve little to accuse the current council due to a lack of significant evidence that can prove a conspicuous link among the increasing amounts of the factories and more patients with respiratory illnesses and the policies that council has adopted. Other factors, such as the emergence of a large amount of new commercial opportunities that automatically attract more investment to build new factories and the aberrant weather which caused an increased respiratory disease rate last year, can also lead to the three phenomena, those of which, however, cannot be simply attributed to the council and may not eliminate even Ann Green is elected.

Aside from the above two points, the possibility of an alternative way also could be considered. Even if the ability of Green in solving pollution problems is confirmed and really better than that of Braun, and a significant link between the actions of current council and the pollution problems and ensuing diseases exists, there may be some other candidates who have better performances as Ann Green should not be the necessary and only one.

In conclusion, the claim is unpersuasive as it stands. To support the editorial, strong evidence, such as more reliable and direct data that reflect the link between the current council’s action and pollution problems, is needed to prove that the current council accounts for them. Simultaneously, researches related to the Green’s and Braun’s positions and experience on relevant issues also should be done for showing that Green has the distinct ability in tackling pollution problems and voting him is the best way to solve pollution problems.
作者: 396857815    时间: 2010-7-12 16:39:58     标题: 占位修改21楼

本帖最后由 396857815 于 2010-7-12 18:46 编辑

7.The following appeared in a letter to the editor of the Clearview newspaper.

"In the next mayoral election, residents of Clearview should vote for Ann Green, who is a member of the Good Earth Coalition, rather than for Frank Braun, a member of the Clearview town council, because the current members are not protecting our environment. For example, during the past year the number of factories in Clearview has doubled, air pollution levels have increased, and the local hospital has treated 25 percent more patients with respiratory illnesses. If we elect Ann Green, the environmental problems in Clearview will certainly be solved."


The argument claims that residents of Clearview should support the member of Good Earth Coalition, Ann Green, to be the next mayor for the purpose of preventing the further deterioration of the environment. To support this recommendation, the editorial enumerated three phenomena: the double-increasing amount of the factories, an increased degree of the air pollution and an increase in the number of patients with respiratory illnesses by 25 percent which are all due to the current council’s negative attitude to the environment protection. However, the editorial suffers from several problems which make it less credible.
本段复述原文,并指出原文中有错。

  To begin with, the figure of a member of Good Earth Coalition (GEC) do not necessarily(这个词用在这里有些别扭可以改为 do not well evince )evince Green will take measures to solve environment pollution problems. Even he exactly takes measures, whether the measures perform effective cannot be ensured. (这句让步说明即使她的GEC身份可以推出他会采取措施,他的措施也不一定会有效。)Similarly, the identity of Braun, a member of the town council, does not infer he support current council’s action and will do nothing to solve the problem of pollution after he takes the power. It is possible that his proposal of taking more care of the environment was not adopted by the council.最后一句这个举例说明很到位,很必要而且表述的也不赘余。

Additionally, the examples stated actually serve little to(抽象词使用很熟练) accuse the current council due to a lack of significant evidence that can prove a conspicuous link among the increasing amounts of the factories and more patients with respiratory illnesses and the policies that council has adopted. Other factors, such as(给出了必要的例证,加分) the emergence of a large amount of new commercial opportunities that automatically attract more investment to build new factories and the aberrant weather which caused an increased respiratory disease rate last year, can also lead to the three phenomena, those of which, however, cannot be simply attributed to the council and may not eliminate even Ann Green is elected.
本段指出给出的环境下降的例子不充分,不足以直接说明环境问题确实存在。

Aside from the above two points, the possibility of an alternative way also could be considered. Even if the ability of Green in solving pollution problems is confirmed and really better than that of Braun, and a significant link between the actions of current council and the pollution problems and ensuing diseases exists, there may be some other candidates who have better performances as Ann Green should not be the necessary and only one.
再次让步,提出了又一个潜在的逻辑错误

In conclusion, the claim is unpersuasive as it stands. To support the editorial, strong evidence, such as more reliable and direct data that reflect the link between the current council’s action and pollution problems, is needed to prove that the current council accounts for them(觉得有些多余). Simultaneously, researches related to the Green’s and Braun’s positions and experience on relevant issues also should be done for showing that Green has the distinct ability in tackling pollution problems(这个用法很地道,赞一个!) and voting him is the best way to solve pollution problems.
总结出了原文如果更有说服力的话需要在哪些方面改进。


很好的文啊,好多词用的都很不错,逻辑方面驳斥的也很严密,但是有些话看着有些熟悉。。。
需要改进的地方嘛,鸡蛋里挑骨头,就说说语言方面吧,有些地方表述还不够流畅,如果再加改进我觉得最起码能拿5分以上
再次赞一个,加油!!
作者: 396857815    时间: 2010-7-12 16:41:51

本帖最后由 396857815 于 2010-7-12 16:43 编辑

这是我的文,未计时。共463words。请改友狂拍

TOPIC: ARGUMENT29 - When Ida McAllister ran for mayor of Lake City four years ago, she failed to win even 30 percent of the vote. But since then, McAllister has made public her commitment to environmental causes. She would be wise, therefore, to announce her intention to close all Lake City parks to automobile traffic, thus following the example of the former mayor of Plainsville, Alecia Yu . On the recommendation of a small group of concerned citizens, Mayor Yu prohibited automobile traffic in all Plainsville parks, and therefore was credited with solving the pollution problem and improving the quality of life in Plainsville. This action would have great appeal to the citizens of Lake City, most of whom participate in the community's recycling program, and would guarantee McAllister's success in her current quest to be Lake City's new mayor.


    The argument states that the former mayor of Plainsville's successful example of acquiring highly praise by prohibiting automobile traffic in all Plainsville parks indicates that Ida McAllister (IM) can also achieve more voters in her current quest if she implements the same measure. The unreasonable comparison leading to this plausible and implicit conlusion makes the argument not thoroughly well-reasoned.
    To begin with, the comparison between Alecia Yu (AY) and IM may mask the discrepancy in the two cities. It may be that citizens in Lake City less likely concern about the environment problem than those in Plainsville. Or almost all the people in Lake City are tend to drive through parks for some inherent factors. For instance, most of the parks in Lake City are in the way of people driving to work. If so, IM’s election this time would obviously not success but possibly fail.
    Next, even though the two cities’ conditions are the same, the author simply draws the conclusion from the result of AY’s action that he has gained the great reputation of better the environment and the quality of life is questionable. There is a buried precondition that AY has been the Plainsville’s mayor at the moment. It is not that the favorable measure that makes him elected to be the mayor. Perhaps, when AY ran for the place of mayor, he didn’t mention any intention about environment.
    In addition, the reason why IM failed four years ago is not given. There may be many other possibilities other than her ignorance of environment problems result in her failure. For example, citizens not elect her for economic policies, education expense, even scandals. If it is not for the problem with the environment policy, she would not be elected no matter what she does on improving the environment.
    Finally, the assertor assumes that the citizens of Lake City mostly participating in recycling program indicate they care about the air pollution caused by automobiles. Accordingly, they would be appealed by the prohibiting cars in parks measure and therefore vote for IM. However, citizens join in the program can prove nothing about their tendency to forbid cars driving in parks. On the other hand, they may quite love to drive through the park for reasons like saving time.
    In conclusion, the argument has told us environmental problems plays an important role in one’s election. But, to make the measure guarantee IM’s success, the more information must be given that the main reason IM failed four years ago is just environmental problems. And the example of mayor AY needs to make clear that it would also work in the election of IM. Besides, more fact should be given to indicate that citizens in Lake City are really concern about the cars running in parks.
作者: 清水风铃_nono    时间: 2010-7-12 17:20:05

本帖最后由 清水风铃_nono 于 2010-7-12 22:24 编辑

369同学你好,很荣幸拜读你的作品~

绿色为个人很喜欢的地方
蓝色是用法建议
红色是个人认为有错误的地方

    The argument states that the former mayor of Plainsville's successful example of acquiring(省略) highly praise by prohibiting automobile traffic in all Plainsville parks indicates that Ida McAllister (IM) can also achieve more voters in her current quest if she implements the same measure(这句话从句的结构是The example…indicates that… 动词有些问题,example是不能indicate的). The unreasonable comparison leading to this plausible and implicit conlusion makes the argument not thoroughly well-reasoned.
    To begin with, the comparison between Alecia Yu (AY) and IM may mask the discrepancy(这个片语不错) in the two cities. It may be that citizens in Lake City are less likely to concern about the environment problem than those in Plainsville. Or almost all the people in Lake City are(去掉) tend to drive through parks for some inherent factors(这里还是用reason比较自然, factor有些怪). For instance, most of the parks in Lake City are in the way of people driving to work. If so, IM’s election this time would obviously not success but possibly fail. (这句话如果写成 If so, the regular life of Lake City residents must be interrupted, which may reversely harm the approval rate of IM 会更好,前一句话摆出了一个例子,说Lake City 的路可能是上班的必经之路,后一句就说IM会输掉选举,中间的推断咱们中国人明白,跟老美说他们会觉得不清楚,加上一句“这样的话人们的正常生活被打乱,一定不会再支持IM”会比较顺些)
    Next, even though(推荐使用even if, even though 都是“即使”,even though有“事实”的感觉,而even if 的假设性更强) the two cities’ conditions are the same, the author simply draws the conclusion from the result of AY’s action that he has gained the great reputation of better the environment and the quality of life is questionable. There is a buried precondition that AY has been the Plainsville’s mayor at the moment. It is not that the favorable measure that makes him elected to be the mayor. Perhaps, when AY ran for the place of mayor, he didn’t mention any intention about environment (这个点很细,能发现很好).
    In addition, the reason why IM failed four years ago is not given. There may be many other possibilities other than her ignorance of environment problems result in her failure. For example, citizens not elect her for economic policies, education expense, even scandals. If it is not for the problem with the environment policy, she would not be elected no matter what she does on improving the environment (这一段很顺畅).
    Finally, the assertor assumes that the citizens of Lake City mostly participating in recycling program indicate they care about the air pollution caused by automobiles (这句话的结构是citizens…indicate…个人不是太习惯indicate的这种用法,好像人们真的向作者“暗示”了,希望确认这个用法的正确性). Accordingly, they would be appealed by the prohibiting cars in parks measure and therefore vote for IM. However, citizens join in the program can prove nothing about their tendency to forbid cars driving in parks(仍然是句式,这句话的结构式citizens…can prove…似乎没有很明确的表达,猜想你的意思是the fact that there are citizens joining the program can prove nothing…). On the other hand, they may quite love to drive through the park for reasons like saving time.
In conclusion, the argument has told us environmental problems plays an important role in one’s election. But, to make the measure guarantee IM’s success, the(去掉) more information must be given that the main reason IM failed four years ago is just environmental problems. And(and是小连接词,尽量不要在句首使用,可以用plus, nevertheless等代替,下文的besides 也很合适) the example of mayor AY needs to make(改为be) clear that it would also work in the election of IM. Besides, more fact should be given to indicate that citizens in Lake City are really concern about the cars running in parks.

总体来说本文还是比较规矩的。逻辑问题发现的也比较细。有一些地方用语看得出是斟酌过的,比较好。文章结构上稍显罗列感,即重点不够突出,感觉是一共找到4个错误,逐一罗列,如果能把较为重要的放在前面,并且让读者体会到这些条目为什么重要,就更好了。笔者今日在本版看到一篇讨论argu论证顺序的帖子,感觉非常受用,也推荐给你。语言上呢,语法错误很少,但句子结构的掌握不够扎实。有一些句子详细分析会觉得主语和谓语的搭配欠考虑,读者需要停下来分析句子结构并加以联想才能体会意思,对影响文章的通顺性有影响,这一点需要注意。
以上评论仅代表个人观点,水平有限,仅供参考。
希望再接再厉!加油!

作者: 清水风铃_nono    时间: 2010-7-12 17:23:40

本帖最后由 清水风铃_nono 于 2010-7-15 11:51 编辑

大家很熟悉的官方范文,我写了一篇自己的,没有计时,企图说明的比较详尽,有很多部分跟官方范文思路不一样或者提到了范文里面没有提到的点。 非常感谢楼下的同学,谢谢。

题目:
Six months ago  the region of Forestville increased the speed limit for vehicles traveling on the region's highways by ten miles per hour.  Since that change took effect, the number of automobile accidents in that region has increased by 15 percent.  But the speed limit in Elmsford, a region neighboring Forestville, remained unchanged, and automobile accidents declined slightly during the same six-month period.  Therefore, if the citizens of Forestville want to reduce the number of automobile accidents on the region's highways, they should campaign to reduce Forestville's speed limit to what it was before the increase.

正文:
Forestville residents, by comparing the automobile accident and speed limit policy of their own with that of the Elmsford. Although clearly presented, the argument, however, fails to take all the alternatives, besides speed limit policy, into consideration. The conclusion of reducing speed limit appears to be reasonable, but could be improved by taking account more specific aspects of Forestville.

The decision of reducing speed limit is completely based on observation of Elmsford, the town nearby, without noticing the possibilities that the data may not compatible with situation in Forestville. Road condition should be considered at the first place. If, for instance, other than smooth city street in Elmsford, the roads in Forestville are mostly build on hills and valleys, full of twists and turns, high speed limit will definitely  lead to high accident rate, while decreasing the limit may not decline the rate optimistically, because despite of driving speed, poor road condition becomes the most severe cause of traffic accidents, which needless to be considered by Elmsford residents. Moreover, when Forestville drivers are more likely to ignore and break traffic rules, including the speed limit, then Elmsford drivers, because current rules are not justified enough, stricter requirement will only annoy them or create dissatisfaction, since more time will be taken to drive to work, not only accident rate may still be high, but potential threaten to security is also undesirable. As with the automobiles, it is possible that more cars are in bad condition in Forestville than in Elmsford, for various reasons such as lack of maintaining service, high maintaining expense, or gasoline of low quality. In this case, the effect of limiting speed may also be limited in Forestville.

In spite of formulating rules according to local conditions, as discussed above, statistic and demographic flaws should also be taken into account when concerning proper solutions to traffic accidents. Accident rate is reported to increase by 15 percent  in Forestville  and slightly decreased in Elmsford. However, the argument does not present the total number of cars in both towns. No evidence shows that the changes  in rate is not affected by huge decrease of total number of Forestville vehicles  and slightly increase of Elmsford cars. Nevertheless, the accident rate provided in the argument are not guaranteed  to be directly caused by speed driving, thus failing to demonstrate that uncontrolled driving speed is, soundly, the most severe cause of traffic accident increase. In this case, reducing speed limit could be useless when the drunken driving or fatigue driving is the main reason of accidents in Forestville. The observation time is also critical. Six month is not enough when the change in accident rate is a function to season and climate change. If Elmsford is a quiet town without a lot of strangers, while, although nearby, Forestville has beautiful landscape and scenery which attracts plenty of tourists driving here to enjoy the best time of the year in the past 6 months, difference of accident rates in the two towns are reasonable, without the impact of speed limit.

Plus, since the argument only provides fact that slight rate decrease is because of unchanged speed limit, even if the situation is completely the same in the both town, which is already nearly impossible, what will happen when speed limit is decreased, in either town, is not provided in the argument. No clear evidence in the statement shows that accident rate is surely to decrease together with the speed limit.

In a conclusion, although it is a good will of Forestville resident to level off accident rate, and the experience of Elmsford is a good reference to some extent, making decisions by only observing a town nearby, for a short time, is not enough, since the situation of two towns can be very different, and statistic errors may be easily ignored. To settle a proper solution, more specific observations of Forestville its own must be conducted.

反馈:
感谢费话先生的修改,很细致很到位,我会在长短句方面多注意一下,还得摒弃一些自己发明的用法以及definately这种绝对词汇,第三段开头决定改成Inspete of..., statistical flaws might lead to wrong conclusions...
再次感谢!

作者: 396857815    时间: 2010-7-13 18:52:26

多谢清水风铃同学的批改,很细致很到位,觉得收获很大!:)另外很想知道你说的那个论证顺序的帖是哪个哈~~~站短我好了,在这里发貌似是违反版规的,斑竹手下留情,多留一段时间再删这层~~~
作者: 费话先生    时间: 2010-7-14 22:40:22

本帖最后由 费话先生 于 2010-7-14 23:15 编辑

题目:
Six months ago  the region of Forestville increased the speed limit for vehicles traveling on the region's highways by ten miles per hour.  Since that change took effect, the number of automobile accidents in that region has increased by 15 percent.  But the speed limit in Elmsford, a region neighboring Forestville, remained unchanged, and automobile accidents declined slightly during the same six-month period.  Therefore, if the citizens of Forestville want to reduce the number of automobile accidents on the region's highways, they should campaign to reduce Forestville's speed limit to what it was before the increase.

正文:
Forestville residents, by comparing the automobile accident andspeed limit policy of their own with that of the Elmsford. Although clearly presented, the argument, however, fails to take all the alternatives, besides speed limit policy, into consideration. (这段逗号还多啦,我个人感觉断的有点不美观,besides那个phrase前后可以考虑不用逗号)The conclusion of reducing speed limit appears to be reasonable, but could be improved by taking account more specific aspects of Forestville.(有转折关系么?应该说appears to be unreasonable, but could be.....,不过我觉得appear 这个词要斟酌,它是说表面看上去怎样,但事实不一定,也就是说后面无论是加unreasonable还是reasonable都有点不妥)

The decision of reducing speed limit is completely based on (the 或 an, 应该要加个冠词吧)observation of Elmsford, the town nearby, without noticing the possibilities that the data may not(be) compatible with situation in Forestville. Road condition should be considered at the first place. If, for instance, other than smooth city street in Elmsford, the roads in Forestville are mostly build on hills and valleys(我觉得有点chiglish,而且road和street不太一样), full of twists and turns, high speed limit will definitely(绝对了点,under high risk of high accident rate) lead to high accident rate, while decreasing the limit may not decline the rate optimistically, because despite of driving speed, poor road condition becomes the most severe cause of traffic accidents, which needless to be considered by Elmsford residents. Moreover, when Forestville drivers are more likely to ignore and break traffic rules, including the speed limit, then Elmsford drivers, because current rules are not justified enough, stricter requirement will only annoy them or create dissatisfaction, since more time will be taken to drive to work, not only accident rate may still be high, but potential threaten to security is also undesirable. As with the automobiles, it is possible that more cars are in bad condition in Forestville than in Elmsford, for various reasons such as lack of maintaining service, high maintaining expense, or gasoline of low quality. In this case, the effect of limiting speed may also be limited in Forestville.

In spite of formulating rules according to local conditions, as discussed above, statistic and demographic flaws (啥叫demographic flaws? 虽然中文是缺陷,但事实上,这个词的意思我觉得吧,是hinder effectiveness,我也说不清了,把这个句子一简化就变成flaws should be considered. 这是要表达的意思么?i'm not sure)should also be taken into account when concerning proper solutions to traffic accidents. Accident rate is reported to increase by 15 percent  in Forestville  and slightly decreased in Elmsford. However, the argument does not present the total number of cars in both towns. No evidence shows that the changes  in rate is not affected by huge decrease of total number of Forestville vehicles  and slightly increase of Elmsford cars. Nevertheless, the accident rate provided in the argument are not guaranteed  to be directly caused by speed driving, thus failing to demonstrate that uncontrolled driving speed is, soundly, the most severe cause of traffic accident increase. In this case, reducing speed limit could be useless when the drunken driving or fatigue driving is the main reason of accidents in Forestville. The observation time is also critical. Six month is not enough when the change in accident rate is a function to season and climate change. If Elmsford is a quiet town without a lot of strangers, while, although nearby, Forestville has beautiful landscape and scenery which attracts plenty of tourists driving here to enjoy the best time of the year in the past 6 months, difference of accident rates in the two towns are reasonable, without the impact of speed limit.

Plus, since the argument only provides fact that slight rate decrease(这语序?要不就the decrease of...,要不就加连词符号好啦 rate-decrease) is because of unchanged speed limit, even if the situation is completely the same in the(删) both town, which is already nearly impossible,(嗯,这个修饰语值得斟酌,有点中式英语的feel,already建议去掉) what will happen when speed limit is decreased, in either town, is not provided in the argument. No clear evidence in the statement shows that accident rate is surely to decrease together with the speed limit.

In a conclusion, although it is a good will of Forestville resident to level off accident rate, and the experience of Elmsford is a good reference to some extent, making decisions by only observing a town nearby, for a short time, is not enough, since the situation of two towns can be very different, and statistic errors may be easily ignored. To settle a proper solution, more specific observations of Forestville its own must be conducted.

嗯,这位版友他因给的那个详实啊,我是非常佩服的。
但是有一些地方可能需要改进,一个就是长句实在是太多了,看得太辛苦呀,特别是TS,太长啦,这样也会冲淡提纲挈领的感觉。我觉得长短句交错会比较流畅一些。另外,有些地方需要简化。我是这样想的,若有不周全就无视我吧。这个说理虽说要充分,但建立在用更少的语言言更多的物上,有些短语吧虽然不重复,但在意思上其实是一样的,给人感觉稍显冗长了些。
anyway,good job!
作者: 费话先生    时间: 2010-7-14 22:42:08

先谢过帮我改楼的大人们~
TOPIC: ARGUMENT45 - The following appeared as an editorial in a wildlife journal.

"Arctic deer live on islands in Canada's arctic region. They search for food by moving over ice from island to island during the course of a year. Their habitat is limited to areas warm enough to sustain the plants on which they feed, and cold enough, at least some of the year, for the ice to cover the sea separating the islands, allowing the deer to travel over it. Unfortunately, according to reports from local hunters, the deer populations are declining. Since these reports coincide with recent global warming trends that have caused the sea ice to melt, we can conclude that the decline in arctic deer populations is the result of deer being unable to follow their age-old migration patterns across the frozen sea."
WORDS: 579
TIME: 00:36:08
DATE: 2010-7-14 11:19:43




Merely based on several substantiated assumptions and reports with vague information, the author concludes than the recent global warming trends caused the sea ice to melt and hence decline the arctic deer population. However, the argument is poor-reasoned and need closer scrutiny.


The threshold problem is that whether the population of arctic deer really declined. Except for a few reports from the local hunters, the author doesn't give any reliable statistics to demonstrate that. It's possible that the arctic deer move to a new habitat where is far away from human's habitat to prevent themselves from killing by the hunters. Or the groups of arctic deer find another favorable habitat with more plants they need, so they just leave their old habitat, as a consequence the hunters less frequently see them than before. Either scenario, if true, will weaken the assumption that the number of the arctic deer did decline. Thus, without more reliable scientific investigate, the author cannot regard the subjective judgment or direct observation of hunters as sufficiently reliable.


Even if the less of arctic deer is unfortunately true, the author cannot justifiably equate the connection between the declining population of arctic deer and the global warming with cause and effect relationship. However, the author gives no evidence the global warming have a marked influence on the ice in Canada' arctic region. Perhaps, the global warming is not so severe enough to melt the ice in Canada (since the temperature in Canada is so low owing to its high latitude). Or the amount of melting ice increases a little and can be neglect with respect to the impact on the arctic deer habitat. Until the author demonstrates the global warming virtually have influenced the habitat of arctic deer and hence decline their population, I cannot accept the assumption at all.


Again, the author fails to take other alternative explanations to the less of arctic deer population into account. It's possible that more arctic deer dead during the past years due to the overhunting of human being rather than the melting ice. Also, the author ignores other possible changes of climate which may also produce adverse effects on the survival of arctic deer except the global warming. Those changes may include the encroachment of arctic deer’s habitats because of the increase of human population. In short, without ruling out other possible alternatives, the author cannot safely say that global warming is attributable to the less of arctic deer.




In the final analyze, suppose that the global warming to some extent accelerate the melting of sea ice, the author claims that the arctic deer wouldn’t change their old-aged migration across the frozen Iceland and accordingly result in their survival crisis. However, the author fails to underpin the assumption with strong evidence (in fact, no evidence). Perhaps, the arctic deer migrates across to another Iceland just to seek for more plants; therefore, if there are enough plants in the Iceland where they currently lived, they will be less likely to migrate. As the author states, the global warming causes the increase of temperature in Canada which may conduce to the growing of plants. As a result, the arctic deer give up their conventional migration pattern for the increasing of food.



In sum, the argument is far away from satisfactorily reliable based on several subjective assumptions. To bolster the argument, the author should provide more statistics to demonstrate the decline of arctic deer population. Also he should prove the declining, if really exists, is the result of global warming other than other possible causes.

作者: koyoi    时间: 2010-7-15 00:28:14

本帖最后由 koyoi 于 2010-7-15 01:26 编辑

占位修改28楼

红色字体语法问题
蓝色字体词句修善问题
紫色字体内容相关问题

Merely based on several substantiated assumptions and reports with (缺定冠词) vague information, the author concludes than the recent global warming trends caused the sea ice to melt and hence decline the arctic deer population. However, the argument is poor-reasoned and need closer scrutiny.

The threshold problem is that whether the population of arctic deer really declined. Except for a few reports from the local hunters, the author
doesn't (正规文体中尽量不要使用缩写) give any reliable statistics to demonstrate that. It's (问题同上) possible that the arctic deer move to a new habitat where is far away from human's habitat to prevent themselves from killing (being killed) by the hunters. Or the groups of arctic deer find another favorable habitat with more plants they need, so they just leave their old habitat,(.) As a consequence the hunters less frequently see them than before. Either scenario, if true, will weaken the (credibility of the / 注意weakenassumption的搭配是不妥的) assumption that the number of the arctic deer did decline. Thus, without more reliable scientific investigate (investigations), the author cannot regard the subjective judgment or direct observation of hunters as (后面应该是名词) sufficiently reliable (evidence).

Even if
the less of (有这样的用法吗?) arctic deer is unfortunately true, the author cannot justifiably equate the connection between the declining population of arctic deer and the global warming with (to the) cause and effect relationship. However, the author gives no evidence the global warming have a marked influence on the ice in Canada' arctic region (前句与后句并无转折关系). Perhaps, the global warming is not so severe enough to melt the ice in Canada (since the temperature in Canada is so low owing to its high latitude). Or the amount of melting ice increases a little (注意a little意为一些,这里用little,即几乎没有,更恰当一些) and can be neglected with respect to the impact on the arctic deer habitat. Until the author demonstrates the global warming virtually have influenced the habitat of arctic deer and hence decline their population, I cannot (这里貌似要用倒装?我不确定 accept the assumption at all.

Again, the author fails to take other alternative explanations to
the less of arctic deer population into account. It's possible that more arctic deer dead (died) during the past years due to the overhunting of human beings rather than the melting ice. Also, the author ignores other possible changes of climate which may also produce (bring about/make etc.Produce表示产生影响貌似显得比较中国化 不地道) adverse effects on the survival of arctic deer except (for 注意exceptexcept for的区别) the global warming. Those changes may include the encroachment of arctic deer’s habitats(北极鹿栖息地的入侵?应该是被入侵)because of the increase of human population (这里可以直接说是人类的入侵,不必提及是因为人口增长才导致的入侵,因为仍然存在一种可能性即人口一定的情况下由于其他因素导致了人类对北极鹿栖息地的侵占). In short, without ruling out other possible alternatives, the author cannot safely say that global warming is attributable to the less of arctic deer.

In the final analyze, suppose that the global warming to some extent accelerate
s the melting of sea ice, the author claims that the arctic deer wouldn’t change their old-aged migration across the frozen Iceland and accordingly result in their survival crisis (逗号前后两句读起来不通,需要rewrite). However, the author fails to underpin the assumption with strong evidencewhat evidence?(in fact, no evidence) (有就是有,没有就是没有,括号注解无意义). Perhaps, the arctic deer migrates across(through) to another Iceland just to seek (for seeking) for more plants; therefore, if there are enough plants in the Iceland where they currently lived, they will be less likely to migrate. (前后不连贯,读起来不通畅) As the author states, the global warming causes the increase of temperature in Canada which may conduce to the growing of plants (作者有暗示这个观点吗?). As a result, the arctic deer give up their conventional migration pattern for because of
the increasing (increase) of food.


In sum, the argument is far away from satisfactorily reliable (宾语在哪?) based on several subjective assumptions. To bolster the argument, the author should provide more statistics to demonstrate the decline of arctic deer population. Also he should prove the declining, if really exists, is the result of global warming other than other possible causes.

总体感觉找的点都蛮准的,而且攻击也到位。在用词和句式保证丰富多样的同时文章显得很客观理性,基本没有很明显的模板痕迹。
但是小的语法错误较多,例如词性和一些词组的区别上需要多加强。另外 transition words的应用不到位,导致文章在有些地方看起来比较吃力。

作者: koyoi    时间: 2010-7-15 00:32:33

本帖最后由 koyoi 于 2010-7-15 01:29 编辑

TOPIC: ARGUMENT35 - The following appeared in thesummary of a study on headaches suffered by the residents of Mentia.



"Salicylates are members of the same chemicalfamily as aspirin, a medicine used to treat headaches. Although many foods arenaturally rich in salicylates, for the past several decades food-processingcompanies have also been adding salicylates to foods as preservatives. Thisrise in the commercial use of salicylates has been found to correlate with asteady decline in the average number of headaches reported by participants inour twenty-year study. Recently, food-processing companies have found thatsalicylates can also be used as flavor additives for foods. With this new usefor salicylates, we can expect a continued steady decline in the number ofheadaches suffered by the average citizen of Mentia."





The researcher claims that the commercial use of salicylates as preservatives in recent decades is the cause of a trend of the decline in the average number of headaches through a 20-year study. In addition, the new function of salicylates as flavour addictives is anticipated to further decline this number in Mentia. It seems a rejoiceful prevision; nevertheless, several loopholes bogged it down.

First of all, the researcher failed to realize that even through salicylates and aspirin belong to the same chemical family, it does not indicate salicylates necessarily have the same function of treating headaches as aspirin. In addition, in the past decades, salicylates have only being used as preservatives, which means people have little opportunity to directly contact with them so that it is of little possibility that the salicylates brought influences on them. As a result, the function of treating headaches remains questionable.

The second flaw relates to the ambiguous statement of the 20-year study. Apart from the uncertainty of the representativeness and size of the patients observed, the possibility of other factors (such as the anomalous weather and changes in people's life styles) that may cause a decline in the average number of headaches is not ruled out so that it could not be acknowledged there is a causality between salicylates and the declining number of headaches.

Finally, the anticipation of the effect brought by the new function of salicylates also remains invalid as it is unknown whether the residents in Mentia have the similar physical conditions as the samples in the study and whether they will consume foods with flavour addictives of salicylates over a long period of time. Even they do, the micro precipitation of the salicylates may affect little on treating headaches.

In sum, the result of the study of the causal link between salicylates and headaches and the anticipation will keep stay in suspicion if the mentioned flaws are not revised. To ameliorate the credibility of them, the researcher should: firstly, corroborate there is a cause-and-effect relationship between salicylates and headaches through scientific experiments; secondly, modify the study, ensuring there is few influence of other factors in spite of salicylates. Furthermore, the content of salicylates as flavour addictives and the conditions of the residents (such as their dietary habits and whether the result of the study can be adopted) are also needed to make a more reliable anticipation.





作者: 江雪    时间: 2010-7-15 00:52:45

eddie_h:请跟据一楼的规则 先删除你的文章 占楼改你楼上的文章 再发你的文章。否则你的楼下直接从你楼上那篇改起 你那篇将被我删除。
作者: figuechen    时间: 2010-7-15 08:54:52

本帖最后由 figuechen 于 2010-7-15 22:09 编辑

修改30楼Argument

The researcher claims that the commercial use of salicylates as preservatives in recent decades is the cause of a trend of the decline in the average number of headaches through a 20-year study. In addition, the new function of salicylates as flavour addictives is anticipated to further decline this number in Mentia. It seems a rejoiceful [我查了韦氏字典也没查到这个词…]prevision; nevertheless, several loopholes bogged it down.
[感觉开头可以再简洁一些,作者重复了两个argument中的claims。个人觉得只用把最后的结论简要的复述一下就好]

First of all, the researcher failed to realize that even through salicylates and aspirin belong to the same chemical family, it does not indicate salicylates necessarily have the same function of treating headaches as aspirin. In addition, in the past decades, salicylates have only being [been]used as preservatives, which means people have little opportunity to directly contact with them [contact with them directly,原来也没什么语法问题,但读着别扭]so that it is of little possibility that the salicylates brought influences on them. As a result, the function of treating headaches remains questionable.
[第一个攻击点攻击的是S可能无法用作治头痛,但在这一段中攻击的对象似乎不太明确。第一句攻击的是原文说SA同属一类化合物;第二句又攻击的却是另一个问题。感觉有点混乱。建议作者将每个攻击的对象写明白,说清楚,勿一句带过。]

The second flaw relates to the ambiguous statement of the 20-year study. Apart from the uncertainty of the representativeness and size of the patients observed, the possibility of [这一整句有语法错误,应该是the possibility that other factors may cause…]other factors (such as the anomalous weather and changes in people's life styles) that may cause a decline in the average number of headaches is not ruled out so that it could not be acknowledged [加个that]there is a causality between salicylates and the declining number of headaches.
[第二个攻击点攻击的是study,攻击的是正解。不过感觉内容很空,最好展开说明一下weatherlife style是如何影响头痛发病率的。]

Finally, the anticipation of the effect brought by the new function of salicylates also remains invalid as it is unknown whether the residents in Mentia have the similar physical conditions as the samples in the study and whether they will consume foods with flavour addictives of salicylates over a long period of time. Even they do, the micro precipitation of the salicylates may affect little on treating headaches.
[第三个攻击点攻击的是结论的充分性,到位了。]

In sum, the result of the study of [这个of想指什么。。。好像没这种用法啊]the causal link between salicylates and headaches and the anticipation will keep stay in suspicion if the mentioned flaws are not revised. To ameliorate the credibility of them, the researcher should: firstly, corroborate there is a cause-and-effect relationship between salicylates and headaches through scientific experiments; secondly, modify the study, ensuring there is few influence of other factors in spite of salicylates. Furthermore, the content of salicylates as flavour addictives and the conditions of the residents (such as their dietary habits and whether the result of the study can be adopted) are also needed to make a more reliable anticipation.

总评:该文优点是语言流畅,攻击点准确,段落内容清晰;缺点是有一些语法错误,并且由于中间段落攻击时有一些例子没有展开仔细讲,所以感觉内容有些单薄,开头也不够新颖。

可能有些吹毛求疵了,还望作者谅解~
作者: figuechen    时间: 2010-7-15 08:58:35

先谢谢楼下了
8 The following appeared in a memorandum issued by the strategic planning department at Omni Inc.                                                                                                                                                  
    "Mesa Foods, a manufacturer of snack foods that currently markets its products within a relatively small region of the country, has strong growth potential. Mesa enjoyed a 20 percent increase in profits last year, and its best-selling product, Diabolique Salsa, has had increased sales over each of the past three years. Since Omni Inc. is interested in reaching 14-to-25 year olds, the age group that consumes the most snack food, we should buy Mesa Foods, and concentrate in particular on marketing Diabolique Salsa throughout the country."

Word:587

    In the memorandum, the author recommends that Omni Inc should purchase Mesa Foods and market Diabolique Salsa in full efforts. To support his claim, the author provides statistics about the profits of Mesa last year compared with the year before and sales trend of Diabolique Salsa the past three years. The claim bases on the assumption that Mesa will continue to earn money and the popularity of Diabolique Salsa will last. There are some obvious unwarranted assumptions in some critical points and therefore the argument is not persuasive.
    First of all, the author fails to provide evidence to illustrate that Diabolique Salsa is welcomed among 14-to-25 year olds consumers, who has been the target of Omni Inc. Perhaps Diabolique Salsa is popular among the old people while the young consumers have shown little interest in it; or perhaps Diabolique Salsa contains a certain kind of hormone so that it is not suitable for children and youngsters. Without ruling out these possibilities, the author cannot confidently draw the assumption that Diabolique Salsa will be accepted by 14-to-25 year old consumers.
    In addition, even if Diabolique Salsa is popular among the 14-to-25 year old consumers, the author provide no evidence to confirm that this profit-increasing trend is going to continue in the future. The author has not accounted for the possibility that snack food safety might arouse public concerns and the sales of snack food will decline in the future; or perhaps potential consumers have already been tired of the smell and taste of Diabolique Salsa and turn to other fast food. Without further information about it, the author cannot convince me that buying Mesa Foods will make money and be a wise choice.
    Moreover, we cannot infer from the fact that Diabolique Salsa is popular in a small region of the country that it will be also welcomed nationwide. The author fails to consider the differences between the region where Mesa Foods locates and the whole country. Common sense tells me that the tastes of people in a region might differ from that of people in another region. For instance perhaps the local residents are fond of hot and spicy fast food while people from other region of the country prefer bland one. Unless the author provides extra evidence to eliminate these possibilities, I cannot accept the claim that marketing Diabolique Salsa throughout the country will bring benefits to Omni Inc.
    Finally, the author does not provide information about the exact profits of Mesa Foods last year and the money we have to pay to buy it, and therefore we cannot evaluate whether the purchase is worthwhile. Perhaps Mesa Foods is just a tiny company which has a high percentage growth but a small total profit; or perhaps Mesa Foods charges twice as the reasonable price for the purchase and Omni Inc does not have the money to buy it. Without considering these possibilities, the author cannot hastily come to the conclusion that Omni Inc should buy Mesa Food.
    In sum, the author’s recommendation that Omni Inc should buy Mesa Foods and that Diabolique Salsa will help make money cannot be bolstered by the limited evidence listed. To convince me of the case, the author has to provide statistic results supporting that Diabolique Salsa is popular among the youngsters throughout the country and Mesa Foods will enjoy an increase in profits the next few years, along with the detailed information about Mesa Foods, including the exact profits and the money Omni Inc has to pay for a purchase.
作者: cant0577    时间: 2010-7-15 18:26:52

本帖最后由 cant0577 于 2010-7-15 22:01 编辑

占位 修改33楼的哦

8 The following appeared in a memorandum issued by the strategic planning department at Omni Inc.                                                                                                                                                  
    "Mesa Foods, a manufacturer of snack foods that currently markets its products within a relatively small region of the country, has strong growth potential. Mesa enjoyed a 20 percent increase in profits last year, and its best-selling product, Diabolique Salsa, has had increased sales over each of the past three years. Since Omni Inc. is interested in reaching 14-to-25 year olds, the age group that consumes the most snack food, we should buy Mesa Foods, and concentrate in particular on marketing Diabolique Salsa throughout the country."

Word:587

    In the memorandum, the author recommends that Omni Inc should purchase Mesa Foods and market Diabolique Salsa in full efforts. To support his claim, the author provides statistics about the profits of Mesa last year compared with the year before and sales trend of Diabolique Salsa the past three years. The claim bases on the assumption that Mesa will continue to earn money and the popularity of Diabolique Salsa will last. 提出让步条件There are some obvious unwarranted assumptions in some critical points and therefore the argument is not persuasive.
    First of all, the author fails to provide evidence to illustrate that Diabolique Salsa is welcomed among 14-to-25 year olds consumers, who has been the target of Omni Inc 这个是一定要论证的,是作者建议的主要原因之一. Perhaps Diabolique Salsa is popular among the old people while the young consumers have shown little interest in it; or perhaps Diabolique Salsa contains a certain kind of hormone so that it is not suitable for children and youngsters. Without ruling out these possibilities, the author cannot confidently draw the assumption that Diabolique Salsa will be accepted by 14-to-25 year old consumers.采用举例例证,我个人觉得你的假设不是很合理,既然都说了is interested in reaching 14-to-25 year olds, the age group that consumes the most snack food, 我们就可以说为什么是interested 的呢? 可以是 该地区只有这么一个牌子的零食,他们没得选择,所以看起来就像是很受欢迎的。
    In addition, even if Diabolique Salsa is popular among the 14-to-25 year old consumers, the author provide no evidence to confirm that this profit-increasing trend is going to continue in the future 第二个要论证的东东. The author has not accounted for the possibility that snack food safety might arouse public concerns and the sales of snack food will decline in the future; or perhaps potential consumers have already been tired of the smell and taste of Diabolique Salsa and turn to other fast food. Without further information about it, the author cannot convince me that buying Mesa Foods will make money and be a wise choice.用于很简洁,例证也很到位,学习
    Moreover, we cannot infer from the fact that Diabolique Salsa is popular in a small region of the country that it will be also welcomed nationwide. The author fails to consider the differences between the region where Mesa Foods locates and the whole country. Common sense tells me that the tastes of people in a region might differ from that of people in another region. For instance perhaps the local residents are fond of hot and spicy fast food while people from other region of the country prefer bland one. Unless the author provides extra evidence to eliminate these possibilities, I cannot accept the claim that marketing Diabolique Salsa throughout the country will bring benefits to Omni Inc.
    Finally, the author does not provide information about the exact profits of Mesa Foods last year and the money we have to pay to buy it, and therefore we cannot evaluate whether the purchase is worthwhile 确实是一个值得考虑的点,想的好仔细呐. Perhaps Mesa Foods is just a tiny company which has a high percentage growth but a small total profit; or perhaps Mesa Foods charges twice as the reasonable price for the purchase and Omni Inc does not have the money to buy it. Without considering these possibilities, the author cannot hastily come to the conclusion that Omni Inc should buy Mesa Food.
    In sum, the author’s recommendation that Omni Inc should buy Mesa Foods and that Diabolique Salsa will help make money cannot be bolstered by the limited evidence listed. To convince me of the case, the author has to provide statistic results supporting that Diabolique Salsa is popular among the youngsters throughout the country and Mesa Foods will enjoy an increase in profits the next few years, along with the detailed information about Mesa Foods, including the exact profits and the money Omni Inc has to pay for a purchase.

总的来说思路很清晰,分析也很到位,几乎每一段最后都会加上意见,表达方式各异,不错不错,学习学习。 在逻辑上,因为这篇 我感觉上基本上和你一样, 基于该产品有升值潜力 和 广大的消费团体 这两个前提 所以主要攻击点是这两点,恩 我觉得你写的很不错 除了第一个感觉假设的不是很合理外。语言能力也很好,值得我学习。。。。我长句一大堆。。。 加油加油啊
作者: cant0577    时间: 2010-7-15 18:27:50

还望楼下大仙不吝赐教~~~

TOPIC: ARGUMENT67 - The following appeared in a letter to the editor of a newspaper serving the villages of Castorville and Polluxton.

"Both the villages of Castorville and Polluxton have experienced sharp declines in the numbers of residents who pay property taxes. To save money and improve service, the two villages recently merged their once separate garbage collection departments into a single department located in Castorville, and the new department has reported few complaints about its service. Last year the library in Polluxton had 20 percent fewer users than during the previous year. It follows that we should now further economize and improve service, as we did with garbage collection, by closing the library in Polluxton and using the library in Castorville to serve both villages."
WORDS: 633          TIME: 。。。。          DATE: 2010/7/14 21:06:53

It is mentioned in the argument that in order to save money and improve service, Castorville and Polluxton merged two garbage collection departments into a single department, and this measure have got nice effect. Based on this, speaker suggests the library in Polluxton should also be closed for the fewer users. Such action seems to take effect, however, the impact made by such two measures to villagers is totally different.

Firstly, in substance, the influence made by merging two garbage collection departments into one department is quiet different from closing the library in Polluxton. Obviously, merging garbage may hardly influence people's daily life, if the process of collecting garbage runs as before. Furthermore, the two villagers may not care where and how to solve garbage from daily life, as long as the new project will not pollute the environment where they live. But, if the library in Polluxton closes just as the speaker's recommend, the life of many people who live in Poluxton must be influenced especially those who go to town library frequently. It is possible that people have to take bus for a long time or walk a long way to the library located in Castorville but it only takes them few minutes to get the town library in Poluxton. Therefore, if we close the library in Poluxton in order to further economize, people are high likely to complaint this measure.

Secondly, closing the library in Polluxtion does not mean the purpose on improving service to be successfully achieved. It is possible that the library in Castorville is so far away and the traffic is so inconvenient that inhabitants in Polluxton are unwilling to go there. In addition, the library will provide services for a larger number of people from two villages than before. Can the capability of book in the library meet the needs from new users? Can the instructions satisfy the increasing number of users? Perhaps a majority of books stocked in the library is about art for the works of most people living in Castorbille is related to art. However, most people in Pulluxton need books concerning with science and technology for they jobs are related to this area. Thus library in Castorville could not provide as the one in Pulluxton. If speaker provides some information about the kinds and the number of books stocked in the library, the location of the library and the traffic condition, and the different needs of villagers, we may make a judgment wheather closing the library in Polluxton will improve service.

Thirdly, there is likely to achieve the opposite result. If the library in Polluxton was closed, the number of people going to the one in Castorbille must be increased. To meet the need of new users, the library is possible to buy more books or convey books from the library in Polluxton. And all these need the government to spend extra money. In addition, what will they do if the library in Castorbille fails to hold so large number of books? Will they enlarge the size of library? If so, the expense is no doubt increased and it is highly possibility that the expense will be higher than before. Hence, we could not just get a conclusion that closing the library in Polluxton will really further economize, for speaker fails to provide the size of Polluxton's library and some other information about the library.

Overall, this measure appears to save money and improve service but lacking so important information about the libraries of two villages, we can hardly make a sense decision. If speaker can make a poll about two villages, evaluate all potential possibilities and provide more information about the library in Castorbille, maybe we can make a right decision and get the aim to further economize and improve service.
作者: 追梦小木耳    时间: 2010-7-15 18:49:15

本帖最后由 追梦小木耳 于 2010-7-15 20:53 编辑

It is mentioned in the argument that in order to save money and improve service, Castorville and Polluxton merged two garbage collection departments into a single department, and this measure have got nice effect. Based on this, speaker suggests the library in Polluxton should also be closed for the fewer users.原文概括的比较简洁,但还是建议不要再复述了 Such action seems to take effect, however, the impact made by such two measures to villagers is totally different.

Firstly, in substance, the influence made by merging two garbage collection departments into one department is quiet different from closing the library in Polluxton. Obviously, merging garbage may hardly influence people's daily life, if the process of collecting garbage runs as before. Furthermore, the two villagers may not care where and how to solve garbage from daily life, as long as the new project will not pollute the environment where they live. But, if the library in Polluxton closes just as the speaker's recommend, the life of many people who live in Poluxton must be influenced especially those who go to town library frequently. It is possible that people have to take bus for a long time or walk a long way to the library located in Castorville but it only takes them few minutes to get the town library in Poluxton. Therefore, if
we close the library in Poluxton is closed in order to further economize, people are high largely likely to
complaint complain this measure.

关闭垃圾站对居民影响小,关闭图书馆造成居民的不方便

Secondly, closing the library in Polluxtion does not mean the purpose on improving service to be successfully achieved. It is possible that the library in Castorville is so far away and the traffic is so inconvenient that inhabitants in Polluxton are unwilling to go there.
和上一段的理由重复了 In addition, the library will has to provide services for a larger number of people from two villages than before. Can the capability of book in the library meet the needs from of new users? Can the instructions satisfy the increasing number of users? Perhaps a majority of books stocked in the library is about art for the works of most people living in Castorbille is related to art. However, most people in Pulluxton need books concerning with science and technology for they their jobs are related to this area. Thus library in Castorville could not provide sufficient specific books as the one in Pulluxton. If speaker provides some information about the kinds and the number of books stocked in the library, the location of the library and, the traffic condition, and the different needs of villagers, we may make a judgment wheather whether closing the library in Polluxton will improve service.
书籍种类不能满足需求
Thirdly, therethe measure is likely to achieve the opposite result. If the library in Polluxton was closed, the number of people going to the one in Castorbille must be increased increasing. To meet the need of new users, the library is possible to buy more books or convey books from the library in Polluxton. And to meet all these need the government has to spend extra money. In addition, what will they do if the library in Castorbille fails to hold so such large number of books? Will they enlarge the size of library? If so, the expense is no doubt increased increasing and it there is highly high possibility that the expense will be higher than before. Hence, we could not just get a conclusion that closing the library in Polluxton will really further economize, for speaker fails to provide the size of Polluxton's library and some other information about the library.
运输书籍、扩充图书馆可能产生更多费用
Overall, this measure appears to save money and improve service but lacking lacks so much important information about the libraries of two villages that we can hardly make a sense sensible decision.
这句话逻辑有问题哦,因为缺少信息我们不能做合理的决定?? If speaker can make a poll about two villages, evaluate all potential possibilities and provide more information about the library in Castorbille, maybe we can make a right decision and get the aim to further economize and improve service.

总体逻辑比较清晰,反驳的理由也比较充分。但是第二部分也涵盖了第一部分的内容,可考虑重新拆分组合。
然后就是语法错误太多,语言表达欠缺了

我的提纲:
由于垃圾站和图书馆所提供的服务是不同的,所以不能用同一种方法处理
(1)    不能说垃圾站合并有好处,合并图书馆就有好处。两地的距离没有考虑、可能会造成不方便
(2)   合并不一定会带来更好的服务。因为书不够,管理员缺少
(3)    合并不一定会更加经济,扩充图书馆,买书,运书都会产生很多费用
作者: 追梦小木耳    时间: 2010-7-15 18:50:31

238The following appeared in a memorandum from the president of Mira Vista College to the college's board of trustees.
"At nearby Green Mountain College, which has more business courses and more job counselors than does Mira Vista College, 90 percent of last year's graduating seniors had job offers from prospective employers. But at Mira Vista College last year, only 70 percent of the seniors who informed the placement office that they would be seeking employment had found full-time jobs within three months after graduation, and only half of these graduates were employed in their major field of study. To help Mira Vista's graduates find employment, we must offer more courses in business and computer technology and hire additional job counselors to help students with their resumés and interviewing skills."

Word 497
The data provided attempt to argue that the employment rate of Mira Vista College is lower than that of Green Mountain College. But the data are not convincing, and the suggestions are groundless as well.

First of all, one must consider how the survey was conducted, especially in Mira Vista College. It seems that not all graduates but only a part of them have participated in the survey. Are they all the hardworking students who represented the educational quality of the college or merely the small group who seldom attended classes and did not spent much time hunting jobs? If the answer is the latter, there would be no wonder that their employments are not satisfactory. Even if all the graduates have informed the placement office their results, the time frame is too short for a graduate of Mira Vista College to find a proper job. Perhaps, those who were not employed in their major are working temporarily in order to support themselves and keep looking for more proper positions. In addition, the argument fails to provide detailed information about the jobs found by graduates of Green Mountain College. Despite the 90percent employment rate, are they all employed in their own field? Are they content with their positions and salaries? Lacking all these specific statistics, the comparison of the graduates’ employment rate of the two colleges is unreasonable.

Second, the fewer amounts of business courses in Mira Vista College does not indicate more business courses could advocate the employment rate. What the role does business course play in hunting jobs? Perhaps, most students in Green Mountain College are majored in business while only a small proportion in MVC are interested and most students’ majors do not concern business at all. Besides, there is no sense to offer more computer technology courses since no evidence demonstrates that it is the poor skill at computer technology that leads to the unsatisfactory employment. On the other hand, increasing the number of courses in business and computer technology would lay burdens to the students, who therefore, have to spend more time on these classes and own insufficient time for major study. Consequently, without diligent study in major courses, it would be even harder for them to find proper positions.

Finally, as the argument’s failure to figure out the essential reason behind the low employment rate of MVC, hiring additional job counselors may hardly take effect. Perhaps, with the help of job counselors, graduates succeed in founding perfect jobs by showing extraordinary resumes and splendid interviewing skills. But after working a period of time, they turn out to be unqualified for the position and get demoted. In this case, possessing more job counselors does not have pros.

To conclude, the arguer fails to take into consideration other possibilities before comparing the statistics which are questionable. Even the assumption that the graduates’ employment of MVC is not as agreeable as that of GMC, vital reasons should be found before any decisions are made.


请楼下同学指点!
作者: tyarel    时间: 2010-7-15 21:56:08

本帖最后由 tyarel 于 2010-7-17 14:22 编辑

占楼改37L




238The following appeared in a memorandum from the president of Mira Vista College to the college's board of trustees.
"At nearby Green Mountain College, which has more business courses and more job counselors than does Mira Vista College, 90 percent of last year's graduating seniors had job offers from prospective employers. But at Mira Vista College last year, only 70 percent of the seniors who informed the placement office that they would be seeking employment had found full-time jobs within three months after graduation, and only half of these graduates were employed in their major field of study. To help Mira Vista's graduates find employment, we must offer more courses in business and computer technology and hire additional job counselors to help students with their resumés and interviewing skills."

Word 497


The data provided attempt to argue that the employment rate of Mira Vista College is lower than that of Green Mountain College. But the data are not convincing, and the suggestions are groundless as well.


First of all, one must consider how the survey was conducted, especially in Mira Vista College. It seems that not all graduates but only a part of them have participated in the survey. Are they all the hardworking students who represented the educational quality of the college or merely the small group who seldom attended classes and did not spent much time hunting jobs? If the answer is the latter, there would be no wonder that their employments are not satisfactory. Even if all the graduates have informed the placement office their results, the time frame is too short for a graduate of Mira Vista College to find a proper job. Perhaps, those who were not employed in their major are working temporarily in order to support themselves and keep looking for more proper positions. In addition, the argument fails to provide detailed information about the jobs found by graduates of Green Mountain College. Despite the 90 percent employment rate, are they all employed in their own field? Are they content with their positions and salaries? Lacking all these specific statistics, the comparison of the graduates’ employment rate of the two colleges is unreasonable.

Second, the fewer amounts of business courses in Mira Vista College do not indicate more business courses could advocate the employment rate. What the role does business course play in hunting jobs? Perhaps, most students in Green Mountain College are majored in business while only a small proportion in MVC are interested and most students’ majors do not concern business at all. Besides, there is no sense to offer more computer technology courses since no evidence demonstrates that it is the poor skill at computer technology that leads to the unsatisfactory employment. On the other hand, increasing the number of courses in business and computer technology would lay burdens to the students, who therefore, have to spend more time on these classes and own insufficient time for major study. Consequently, without diligent study in major courses, it would be even harder for them to find proper positions.

Finally, as the argument’s failure to figure out the essential reason behind the low employment rate of MVC, hiring additional job counselors may hardly take effect. Perhaps, with the help of job counselors, graduates succeed in founding perfect jobs by showing extraordinary resumes and splendid interviewing skills. But after working a period of time, they turn out to be unqualified for the position and get demoted. In this case, possessing more job counselors does not have pros.

To conclude, the arguer fails to take into consideration other possibilities before comparing the statistics which are questionable. Even the assumption that the graduates’ employment of MVC is not as agreeable as that of GMC, vital reasons should be found before any decisions are made.


语言读下来感觉也很顺畅,很舒服
对于开头结尾提一点小小的建议。首段很简洁,不过感觉简单得过头了,感觉是个什么文章都可以套的开头段,其实可以在第一段中做点小小的铺垫,引一下后面要反驳的点,这样,读者在读了开头段的之后就能马上就能了解你会从哪些方面去反驳。如果在后面每段能轻易找到对应的反驳,那就显得你的逻辑很清晰。
https://bbs.gter.net/viewthread.php?tid=920961&highlight=  
这个帖子我觉得对于改进Argument的开头思路蛮有好处的
至于结尾,可以多提些建议,毕竟全文都是在反驳,这里继续反驳其实也反驳不出什么了,倒是提下建议,显得文章丰满。这个也是人家给我Argument结尾的建议,你自己可以斟酌一下~~


对于全文的逻辑和反驳点,我提些自己的看法。我个人觉得追梦的全文结构安排稍有问题。我觉得这个文章的逻辑线条是GMCbusiness课程较多+GMC的就业率较好→较多的business和counselors导致了高就业率→MVC按照建议来能提高就业率。那么可以抓的点就是,1.GMC就业率是否真的好? 2.business和counselor是否导致就业率好? 3.按建议来是否就有效 来做反驳。因为我感觉原文里表述有点杂,而且他的逻辑线是单线的,没有其它事实的参杂,所以按逻辑顺序可能条理更清楚。在第2段的反驳里,我觉得涉及到了2个反驳点。一个就是我前面讲到的反驳点2,另外就是前面讲到的反驳点3,然后第2的大部分和第3反驳又都是围绕最后的建议来反驳的。这样的话,我倒觉得可以把第2段前半部分拆出来,单独成段。而后半部分和第3段可以放在一起的,本质上差不多,就是说这个建议未必能是就业率提高上去。这样子显得比较清晰,否则第2段看着很乱。全文的反驳也缺一条清晰的线路。


作者: tyarel    时间: 2010-7-15 21:57:27

TOPIC: ARGUMENT61 - The following appeared in a report by the School District of Eyleria.

"Nationally, the average ratio of computers to students in kindergarten through grade 12 (K-12) is 1:5. Educators indicate that this is very good ratio. This means that across the country, all students have access to and can use computers daily in their classrooms. In Eyleria's K-12 schools, the ratio of computers to students is 1:7. This number is sufficient to ensure that all of Eyleria's students, by the time they graduate from high school, will be fully proficient in the use of computer technology. Thus, there is no reason to spend any of the schools' budget on computers or other technology in the next few years."
WORDS: 460      

Based on the survey of the computers ratio to students in nation and in Eyleria, the author inferred that the number of computers is adequate to students here and suggested that schools' budget should not be spent on computers or other technology in the next few years. Unfortunately, that is not the case and his reasons are unwarranted at the same time.

Foremost, the author drew the conclusion that there are enough computers in Eyleria relying on the ratio of computers compared with that of whole country.  However, the inference is not so potent since the survey was  about the number of computers in kindergarten through 12 and no accurate number or ratio were offered to show the how many computers exactly in each grades. Maybe, in Eyleria, the owners of the majority computers are the kindergarten and there is few computers in higher grade. Thus, it is uncertain whether the number of computers is high enough to students.

Even if the assumption that there are sufficient in Eyleria to all the students is correct, that doesn't mean every student will have access to computers. The average ratio is about the whole Eyleria. Hence it is highly possible that the ratio in some schools will be much lower than that and students in those schools may have no chance to use computers, let alone adept the use of computer technology. If that is the case, schools’ budget spent on computers is essential and necessary obviously.

What's more, even though it has been to an ample degree right now, the number of computers cannot be guaranteed enough in the future. Perhaps there will be some trouble in these computers and they will be discarded. Or, the increasing number of students in Eyleria will also low the ratio. Without excluding these possibilities, it is unreasonable to assert that pursuing more computers is meaningless.

Last but not least, ruling out all the rebuttals I provided above, the conclusion is also not that cogent.  The author merely gave the reason why computers-pursuit is unnecessary. Yet, he didn't supply any reason why other technology should not be bought. If there is a grave shortage of facilities of these technologies, why shouldn't schools' budget be spent on them? So, lacking information about the situation of other technology, like the equipment needed in experiment or other facilities, it is reckless for the author to get the suggestion like that.

All in all, besides not supplying enough accurate ratios in each school and grade, the author didn't do any analysis about the likely situation in the future. And that render his conclusion rather weak. If he can show that the numbers of computers are and will really enough to all the students, the argument may be more persuasive.

说实话,对于这篇文章自己找的逻辑错误不是很有信心,请LX狠拍,谢谢
作者: beanie加油    时间: 2010-7-15 22:30:19

本帖最后由 beanie加油 于 2010-7-16 13:12 编辑

TOPIC: ARGUMENT61 - The following appeared in a report by the School District of Eyleria.

"Nationally, the average ratio of computers to students in kindergarten through grade 12 (K-12) is 1:5. Educators indicate that this is very good ratio. This means that across the country, all students have access to and can use computers daily in their classrooms. In Eyleria's K-12 schools, the ratio of computers to students is 1:7. This number is sufficient to ensure that all of Eyleria's students, by the time they graduate from high school, will be fully proficient in the use of computer technology. Thus, there is no reason to spend any of the schools' budget on computers or other technology in the next few years."
WORDS: 460      

Based on the survey of the computers ratio to students in nation and in Eyleria, the author inferred that the number of computers is adequate to students here and suggested that schools' budget should not be spent on computers or other technology in the next few years. Unfortunately, that is not the case and his reasons are unwarranted at the same time.

Foremost, the author drew the conclusion that there are enough computers in Eyleria relying on the ratio of computers compared with that of whole country.  However, the inference is not so potent since the survey was  about the number of computers in kindergarten through 12 and no accurate number or ratio were offered to show the how many computers exactly in each grades. Maybe, in Eyleria, the owners of the majority computers are the kindergarten and there is few computers in higher grade. Thus, it is uncertain whether the number of computers is high enough to students.

Even if the assumption that there are sufficient in Eyleria to all the students is correct, that doesn't mean every student will have access to computers. The average ratio is about the whole Eyleria. Hence it is highly possible that the ratio in some schools will be much lower than that and students in those schools may have no chance to use computers, let alone adept the use of computer technology. If that is the case, schools’ budget spent on computers is essential and necessary obviously.

What's more, even though it has been to an ample degree right now, the number of computers cannot be guaranteed enough in the future. Perhaps there will be some trouble in these computers and they will be discarded. Or, the increasing number of students in Eyleria will also low the ratio. Without excluding these possibilities, it is unreasonable to assert that pursuing more computers is meaningless.

Last but not least, ruling out all the rebuttals I provided above, the conclusion is also not that cogent.  The author merely gave the reason why computers-pursuit is unnecessary. Yet, he didn't supply any reason why other technology should not be bought. If there is a grave shortage of facilities of these technologies, why shouldn't schools' budget be spent on them? So, lacking information about the situation of other technology, like the equipment needed in experiment or other facilities, it is reckless for the author to get the suggestion like that.

All in all, besides not supplying enough accurate ratios in each school and grade, the author didn't do any analysis about the likely situation in the future. And that render his conclusion rather weak. If he can show that the numbers of computers are and will really enough to all the students, the argument may be more persuasive.

占位改39
首先感觉你段落比较清楚,每段的字数相当,思路也明白,还是不错的,肯定下,下面简单分析下你每段的东东
开头段:言简意赅,不错,但缺点是模板化较明显,作者可以稍加修改,避免套路的大众化。
正文段:攻击的是第2个例子。你的观点是不一定每个年级都有相同的电脑,therefore就不能推出the number of computers is high enough to students.。在此段你提出了一个possibility,那就是可能低年级电脑多高年级可能相对少。这个例子是可以的。但是你忽略了一个很重要的东西,我觉得。。文中提到“by the time they graduate from high school, will be fully proficient in the use of computer technology”,就是说这个比例能都确定学生们在毕业时能高有一定的电脑水平?evidently, it is not the case. 你可以举例说因为电脑学生比率是17 所以不一定每个学生能够经常用到电脑,因为他们平时还有很繁重的课业负担。另外一点我自己想的不知道这点何不合理,就是很有可能很多学生到高中毕业之前才掌握电脑技能,这也反映了一是电脑的不够用,二是可能没有专业的人去教,所以在结论中说不去投资是不合理的。额。。。不知道这点对不对。。
第二段继续反驳结论,说的是全国范围不等于地方范围,很好。
第三段举了两个possibility,一是电脑在未来的数量,二使学生的数量。Good
紧接着第4段提到其他的设备,我觉的折磨说可能更好:
Last but not least, the conclusion is quite definitive which unfaily rules out other useful technologies that will benefit the students’ proficiency in using computer.It is more likely that the software programs including the photoshop will greatly improve the students ability of acquring the computer skills . without considering the advantages of introducing these kinds of technologies, the author can not draw the conclusion that the computers-pursuit is unnecessary.
因为你之前写的那个批评较多我感觉,所以把一些批评得东西删掉加了一些possibilty
最后一段,还是批评较多,argument主要不是批评,是指出里面的错误并帮他改正。所以最后一段你可以稍加改改,多一点建议,少一点批评。
总的来说,逻辑比较清楚,思维很明显,每段的安拍较合理,但语言缺乏variety。建议写的时候思考着如何让语言再漂亮些,嘿嘿,这样的你文章就比现在无敌多了。
个人意见,仅供参考。

作者: beanie加油    时间: 2010-7-16 11:07:36

本帖最后由 beanie加油 于 2010-7-16 11:37 编辑

149. The following is a memorandum from the director of personnel to the president of Get-Away Airlines.
"Since our mechanics are responsible for inspecting and maintaining our aircraft, Get-Away Airlines should pay to send them to the Quality-Care Seminar, a two-week seminar on proper maintenance procedures. I recommend this seminar because it is likely to be a wise investment, given that the automobile racing industry recently reported that the performance of its maintenance crews improved markedly after their crews had attended the seminar(1). These maintenance crews perform many of the same functions as do our mechanics, including refueling and repairing engines(2). The money we spend on sending our staff to the seminar will inevitably lead to improved maintenance(3) and thus to greater customer satisfaction(4) along with greater profits for our airline(5)."
     The speaker asserts that Get-Away Airlines should send the mechanics to a seminar on proper maintenance procedures due to the markable performance of the crews in automobile racing industry did after their attendance of it. Believing it is a wise investment, the director think that it will inevitably lead to improved maintenance and thus a greater customer satisfaction along with greater profits. It seems to be compelling at the first glance, however, it we consider it deeply, we may not hard to find out several critical fallacies in the argument, greatly undermine the persuasive stand the argument tends to demonstrate.
     The director embraces that by sending our staff to the seminar will improve our maintenance level. The reason for him to strongly hold the position is contributed to the improved performance of the crews in automobile racing industry after they took the seminar. But the director fails to differentiate between the two companies before he wrongly expect the Get-Away Airlines to achieve the same benefit. Common sense tells us that there exist great differences between the same jobs in the two companies. For instance, the inner structures of many apparatus in the automobile are quite likely to be different with those in the airplane. Given that reason, it is unwise to send our staff to take the same seminar for it is actually leading them to a wrong way and even worse may increase the incidence of accidents of both the staff and the airplane. So in order to promote a better level of maintenance, the director can encourage their staff to go to a seminar which is specifically designed for those working in an airway company rather than a general lecture.
     When comes to the potential raising customer satisfaction, the director states his opinion that by improving the level of maintenance, the company can easily attain the goal, which is in fact not the case. Taking into account the factors that influence the customer satisfaction, we may reach an agreement that it constitutes several parts instead of only relying on one high level of maintenance. Others serving to it are including the customer service, ticket price, airplane safety, and its punctuality. Each element plays an indispensable role and without working it well, the greater customer satisfaction can not be easily achieved. Hence, the director should lay his attention not simply on the department of inspecting and maintaining, but on the whole company, which will help him realize his aim.
     Mentioned about the greater profits for the airplane company, the director assumes that the company will be quite beneficial by them. However, this is not the case. Lacking of considering the cost the company will pay on the seminar and the revenue they will receive in their future work, the director can not conclude that they will get profits rather than lose their money. So it is quite possible that the airplane company will pay much money on the seminar and earn a little on account of the wrong seminar their staff attend. If the company wants to get the greater profits mentioned, they need to require their staff to take the professional seminar first, raise their customer satisfaction, and then probably run an advertisement campaign on a nationwide scale or even a worldwide scale.
     To simply put, the director makes a wrong comparison between the two companies and then gets a conclusion that the airplane company needs to adopt the method that is proved accessible by the automobile industry. However, if he wishes to improve his company’s level of maintenance, the customer satisfaction and the greater profits, what he should do is to start thinking from his own company to find out the existing flaws in their work and then to correct them in proper ways.
作者: 秋尽江南    时间: 2010-7-16 15:50:39

本帖最后由 秋尽江南 于 2010-7-16 16:28 编辑

149. The following is a memorandum from the director of personnel to the president of Get-Away Airlines.
"Since our mechanics are responsible for inspecting and maintaining ouraircraft, Get-Away Airlines should pay to send them to the Quality-CareSeminar, a two-week seminar on proper maintenance procedures. Irecommend this seminar because it is likely to be a wise investment,given that the automobile racing industry recently reported that theperformance of its maintenance crews improved markedly after theircrews had attended the seminar(1). These maintenance crews perform manyof the same functions as do our mechanics, including refueling andrepairing engines(2). The money we spend on sending our staff to theseminar will inevitably lead to improved maintenance(3) and thus togreater customer satisfaction(4) along with greater profits for ourairline(5)."

     The speaker asserts that Get-Away Airlines should send themechanics to a seminar on proper maintenance procedures due to themarkable performance of the crews in automobile racing industry didafter their attendance of it(attending it,用动名词形式是不是更简洁些?). Believing it is a wise investment, thedirector thinks that it will inevitably lead to improved maintenance andthus a greater customer satisfaction along with greater profits. Itseems to be compelling at the first glance(at first glance), however, it(if) we consider itdeeply, we may not hard to(一般用法应该是形式主语吧,it is not hard to...但为了和前面的从句保持主谓一致,也许可以改成without difficulty来表达同样的意思,或者干脆删去,keep concise) find out several critical fallacies in theargument, greatly undermine the persuasive stand the argument tends todemonstrate.
     The director embraces that by(把by删去) sending our staff to the seminarwill improve our maintenance level. The reason for him to strongly holdthe position is contributed to the improved performance of the crews inautomobile racing industry after they took the seminar. But thedirector fails to differentiate between the two companies before hewrongly expect the Get-Away Airlines to achieve the same benefit.Common sense tells us that there exist great differences between thesame(similar?) jobs in the two companies. For instance, the inner structures ofmany apparatus in the automobile are quite likely to be different with(from)those in the airplane. Given that reason, it is unwise to send ourstaff to take the same seminar for it is actually leading them to awrong way and even worse may increase the incidence of accidents ofboth the staff and the airplane(for it may mislead them, and in even worse condition, increase the incidence of accidents). So in order to promote a better(promote和better是不是重复了) levelof maintenance, the director can encourage their staff to go to aseminar which is specifically designed for those working in an airwaycompany rather than a general lecture.
     When comes to the potential raising customer satisfaction, thedirector states his opinion that by improving the level of maintenance,the company can easily attain the goal, which is in fact not the case.Taking into account the factors that influence the customersatisfaction, we may reach an agreement that it(感觉it的指代不太明确) constitutes severalparts instead of only relying on one high level of maintenance. Othersserving to it are including the customer service, ticket price,airplane safety, and its punctuality. (可以挑一个具体展开说明一下)Each element plays anindispensable role and without working it well, the greater customersatisfaction can not be easily achieved(主谓不一致). Hence, the director should layhis attention not simply on the department of inspecting andmaintaining, but on the whole company, which will help him realize hisaim.
     Mentioned about the greater profits for the airplane company, thedirector assumes that the company will be quite beneficial by them.However, this is not the case. (与上一段的开头句重复了,which is in fact not the case)Lacking of considering the cost thecompany will pay on the seminar and the revenue they will receive intheir future work, the director can not conclude that they will getprofits rather than lose their money. So it is quite possible that theairplane company will pay much money on the seminar and earn a littleon account of the wrong seminar their staff attend.(感觉这句话和上面一句有点重复:The expense of the seminar may greatly exceed what the company can achieve. 这样改不知道怎么样?) If the companywants to get the greater profits mentioned, they need to require theirstaff to take the professional seminar first, raise their customersatisfaction, and then probably run an advertisement campaign on anationwide scale or even a worldwide scale.
     To simply put, the director makes a wrong comparison between thetwo companies and then gets a conclusion that the airplane companyneeds to adopt the method that is proved accessible by the automobileindustry. However, if he wishes to improve his company’s level ofmaintenance, the customer satisfaction and the greater profits, what heshould do is to start thinking from his own company to find out theexisting flaws in their work and then to correct them in proper ways(最后给出的解决方案会不会太泛泛了一些?).

other possibilities列举的很充分,值得学习~举完反例之后可以再具体详细阐述一下,更具有说服力一些~
语法方面有一些长句在表意上不太明确,也不易理解(不排除我理解能力上的问题……)
作者: 秋尽江南    时间: 2010-7-16 15:56:00

TOPIC: ARGUMENT186 - The following is a recommendation from the director of personnel to the president of Professional Printing Company.

"In a recent telephone survey of automobile factory workers, older employees were less likely to report that having a supervisor present increases their productivity. Among workers aged 18 to 29, 27 percent said that they are more productive in the presence of their immediate supervisor, compared to 12 percent for those aged 30 or over, and only 8 percent for those aged 50 or over. Clearly, if our printing company hires mainly older employees, we will increase productivity and save money because of the reduced need for supervisors."

提纲:
汽车厂工人的例子不能用于印刷厂
即使印刷厂的情况和汽车厂类似,文中的调查结果只给出了老工人、年轻工人分别在有无监督者的情况下的工作效率,不能说明老工人的生产效率比年轻人高。
即使老工人的生产效率高,也不一定能达到省钱的目的。也许还有其他省钱的方法。


In this argument, the author recommends that Professional Print Company can increase productivity and save money by hiring mainly older employees. The recommendation seems reasonable at first glance, however, the author makes false analog by comparing automobile factory workers to printing company ones and the recommendation depends on several unwarranted assumptions. The implementation of such baseless recommendation may not achieve the goal as the director has expected.

The survey may describe the status of automobile factory workers, yet no evidence is provided that the same situation occurs in the printing company. Since the two industries differ in several aspects, from working environment to the machines they used, it is entirely possible that the trend just reverses at PPC: the productivity of older workers increases in the presence a supervisor while that of the young ones falls. Without more statistics to substantiate the analog between automobile factory workers and printing company ones, the author cannot convince me of the recommendation.

Moreover, the author hastily concludes that older workers work more efficiently than young ones, which is not supported by the statistics. The survey separately discusses the productivity of old and young workers in the presence and absence of a supervisor, but not comparatively. Common sense tells us that generally, young workers are more productive because they are agile, vigorous and in good physical condition. The author may overlook the possibility that older workers are still less productive than young ones without a supervisor. The author cannot judge the comparative productivity according to limited statistics.

Assuming that hiring older workers, and less supervisor at the same time, can increase productivity of the company, it does not suffice to save money. Perhaps experienced, skillful old workers charge more salary than younger ones, if so, these higher wages might offset production gains and payroll savings accruing from reduced supervision. For that matter,  the author’s assumption that hiring older workers will also save money does not stand as it states. Further, there exist other approaches to save money such as laying off unnecessary or inefficient employees, economizing on administrative costs and relocation to a place where charges low rent. With an overall analysis on other methods on saving, the author can make a balanced recommendation.

In sum, the author cannot justify his recommendation of hiring mainly older workers in the argument. To better support the argument, the author has to conduct similar survey in printing company or obtain relative statistics, and have a complete view on other possible solutions.
作者: 清水风铃_nono    时间: 2010-7-16 17:18:49

本帖最后由 清水风铃_nono 于 2010-7-17 11:50 编辑

In this argument, the author recommends that Professional Print Company can increase productivity and save money by hiring mainly older employees. The recommendation seems reasonable at first glance(像是套用模板,但其实,“at the first glance”并不觉得主要雇用老员工“看似合理”,相反,绝大多数都雇用老人很奇怪的,如果想这么用,起码要说,“从全文的推理来看”。模板的使用要当心,套用以前最好再多捉摸一下逻辑~), however, the author makes false analog by comparing automobile factory workers to printing company ones and the recommendation depends on several unwarranted assumptions(这个短语好,学习学习). The implementation of such baseless recommendation may not achieve the goal as the director has expected. (开头清晰间接,点出了文章的两个主要错误,盲目比较和无关推断)

The survey
may(
这个地方用may欠妥当,survey“确实”而不是“可能”描述了汽车厂的情况,可以直接用the survey describes…) describe the status(这个词的确有“情况”的意思,可根据金山词霸的例句和google上搜到的句子用法,status这个词更多的是表示“地位”,它的“情况”的意思也跟地位多少有关系,我对这个词也不太熟,希望使用的时候最好小心一些) of automobile factory workers, yet no evidence is provided that the same situation occurs in the printing company. Since the two industries differ in several aspects, from working environment to the machines they used, it is entirely possible that the trend just (省略) reverses at PPC(看了半天才明白这个缩写是指那个公司,如果一定要用缩写,最好在第一次提到全称的时候写一个括号:PPC 或者用其他合适的方法标一下): the productivity of older workers increases in the presence a supervisor while that of the young ones falls. Without more statistics to substantiate the analog between automobile factory workers and printing company ones, the author cannot convince me of the recommendation. (这一段的思路很清晰,错误类比的确是本文的一个大问题。只是,为什么印刷厂和汽车工厂是不同的,个人觉得要是能在具体些就好了。现在说的是印刷厂的情况完全可能跟汽车厂相反,这样没错,5分肯定是有了,只是要想六分,咱们就得琢磨这两个厂在什么地方的区别足以导致类比的失败……我还没想好,你如果想到了站短我~)
Moreover, the author hastily(仓促的,好词!) concludes that older workers work more efficiently than young ones, which is not supported by the statistics. The survey separately discusses the productivity of old and young workers in the presence and absence of a supervisor, but not comparatively. Common sense tells us that generally, young workers are more productive because they are agile, vigorous and in good physical condition(这三个词用得很到位,学习!). The author may overlook the possibility that older workers are still less productive than young ones without a supervisor. The author cannot judge the comparative productivity according to limited statistics. (这一段从Common开始后面都写的很流畅,只是画蓝色的那一句有些没点透,而且这还是比较关键的一句。文中是有比较的,27%,12% ,8%就是比较,生产效率和上司的存在也是有联系的,相应比例的人认为they are more productive in the presence of their immediate supervisor。这处的真正问题在于,调查里说的是,认为有上司在,他们的生产率就提高的人数,随年龄递减;但不等同于,生产率随年龄递增,也不等同于,需要上司的人数随年龄递减,作者做了这样两个推理是不对的。至于为什么不等同,本段从common开始后面都解释得比较清楚。并且,下面一段的逻辑也是接着这个的。个人认为这个逻辑很重要,希望我说明白了,有什么想法我们一块探讨~)

Assuming that hiring older workers, and less supervisor at the same time, can increase productivity of the company, it does not suffice to save money. Perhaps experienced, skillful old workers charge more salary than younger ones, if so, these higher wages might offset production gains and payroll savings accruing from reduced supervision.(
情况1,老年熟练工人工资更高) For that matter,  the author’s assumption that hiring older workers will also save money does not stand as it states. Further(好词!), there exist other approaches to save money such as laying off unnecessary or inefficient employees, economizing on administrative costs and relocation to a place where charges low rent. With an overall analysis on other methods on saving, the author can make a balanced recommendation (后面半段说的是其他省钱的方法,个人觉得这个讨论是正确的,只是稍有点远。既然这一段的攻击目标是最后一句,不妨换一种思路:说完老年熟练工人工资更高这种情况,将老人经验多作为一个让步。经验多,但问题更多。跟年轻人相反,老人Less energetic, poor physical condition,如果老生病,雇2个只能用一个,产量更低。这是攻击“increase productivity”。老人需要和生活保障,保险之类,雇老人又干不了多少年就得退休,还得费劲雇新的,这都得花钱,攻击“save money”这样一来,驳斥相对有力一些,你觉得呢?).

In sum, the author cannot justify his recommendation of hiring mainly older workers in the argument. To better support the argument, the author has to conduct similar survey in printing company or obtain relative statistics, and have a complete view on other possible solutions.


文章的思路架构是很清晰的。特别突出的是连接词和某些小词的运用很准确,看得出来在炼字上下了很大的功夫。各种学习你的用法ing
总体来说没有什么大的问题,下一步需要提高的就是攻击的有力性和逻辑把握的准确性了。详见每段后面的紫色描述,要是这一点上再有提高,个人觉得就没什么问题了~
欢迎交流!加油加油!
作者: 清水风铃_nono    时间: 2010-7-16 17:25:14

本帖最后由 清水风铃_nono 于 2010-7-17 11:58 编辑

TOPIC: ARGUEMENT 8 -The following appeared in a memorandum issued by the strategic planning department at Omni Inc.
"Mesa Foods, a manufacturer of snack foods that currently markets its products within a relatively small region of the country, has strong growth potential. Mesa enjoyed a 20 percent increase in profits last year, and its best-selling product, Diabolique Salsa, has had increased sales over each of the past three years. Since Omni Inc. is interested in reaching 14-to-25 year olds, the age group that consumes the most snack food, we should buy Mesa Foods, and concentrate in particular on marketing Diabolique Salsa throughout the country."


提纲:
收购MF的决定有问题,MF不一定好,Omni 不一定有能力,两家公司不一定适合
即使收购MF,是否推广DS食品也值得商榷,DS是否在当地受欢迎都不一定,何况要对广全国。


According to the argument, optimistic performance  of Mesa Foods, the snack food manufacturer, rationalize the decision of Omni Inc. to buy Mesa Foods, and to concentrate on marketing Diabolique Salsa, the best-seller of Mesa Foods. However, either the purchase or the marking strategy is lack of consideration. If Omni execute this strategy, crisis is likely to be around the corner.

The merging plan should be on the top of reconsideration. Mesa Food is stressed to have “strong growth potential” in the argument, but not strongly supported by the data. Either 20 the increase in profit last year or the increase in sales of Diabolique Salsa only suggest that Mesa Food is experiencing increase in the past. The arguer fails to provide enough hints, such as strong R&D ability, innovation, reliable quality control and mature marking strategy, to guarantee Mesa Food’s promising future. For instance, if the snacks are reported to contain poisoned ingredients after Omni taken charge of Mesa Food, Omni may be in endless trouble of consumer’s complaining, suffering credibility crisis; or, it may forced to afford huge compensation, leading to financial crisis.  

Even if Mesa Food is merit enough and have great potential, it is not guaranteed that buying it the a wise choice of Omni Inc. Instead, the Omni should evaluate the financial ability of the both company. If Omni is much smaller Mesa Food, and not enough credits or loans are available, it is nearly impossible of Omni to perform the purchase. Moreover, the argument only mentions that the target consumers of Omni are 14-to-25 year olds, but does not present the fact that Omni is in the food industry. Instead, if Omni is a well performed youth clothing company, is it of high risk to buy a corporation of food industry, which is an unfamiliar field to Omni.

If finally the entire  crisis and risks are predicted, and Omni is appropriate as well as having the ability to buy Mesa Food, the chef of Omni may not have a breathing spell. The new problem is, is marking Diabolique Salsa throughout the country feasible ? It is highly doubted. Although DS is the bestseller of Mesa Food, neither the market share nor the growth rate of it is provided through out the argument. The acceptance degree of DS is unknown thus the sales situation in the future is hardly predicted. Provided that DS is well accepted, it is not guaranteed that DS is wide accepted, because the former market of DS is within a relatively small region of the country. If the flavor of DS is weird to other parts of the country, the promotion of DS will be more difficult, and might be a failure.

To summarize, purchasing a company and promote a new marketing strategy deserves close analysis and inspection. According the argument, the feasibility of Omni Inc. buying Mesa Foods and is highly doubted. It is strongly recommended of perform more specific observations and comparisons about both companies.

提前感谢楼下~谢谢!

给楼下的反馈:
非常感谢!恩!这是没限时写的,下一步该限时了,估计就会惨痛了。。。继续努力ing...
have a breathing spell 是“喘息时机,短暂的休息,考虑的机会”想表达“也不到这个人长出一口气的时候(后面问题还多着呢)”不知道能不能这么用……
后面那一句确实有点问题,改成The new problem is that weather markting DS throughout the country is feasible.是不是好一些~?
作者: cant0577    时间: 2010-7-16 19:25:33

本帖最后由 cant0577 于 2010-7-16 20:43 编辑

占位 改 L45

According to the argument, optimistic performance  of Mesa Foods, the snack food manufacturer, rationalize the decision of Omni Inc. to buy Mesa Foods, and to concentrate on marketing Diabolique Salsa, the best-seller of Mesa Foods. 非常简洁的复述了原题同时做出了让步,额,强势的语言功底啊However, either the purchase or the marking strategy is lack of consideration. If Omni execute this strategy, crisis is likely to be around the corner好用法 学习学习.个人觉得如果能把lack of consideration 稍微具体化一些,会更好 能让ETS一下子抓住你要攻击的点。

The merging plan should be on the top of reconsideration. Mesa Food is stressed to have “strong growth potential” in the argument, but not strongly supported by the data. Either 20 the increase in profit last year or the increase in sales of Diabolique Salsa only suggest that Mesa Food is experiencing increase in the past 犀利的评判. The arguer fails to provide enough hints, such as strong R&D ability, innovation, reliable quality control and mature marking strategy, to guarantee Mesa Food’s promising future. For instance, if the snacks are reported to contain poisoned ingredients after Omni taken charge of Mesa Food, Omni may be in endless trouble of consumer’s complaining, suffering credibility crisis; or, it may forced to afford huge compensation, leading to financial crisis.  直接攻击假设 质疑发展潜力

Even if Mesa Food is merit enough and have great potential, it is not guaranteed that buying it the a wise choice of Omni Inc. Instead, the Omni should evaluate the financial ability of the both company. If Omni is much smaller Mesa Food, and not enough credits or loans are available, it is nearly impossible of Omni to perform 学习学习啊 the purchase. Moreover, the argument only mentions that the target consumers of Omni are 14-to-25 year olds, but does not present the fact that Omni is in the food industry. 又是一个潜在假设,一般人会直接忽略Omi是什么类型的公司Instead, if Omni is a well performed youth clothing company, is it of high risk to buy a corporation of food industry, which is an unfamiliar field to Omni. 攻击潜在假设,是否有能力购买

If finally the entire  crisis and risks are predicted, and Omni is appropriate as well as having the ability to buy Mesa Food, the chef of Omni may not have a breathing spell 没有看懂. The new problem is, is marking Diabolique Salsa throughout the country feasible ? 这句话可以这么表达?两个谓语? It is highly doubted. Although DS is the bestseller of Mesa Food, neither the market share nor the growth rate of it is provided through out the argument. The acceptance degree of DS is unknown thus the sales situation in the future is hardly predicted. 学习学习。表达太精炼了 Provided that DS is well accepted, it is not guaranteed that DS is wide accepted, because the former market of DS is within a relatively small region of the country. If the flavor of DS is weird to other parts of the country, the promotion of DS will be more difficult, and might be a failure.
商品是否能被推被其他地区所接受
To summarize, purchasing a company and promote a new marketing strategy deserves close analysis and inspection. According the argument, the feasibility of Omni Inc. buying Mesa Foods and is highly doubted. It is strongly recommended of perform more specific observations and comparisons about both companies.

思路非常清晰 先攻击两个假设,在攻击是否能被其他地区接受。
语言能力强悍,表达非常干练,总之,跟你的一比,我的完全属于BABY ARGUMENT,几乎都在向你学习!!!onz

作者: cant0577    时间: 2010-7-16 19:26:14

本帖最后由 cant0577 于 2010-7-16 20:04 编辑

我也是A8 虽然自己改了一下,但是思路有限 还要麻烦楼下不吝赐教啊~~~~

TOPIC: ARGUMENT8 - The following appeared in a memorandum issued by the strategic planning department at Omni Inc.

"Mesa Foods, a manufacturer of snack foods that currently markets its products within a relatively small region of the country, has strong growth potential. Mesa enjoyed a 20 percent increase in profits last year, and its best-selling product, Diabolique Salsa, has had increased sales over each of the past three years. Since Omni Inc. is interested in reaching 14-to-25 year olds, the age group that consumes the most snack food, we should buy Mesa Foods, and concentrate in particular on marketing Diabolique Salsa throughout the country."
WORDS: 534
TIME: 01:19:51
DATE: 2010/7/15 21:28:31


In the argument, speaker suggests buy Mesa Foods, for its strong growth potential and it is popular with 14-to-25 young people. It seems that this action may get benefits and the products will be welcome in the whole country. However, after taking a second thought, we would find that the proof of the age group consuming the most snack food is not powerful and the assumption of strong growth potential is doubtful.

To begin with, it cannot be the powerful reason that Omni Inc. is interested in the largest group consuming the most snack food to buy Mesa Food. Firstly, there is not statistics to proof that the group aged from 14 to 25 consumes the most snack food. Is statistics that the most consumers of snack food are young people based on the small region or the whole country? If the statistics is just based on the small region, perhaps just young people there are the main consumer of snack food. And, the speaker does not provide the statistics to support that Omni Inc. is popular in this age group. It is entirely possible that Mesa Foods is the only one snack food company in the small region, and Diabolique Salsa is the best snack among all kinds of products. Therefore, the young group has to choose it as their "interested snack food" for they have no choice.

Even if it is granted that Omni Inc. is interested in young people, there is no powerful evidence to proof the assumption that Mesa Foods has strong growth potential. As mentioned in the argument, in last years, Mesa enjoyed a 20 percent increase in profits. Perhaps Mesa produces a new product and it raises people's curiosity to buy one to taste. After a period of time, people are possible never purchase it again. Moreover, maybe in the past three years, the number of population in the region increases. It is not unusual that the new inhabitants would stimulate the consumptions. And they, especially young people, would like to try the snack foods which they have not tasted before. Therefore, if the speaker can provide statistics about the sales of Mesa, and the population of the region in recent 3 years, it would be better to illustrate the growth potential of the snack food company.

Finally, the success of Mesa Foods reached in the relatively small region does not mean that it will also be popular throughout the country. Consumers living in different areas have quiet different tastes. Perhaps the consumers in the small regions enjoy hot food, however, in another region, consumers prefer sweet snack food. In addition, when the products are introduced into a new market, Mesa must come across a plenty of competitions from other snack food companies. Will Mesa share the snack market? For lacking the information about Mesa's products, we cannot safely assume that Mesa Foods will be welcome throughout the country.

Overall, it appears to receive success to buy Mesa Foods. According to the analyses above, however, we should take possible alternatives about the increasing profits Mesa enjoyed and we had better to make a poll before making final conclusions. After all, a false decision may lead to negative results.

作者: figuechen    时间: 2010-7-16 21:09:35

本帖最后由 figuechen 于 2010-7-17 20:06 编辑

In the argument, speaker suggests buy Mesa Foods, for its strong growth potential and it is popular with 14-to-25 young people. It seems that this action may get benefits and the products will be welcome in the whole country. However, after taking a second thought, we would find that the proof of [这个介词值得商榷]the age group consuming the most snack food is not powerful [这个形容词用得不对,证据不能说是powerful,可以说是persuasive]and the assumption of strong growth potential is doubtful.
To begin with, it cannot be the powerful [同上]reason that Omni Inc. is interested in the largest group consuming the most snack food to buy Mesa Food. [这句完全不通,建议自己多读两遍然后改一下吧,我不知道你想表达什么。。。]Firstly, there is not statistics to proof [prove]that the group aged from 14 to 25 consumes the most snack food. Is statistics that the most consumers of snack food are young people based on the small region or the whole country? If the statistics is just based on the small region, perhaps just young people [这个句子成分是什么?]there are the main consumer of snack food. And, the speaker does not provide the statistics to support that Omni Inc. is popular in this age group. It is entirely possible that Mesa Foods is the only one snack food company in the small region, and Diabolique Salsa is the best snack among all kinds of products. Therefore, the young group has [主语错误,young group是无法做choose的,只有人才可以]to choose it as their "interested snack food" for they have no choice.
[除了语法错误以外,该段废话太多,直到段落的最后才开始举例展开说明,例子之前的那几句话可以好好精简一下]
Even if it is granted that Omni Inc. is interested in young people, there is no powerful evidence to proof the assumption that Mesa Foods has strong growth potential. As mentioned in the argument, in last years, Mesa enjoyed a 20 percent increase in profits. Perhaps Mesa produces a new product and it raises people's curiosity to buy one to taste. After a period of time, people are possible never purchase it again. [It is possible for people not to purchase it again]Moreover, maybe in the past three years, the number of population in the region increases. It is not unusual that the new inhabitants would stimulate the consumptions. And they, especially young people, would like to try the snack foods which they have not tasted before. [这句话想表达的意思和上一句有什么区别么?]Therefore, if the speaker can provide statistics about the sales of Mesa, and the population of the region in recent 3 years, it would be better to illustrate the growth potential of the snack food company.
Finally, the success of Mesa Foods reached in the relatively small region does not mean that it will also be popular throughout the country. Consumers living in different areas have quiet [quite]different tastes. Perhaps the consumers in the small regions enjoy hot food, however, in another region, consumers prefer sweet snack food. In addition, when the products are introduced into a new market, Mesa must come across a plenty of competitions from other snack food companies. Will Mesa share the snack market? For lacking the information about Mesa's products, we cannot safely assume that Mesa Foods will be welcome throughout the country.
Overall, it appears to receive [???]success to buy Mesa Foods. According to the analyses above, however, we should take possible alternatives about the increasing profits Mesa enjoyed and we had better to make a poll before making final conclusions. After all, a false decision may lead to negative results.
[最后一段缺少对argument的改进部分]

综述:
作者在攻击点方面做得还不错,基本的攻击点都到位了,也都辅以例证来具体说明。但是作者在语言方面功底不够,单词拼写和语法错误层出不穷,不少句子成分前后颠倒,难以理解,造成阅读障碍。建议作者在语言方面认真加以改进。

祝共同进步!
作者: figuechen    时间: 2010-7-16 21:10:57

Argument17  The following appeared in a letter to the editor of the Walnut Grove town newspaper.                    
"Walnut Grove's town council has advocated switching from EZ Disposal (which has had the contract for trash collection services in Walnut Grove for the past ten years) to ABC Waste, because EZ recently raised its monthly fee from $2,000 to $2,500 a month, whereas ABC's fee is still $2,000. But the town council is mistaken; we should continue using EZ. EZ collects trash twice a week, while ABC collects only once. Moreover, EZ—which, like ABC, currently has a fleet of 20 trucks—has ordered additional trucks. Finally, EZ provides exceptional service: 80 percent of respondents to last year's town survey agreed that they were 'satisfied' with EZ's performance."


Word:605
未限时

    In this letter, the author claims that the decision of Walnut Grove’s town council to replace EZ Disposal for ABC Waste is mistaken. To support his claim, the author provides evidence to illustrate that EZ is superior to ABC in several aspects. The evidences are as follows: EZ collects trash more frequently; EZ will have more trucks; survey results showing the satisfactory attitudes of people towards EZ. Even if three facts are listed, there are several unwarranted assumptions and therefore the argument is unconvincing.
    To begin with, the author cannot draw the conclusion that EZ is superior according to the fact that EZ collects trash more frequently and has planned to purchase more trucks. The author unfairly assumes that the trash company which collects trash more frequently and owns more trucks is better, which is obviously not the case. Perhaps there is not so much trash in Walnut Grove town, and therefore it is adequate for trash company to collect trash once a week. The more frequent the company collects, the less efficient it is; or perhaps EZ takes charge of the trash of another two towns, and therefore the area that EZ takes charge of is far larger than that of ABC. Even if EZ owns more trucks, the trucks per area might be lower. Without considering these possibilities, the author cannot illustrate the point that EZ is superior with these weak evidences.
    Moreover, from the survey result, we cannot come to the claim that EZ provides exceptional service and therefore it is superior. First of all, the author does not provide evidence to prove that the respondents in this survey are representative to all the citizens in Walnut Grove. If not, the survey result might not correctly reflect the people’s attitudes. In addition, even if the result is statistically reliable, without comparable survey result on the satisfactory rate of ABC, the author too hastily concludes that EZ is superior. There exists the possibility that if a survey is to be conducted to evaluate the attitudes of citizens towards ABC, there will be 90% of residents who said they were “satisfied” with ABC’s performance. Unless the author provides extra information or survey results to eliminate these possibilities, I cannot accept the claim that EZ provides exceptional service and therefore it is superior.
    Finally, even if all those assumptions above are warranted and EZ is indeed superior than ABC, we cannot either draw the conclusion that Walnut Grove’s town council should choose EZ. The author neglects factors that might influence the decision other than the quality of the service. For example, we do not know whether the town council has enough money to pay EZ for the 50% rise in charge. If not, then ABC might not be a bad choice; another example is that the qualities of both companies are perfect and it is sufficient to have either one perform the trash collection. In this case, it is apparent that town council will prefer ABC since the cheaper, the better. If the author cannot provide further information in these aspects, he cannot convince me that the town council should choose EZ.
    In sum, the author has several mistaken or unwarranted assumptions and therefore the claim that EZ should be the first choice to town council is unconvincing. To better bolster his claim, the author had better provide information or evidence as follows: The amount of trash in Walnut Grove; the trucks per area of ABC and EZ; improved survey results which has a comparable result and whose respondents must be representative; the budget of the town council to make contract with the trash company.

恳请楼下狠拍,少谈优点,多说不足~先谢谢了
作者: beanie加油    时间: 2010-7-17 15:58:36

本帖最后由 beanie加油 于 2010-7-19 15:24 编辑

In this letter, the author claims that the decision of Walnut Grove’s town council to replace EZ Disposal for ABC Waste is mistaken. To support his claim, the author provides evidence to illustrate that EZ is superior to ABC in several aspects. The evidences are as follows: EZ collects trash more frequently; EZ will have more trucks; survey results showing the satisfactory attitudes of people towards EZ. Even if three facts are listed, there are several unwarranted assumptions and therefore the argument is unconvincing. 首先,第一段我可以看出你在尽量向阅卷者展示稳重的因果关系。但是,作为文章开头,你这种因果表达是很值得arguable的!全部意思复述了一遍,语言啰嗦,是最主要的毛病。个人建议可以这默写:In this letter, the editor tries to refute the position hold by the town council of cooperating with ABC Waste rather than EZ Disposal. To illustrate his stand, he subsequently lists the reasons that seem to persuasively articulate the advantages of choosing EZ Disposal. However, it is not well-reasoned
    To begin with, the author cannot draw the conclusion that EZ is superior according to the fact that
EZ collects trash more frequently and has planned to purchase more trucks. The author unfairly assumes that the trash company which collects trash more frequently and owns more trucks(重复吗?) is better, which is obviously not the case. Perhaps there is not so much trash in Walnut Grove town, and therefore it is adequate for trash company to collect trash once a week.(the first possibility) The more frequent the company collects, the less efficient it is; or perhaps EZ takes charge of the trash of another two towns, and therefore the area that EZ takes charge of is far larger than that of ABC. Even if EZ owns more trucks, the trucks per area might be lower.(the second possibility) Without considering these possibilities, the author cannot illustrate the point that EZ is superior with these weak evidences.两个possibility加最终一个without…,很好!!(Ps:文章结构用不同颜色标出来之后,中间那行黑体字你不觉得多余吗?)
    Moreover, from the survey result, we cannot come to the claim that EZ provides exceptional service and therefore it is superior.
First of all, the author does not provide evidence to prove that the respondents in this survey are representative to all the citizens in Walnut Grove. If not, the survey result might not correctly reflect the people’s attitudes. (第一个possibility,其实换种possibility更强我觉得:80%的回应者可能都是对EZ有好感的,那些没回应的很有可能是支持ABC的呀,therefore…..In addition, even if the result is statistically reliable, without comparable survey result on the satisfactory rate of ABC, the author too hastily concludes that EZ is superior. There exists the possibility that if a survey is to be conducted to evaluate the attitudes of citizens towards ABC, there will be 90% of residents who said they were “satisfied” with ABC’s performance. (第二个possibility,很有力的攻击,但是你不觉得用五行说太“详细”了吗?直接从第二句开始就可以了)Unless the author provides extra information or survey results to eliminate these possibilities, I cannot accept the claim that EZ provides exceptional service and therefore it is superior.
    Finally, even if all those assumptions above are warranted and EZ is indeed superior than ABC, we cannot either draw the conclusion that Walnut Grove’s town council should choose EZ. The author neglects factors that might influence the decision other than the quality of the service.
For example, we do not know whether the town council has enough money to pay EZ for the 50% rise in charge. (第一个possibility)If not, then ABC might not be a bad choice; another example is that the qualities of both companies are perfect and it is sufficient to have either one perform the trash collection.(第二个possibility,有点勉强,或许可以说ABC信誉好,收垃圾方法科学。。。额。。。可以考虑下)
In this case, it is apparent that town council will prefer ABC since the cheaper, the better. If the author cannot provide further information in these aspects, he cannot convince me that the town council should choose EZ.
    In sum, the author has several mistaken or unwarranted assumptions and therefore the claim that EZ should be the first choice to town council is unconvincing.
To better bolster his claim, the author had better provide information or evidence as follows: The amount of trash in Walnut Grove; the trucks per area of ABC and EZ; improved survey results which has a comparable result and whose respondents must be representative; the budget of the town council to make contract with the trash company.(同第一段缺点。。)
恩。。。既然你想让我猛拍,我就说啦。首先,写argument,你得搞清楚写的目的是神魔,不是他的观点符不符合你的意思,能不能说服你,而是你在看完他的reason之后,根据他那个什莫line of reasoning提出它所存在的不足,并予改正。你这篇文章可以自己数下有不少地让人感觉你是那个town council..…
关于语言,chienglish很严重。建议在思考时避免中文的思维和套路。多阅读些美国文章,有益无害。
给你回得晚了些,嘿嘿,抱歉阿。以上纯属个人意见,如有疑问可共同讨论!

作者: beanie加油    时间: 2010-7-17 15:59:33

TOPIC: ARGUMENT133 - The following is a memo from the principal of Academia High School.

"Academia High School should abolish its after-school performing-arts programs and replace them with computer-technology programs. When nearby Techno High School did so last year, total enrollment in all of its after-school activities remained about the same. Moreover, on entering college, many Techno students chose a major directly related to their after-school activities. On the other hand, last year only 10 percent of Academia's graduating seniors chose performing arts as their major field of study in their first year of college, clearly indicating that most students do not have a strong interest in the performing arts."


     The article is well-organized but not well-reasoned. By asserting that the total enrollment in all of its after-school activities remained the same and the only 10 percent of Academia's graduating seniors chose performing arts as their major, the speaker consequently concludes that most students are not strongly interested in the performing arts. However, it is certainly not the case.
     Mentioned about the total enrollment in its after-school activities in Techno High School, it is quite possible that students in that school are specialized in computer-technology programs. The abolishment of the performing-arts programs may influence only a small part of the students' preference on that field. But compared with the significance of their major learning, they are not reluctantly to attend the programs related to their major. Therefore, it explains why the total enrollment remained about the same.
     Secondly, the speaker reaches his position that most students do not have a strong interest in the performing arts in the case of only 10 percent of Academia's graduating seniors chose that as their major field of study in their first year of college. But chances are that many other students are more inclined to select some required courses in their first year of college, and when they turn to be a sophomore, a specialized major such as performing arts may be included in their consideration. Are the students whose major is not in performing arts indicating that it is not their favorite field? If not so, probably there are a number of students who have a strong interest in performing art but give up their dream in consideration of the social needs. Without ruling out the possible alternatives, the conclusion made by the principal is not as convincing as it stands.
     Also, the students in the two different schools have a great different in their individual interest and needs. For those in Techno High School, students are quite likely to be focused on computer-technology programs, whereas in Academia High School, students are driven by their interests in performing arts. Hence, the program can not be cancelled on account of the importance and necessity it owns in the students’ learning.
     Overall, the reasoning behind the abolishment of the performing arts programs seems logical as presented above since the principal is fully working best for the interest of the students in his school. However, before any final decisions are made about cancelling the programs, a more thorough evaluation should be taken concerning with its benefits of the policy.
不多说了,望猛拍,谢谢
作者: 追梦小木耳    时间: 2010-7-17 22:16:17

本帖最后由 追梦小木耳 于 2010-7-18 11:02 编辑


     The article is well-organized but not well-reasoned. By asserting that the total enrollment in all of its after-school activities remained the same and the only 10 percent of Academia's graduating seniors chose performing arts as their major, the speaker consequently concludes that most students are not strongly interested in the performing arts. However, it is certainly not the case.
     Mentioned about the total enrollment in its after-school activities in Techno High School, it is quite possible that students in that school are specialized in computer-technology programs. The abolishment of the performing-arts programs may influence only a small part of the students' preference on that field. But compared with the significance of their major learning, they are not reluctantly to attend the programs related to their major. Therefore, it explains why the total enrollment remained about the same.

这一段解释了为什么在T学校参加电脑活动的人和表演活动的人一样多。但是你只是解释了这个现象出现的原因,并没有和A学校类比。
     Secondly, the speaker reaches his position that most students do not have a strong interest in the performing arts in the case of only 10 percent of Academia's graduating seniors chose that as their major field of study in their first year of college. But chances are that many other students are more inclined to select some required courses in their first year of college, and when they turn to be a sophomore, a specialized major such as performing arts may be included in their consideration.
你是想说大一只学基础课,大二才选修专业课?但我认为题目中说的major就是指在大学中学的专业。就算学生把表演当选修课学了,也与题目中说课外活动决定major不相符合。Are the students whose major is not in performing arts indicating that it is not their favorite field? If not so, probably there are a number of students who have a strong interest in performing art but give up their dream in consideration of the social needs.我觉得这一段前面那些都没有说到点上,这一句才点明了你想表达的意思。 Without ruling out the possible alternatives, the conclusion made by the principal is not as convincing as it stands.
     Also, the students in the two different schools have a great
different difference in their individual interest and needs. For those in Techno High School, students are quite likely to be focused on computer-technology programs, whereas in Academia High School, students are driven by their interests in performing arts. Hence, the program can not be cancelled on account of the importance and necessity it owns in the students’ learning.
这句话意思模糊。

这一段假设了两个学校的学生兴趣有差异,那到底和课外活动有什么关系呢,与大学专业有什么关系呢?都没有说出来。
     Overall, the reasoning behind the abolishment of the performing arts programs seems logical as presented above since the principal is fully working best for the interest of the students in his school. However, before any final decisions are made about cancelling the programs, a more thorough evaluation should be taken concerning with its benefits of the policy.


我认为作者的这篇文章逻辑有些混乱,没有抓住题目中的攻击点。
我认为题目中的攻击点为:
1 错误的类比了两个学校。在T学校,将表演课换成电脑课,参加的人数一样多,并不表示同样的情况会在a学校出现。也许a学校的学生对电脑技术没有兴趣。
而作者只是解释了t学校换课后参加人数一样多,并没有指明A学校的情况。
2
课外活动决定专业是没有根据的。也许是因为电脑技术好就业,其实学生并不感兴趣。

这一点作者倒是说到了,但是没有详细展开。可以说一说可能是现在社会的需求、就业的形势什么的
3
即使课外活动决定专业,那么10%的数字也不能说明A学校学生对表演没有兴趣。也许这个学校的总人数很多,10%说明有很大的需求。而换成别的可能还不到10%

作者说了两学校学生兴趣的不同,到底是什么情况,却没有给出解释。

还有一点攻击力不够的原因是,缺少详细的解释。虽然你给出了一些假设,但是都缺少深入的细节,很难让人相信假设的成立

以上为个人意见,欢迎反驳

作者: 追梦小木耳    时间: 2010-7-17 22:16:41

首先感谢所有帮我改文的童鞋!
还有10天,第一次计时,没写完,已补充并修改,请楼下同学指教!

TOPIC: ARGUMENT161 - In a study of reading habits of Leeville citizens conducted by the University of Leeville, most respondents said they preferred literary classics as reading material. However, a follow-up study conducted by the same researchers found that the type of book most frequently checked out of each of the public libraries in Leeville was the mystery novel. Therefore, it can be concluded that the respondents in the first study had misrepresented their reading habits.
WORDS: 251          TIME: 00:30:00          DATE: 2010-7-17 21:21:25

Final words 458

The argument sounds sensible by comparing the two study results, in which one indicated that the residents like literary classics while the other demonstrated that mystery novels are their favorite. However, the conclusion that the first study has misrepresented the reading habits is presumptuous, which obviously ignores other possibilities.

In the first place, one must question the ways in which the two studies were conducted. Probable, the respondents of the first study and the frequent clients of public libraries are totally two groups of people. As for the first study, are the participants limited to a certain age range, who happen to like classic literature? It is entirely possible that the elder people in the town prefer classic literature since they could hardly accept the modern conceptions presented in contemporary literature. If the respondents are limited to them, the result would be misleading. On the other hand, the result of the second study remains questionable. The researcher fails to consider the main clients of the public libraries. Perhaps, it is only a certain small groups of people like teenagers who went there frequently and kept checking mysterious novels. If so, the result is not convincing.

In addition, only judging from the simple comparison of the two studies' results one can hardly decide the reading habits of the town. Perhaps, the literary classics are the most popular books, and therefore, most residents bought the books and repeated reading them rather than borrow them from libraries. In this case, the researchers must study the sales of all kinds of books, and account for this study results as well. There is also possibility that the clients in libraries were merely checking the mysterious novels out of curiosity but not truly reading them. Thus, the study conducted only by discovering the books that have been checked for most times in public libraries can not reflect residents’ reading habit either.

Finally, the arguer also fails to take into consideration time difference of the two studies. Are they conducted at the same time or separately? If it is the latter case, the reading habits of the residents probably are affected by other factors. For instance, most people prefer literary classics, which was correctly revealed in the first study. But when the second study was proceeding, mysterious books became popular due to one popular mysterious movie, which arouse people’s interest, therefore causing relevant novel’s popularity in public libraries.

To conclude, as the two studies were conducted in questionable ways and their ignorance of some crucial information, neither can they reveal the reading habits of Leevile citizens. In order to make the argument forceful, the researchers should not only examine the ways the above two studies were conducted, but also account for local sales of books.
作者: 费话先生    时间: 2010-7-18 00:48:33

本帖最后由 费话先生 于 2010-7-18 01:21 编辑

这篇文章我写过。。。所以我将尽力改之
The argument sounds sensible by comparing the two study results, in which one indicated that the residents like literary classics while the other demonstrated that mystery novels are their favorite. However, the conclusion that the first study has misrepresented the reading habits is presumptuous, which obviously ignores other possibilities.

In the first place, one must question the ways in which the two studies were conducted. Probable(
用副词), the respondents of the first study and the frequent clients of public libraries are totally two groups of people. As for the first study, are the participants limited to a certain age range, who happen to like classic literature? It is entirely possible that the elder people in the town prefer classic literature since they could hardly accept the modern conceptions presented in contemporary literature. If the respondents are limited to them, the result would be misleading. On the other hand, (哎,前面都没有on one hand,我香港的老师说的,好像没有只用一半的情况哦,虽然我原来也经常用。。。而且是表对比,而非并列)
the result of the second study remains questionable. The researcher fails to consider the main clients of the public libraries. Perhaps, it is only a certain small groups of people like teenagers who went there frequently and kept checking mysterious novels. If so, the result is not convincing.

In addition, only judging from the simple comparison of the two studies' results one can hardly decide(reveal) the reading habits of the town. Perhaps, the literary classics are the most popular books, and therefore, most residents bought the books and repeated reading them rather than borrow them from libraries. In this case, the researchers must study the sales of all kinds of books, and account for this study results as well. There is also possibility that the clients in libraries were merely checking the mysterious novels out of curiosity but not truly reading them.
(这点其实有点牵强。。。去图书馆借书其实不是为了看的。。这比例能有多大呢。。。)
Thus, the study conducted only by discovering the books that have been checked for most times in public libraries can not reflect residents’ reading habit either.

Finally, the arguer also fails to take into consideration time difference of the two studies. Are they conducted at the same time or separately? If it is the latter case, the reading habits of the residents probably are affected by other factors.
(我觉得要突出中间时间长,否则虽说是分开做,如果时间就个把月,那么可信度又下降了,if the time span between the two surveys is considerable long, the habits of residents probably have changed obviously during the period
For instance, most people prefer literary classics, which were correctly revealed in the first study. But when the second study was proceeding, mysterious books became popular due to one popular mysterious movie, which arouse people’s interest, therefore causing relevant novel’s popularity in public libraries.

To conclude, as the two studies were conducted in questionable ways and their ignorance of some crucial information, neither can they reveal the reading habits of Leevile citizens. In order to make the argument forceful, the researchers should not only examine the ways the above two studies were conducted, but also account for local sales of books.


     我觉得这篇是典型的调查问题类文章,楼主的框架都不错, 语言错误也很少,不过我觉得在他因列举上可以再向insightful和reasonable上靠拢。这篇文章我自己当时限时也失败了。。。主要是攻击点其实就俩。。。然后他因一大堆。。。然后我就忧郁了。。。版友结尾写得很好!
作者: 费话先生    时间: 2010-7-18 01:06:07

ARGUMENT20 - The following appeared in a letter to the editor of the Balmer Island Gazette.

"The population of Balmer Island increases to 100,000 duing the summer months. To reduce the number of accidents involving mopeds and pedestrians, the town council of Balmer Island should limit the number of mopeds rented by each of the island's six moped and bicycle rental companies from 50 per day to 30 per day during the summer season. By limiting the number of rentals, the town council is sure to attain the 50 percent reduction in moped accidents that was achieved last year in the neighboring island of Torseau, when Torseau's town council enforced similar limits on moped rentals."

   Merely based on a series of unfound assumptions and vague facts, the author suggests limiting the number of mopeds rented by each of the Balmer Island’s (B) six rental companies to 30 per day during the summer season for the purpose of reducing accidents involving mopeds and pedestrians. However, the suggestion is unwarranted and hasty without close scrutiny,
    The first assumption is that the cause of accidents is due to the overmany rented moped and bicycles. However, the author fails to provide any convincing evidence to support it. It is possible that most of accidents are due to the carelessness of pedestrians who ignore the traffic rules. Or perhaps the majority of accidents occurs in harsh weather; especially the rainy days which may not be conduce to brake in time. Thus, it is more reasonable to educate pedestrians to obey the traffic rules or install the warning signs during the bad weather or on some dangerous curves to decrease the number of accidents.  
     Another threshold assumption in the argument is that the only means to access to mopeds and bicycles is renting. Yet, here may be not the case. Residents in B Island are more likely to buy mopeds and bicycles for their own because of the cheap price and convenience to use. If the council of B Island limits the number of mopeds rented, people would possibly turn to buy them a moped rather than to rent one. Another possible situation is that the number of companies providing renting service increases, if the demands of residents remain strong like the present. Naturally, despite the limits on moped rentals, the number of mopeds in street may turn out to increase at last. Therefore, the author should evaluate all possible means to access to moped if he really wants to decrease the mopeds in street.
    In addition, the author infers the conclusion that B island will have the similar effects like T island after adopting the limits from an improper analogy. However, the author does not provide any details to demonstrate that the conditions like population, the consuming habits and the demographic data in the two islands are nearly the same. If the majority of mopeds users are tourists in T Island, the limits may effect well since the tourists are less likely to buy mopeds themselves. Whereas most mopeds users in B Island are children, they will ask their parents to buy mopeds for them instead of giving up their habits, which will reduce the effect of limits. Until the author demonstrates that the two islands are nearly same and hence the effects of limits, the author should not hastily follow the example of T Island.
    Overall, the advice by the author is poor reasoned indeed. To bolster it, the author should provide more details about the causes of accidents happened in B Island. He is also suggested to evaluate the possible effect of the limit; what’s more, he’d better prove that the experience of the other island will work as well.
作者: beanie加油    时间: 2010-7-18 20:00:55

本帖最后由 beanie加油 于 2010-7-21 16:55 编辑

duing the summer months. To reduce the number of accidents involving mopeds and pedestrians, the town council of Balmer Island should limit the number of mopeds rented by each of the island's six moped and bicycle rental companies from 50 per day to 30 per day during the summer season. By limiting the number of rentals, the town council is sure to attain the 50 percent reduction in moped accidents that was achieved last year in the neighboring island of Torseau, when Torseau's town council enforced similar limits on moped rentals."

   Merely based on a series of unfound assumptions and vague facts, the author suggests limiting the number of mopeds rented by each of the Balmer Island’s (B) six rental companies to 30 per day during the summer season for the purpose of reducing accidents involving mopeds and pedestrians过于啰嗦. However, the suggestion is unwarranted and hasty without close scrutiny,
    The first assumption is that the cause of accidents is due to the overmany
没这词。。rented moped and bicycles. However, the author fails to provide any convincing evidence to support it. It is possible that most of accidents are due to the carelessness of pedestrians who ignore the traffic rules. Or perhaps the majority of accidents occurs in harsh weather; especially the rainy days which may not be conduce to brake in time.两个possibility,good Thus, it is more reasonable to educate pedestrians to obey the traffic rules or install the warning signs during the bad weather or on some dangerous curves to decrease the number of accidents.解决方案,good!  
     Another threshold assumption in the argument is that the only means to access to mopeds and bicycles is renting. Yet, here may be not the case. Residents in B Island are more likely to buy mopeds and bicycles for their own because of the cheap price and convenience to use. If the council of B Island limits the number of mopeds rented, people would possibly turn to buy them a moped rather than to rent one. Another possible situation is that the number of companies providing renting service increases, if the demands of residents remain strong like the present. Naturally, despite the limits on moped rentals, the number of mopeds in street may turn out to increase at last. Therefore, the author should evaluate all possible means to access to moped if he really wants to decrease the mopeds in street.
    In addition, the author infers the conclusion that B island will have the similar effects like T island after adopting the limits from an improper analogy. However, the author does not provide any details to demonstrate that the conditions like population, the consuming habits and the demographic data in the two islands are nearly the same. If the majority of mopeds users are tourists in T Island, the limits may effect well since the tourists are less likely to buy mopeds themselves.
more likely to rent… Whereas most mopeds users in B Island are children, they will ask their parents to buy mopeds for them instead of giving up their habits, which will reduce the effect of limits. Until the author demonstrates that the two islands are nearly same and hence the effects of limits, the author should not hastily follow the example of T Island.这段明白你再说什莫,但是说得有点不清楚。。
    Overall, the advice by the author is poor reasoned indeed. To bolster it, the author should provide more details about the causes of accidents happened in B Island. He is also suggested to evaluate the possible effect of the limit; what’s more, he’d better prove that the experience of the other island will work as well.

这篇文章逻辑结构很好,body第一段说many other possibilities cause to accidents.第二段说即使limit the mopeds rental, residents may choose to buy them. 质疑了方法的准确性,good!!!第三段比较了两个岛的不同。个人感觉第二段的逻辑很好!因为我的时候没考虑到这个,嘿嘿。总体来说逻辑没问题,有点问题的是语言,个别地方有语法错误。
Good!


the second edtion:
duing the summer months. To reduce the number of accidents involving mopeds and pedestrians, the town council of Balmer Island should limit the number of mopeds rented by each of the island's six moped and bicycle rental companies from 50 per day to 30 per day during the summer season. By limiting the number of rentals, the town council is sure to attain the 50 percent reduction in moped accidents that was achieved in last year in the neighboring island of Torseau, when Torseau's town council enforced similar limits on moped rentals."

   Merely based on a series of unfound assumptions and vague facts, the author suggests limiting the number of mopeds rented by each of the Balmer Island’s (B) six rental companies to 30 per day during the summer season for the purpose of reducing accidents involving mopeds and pedestrians
过于啰嗦. 可以变为In order to reduce accidents involving mopeds and pedestrians, the author suggests the rental companies to limit their number of mopeds rented to 30 per day,因为你这段的第一句话和最后一句话说的是一个意思
However, the suggestion is unwarranted and hasty without close scrutiny,
    The first assumption is that the cause of accidents is due to the overmany
没这词。。
rented moped and bicycles. However, the author fails to provide any convincing evidences to support it. It is possible that most of accidents are due to the carelessness of pedestrians who ignore the traffic rules. Or perhaps the majority of accidents occur(s) in harsh weather; especially the rainy days which when it may not be conduce(conducive) to brake in time.两个possibility,good Thus, it is more reasonable to educate pedestrians to obey the traffic rules or install the warning signs during (the) bad weather or on some dangerous curves to decrease the number of accidents.解决方案,good!  
     Another threshold assumption in the argument
is
两个is不重复吗?可以用involves that the only mean(s) to access to mopeds and bicycles is renting. Yet, here may be not the case. Residents in B Island are more likely to buy mopeds and bicycles for their own(private use) because of the cheap price and convenience to use(its cheap price and convenience). If the council of B Island limits the number of mopeds rented, people would possibly turn to buy them a moped(buy a moped for them) rather than to rent one. Another possible situation is that the number of companies providing renting service increases, if the demands of residents remain strong like the present.(这个。。主将从现吧。Another probablity is that if the residents’ interest to the moped remain…., chances are that the number of companeis…will be subsequently increased) Naturally, despite the limits on moped rentals, the number of mopeds in street may turn out to increase at last. Therefore, the author should evaluate all possible means to access to moped if he really wants to decrease the mopeds in street.感觉这个反例不是很strong。。
    In addition, the author hastily infers
to
the conclusion that B island will have the similar effects like(with) T island after adopting the limits(the policy of limitation on the number of moped rental) from an improper analogy
(不要,啰嗦,前面加一个hastily. However, the author does not provide any details to demonstrate that the conditions like(such as) population, the consuming habits and the demographic data in the two islands are(用太多了,remain) nearly the same. If the majority of mopeds users are tourists in T Island, the limits may effect well(then the proposal would be well-effective) since the tourists are less likely to buy mopeds themselves.more likely to rent…(for themselves instead more willing to rent) Whereas most mopeds users in B Island are children, they will ask their parents to buy mopeds for them instead of giving up their habits, which will reduce the effect of limits. Until the author demonstrates that the two islands are nearly same and hence the effects of limits,(Unless the author demonstrates the nealy same situations in the two islands)
the author should not hastily follow the example of T Island.
这段明白你再说什莫,但是说得有点不清楚。。
    Overall, the advice by the author is poor reasoned indeed(the advice offered by the author is indeed poor-reasoned). To bolster it, the author should provide more details about the causes of accidents happened in B Island. He is also(Also, it would be better for he to evaluate the..)suggested to evaluate the possible effect of the limit; what’s more, he’d better prove that the experience of the other island will work as well.

作者: beanie加油    时间: 2010-7-18 20:01:39

TOPIC: ARGUMENT53 - Thirteen years ago, researchers studied a group of 25 infants who showed signs of mild distress when exposed to unfamiliar stimuli such as an unusual odor or a tape recording of an unknown voice. They discovered that these infants were more likely than other infants to have been conceived in early autumn, a time when their mothers' production of melatonin-a hormone known to affect some brain functions-would naturally increase in response to decreased daylight. In a follow-up study conducted earlier this year, more than half of these children-now teenagers-who had shown signs of distress identified themselves as shy. Clearly, increased levels of melatonin before birth cause shyness during infancy and this shyness continues into later life.
WORDS: 279 REVISED: 464       TIME: 00:30:00  1:00:00      DATE: 2010-7-18 19:07:58

     The article is well-organized, but not well-reasoned. By claiming that the group of infants who showed signs of mild distress was due to their mothers' production of melatonin and this kind of hormone is responsible for the later shyness of these children. The argument seems to be accurate and convincing at first glance. However, a deep analysis reveals many critical fallacies which challenge the strong stand it shows.
     Foremost, the researchers assert that the mild distress of the group of infants comes from the melatonin their mother product, but it is actually not the case. Chances are that their reactions to the phenomenon could be explained by other genetic or environmental factors instead of the melatonin. For instance, probably when they are brought to a new environment such as to the laboratory, the unfamiliar situation causes them to feel distressed rather than what the unusual odor or the unknown voice work on them. Or facts are quite likely that it is many other hormones not the melatonin that push them to be nervous under that circumstance. And melantonin, described in the argument as a hormone known to affect some brain-functions-would naturally increase their curiosity to the unknown stuff. Without ruling out these possibilities, the researchers can not attribute the infants' distress to their mothers' production of melatonin.
     A follow-up study shows that most children who had felt mildly distress identified themselves as shy. However, I have to reject the argument because of the non-relationship between the two words. To define the term shyness, we need to start from both the genetical factors and the environmental factors. On account of the former one, it is quite the chance that those children's parents are introvert and therefore this kind of personality leads to the same shyness in their next generation. But the latter factor plays a role as indispensable as the first one. Probably their characters are shaped largely by the environment where they grow up or because the negative effects they are influenced in some certain types of things, which all can best explain their shyness. Hence, these would have to be ruled out in order for the researchers’ position to be valid.
     Overall, we can get to know that the researchers try to create a relationship between the infants' signs of mild distress, their mothers' production of melatonin and the later shyness. However, lacking of thinking about many counter examples that may refute the argument, they hastily reach the conclusion that it is the malatonin that causes the children's shyness both in their infancy and in their later life. So in order to perceive a clearer and more accurate answer of distress and shyness in the studies, the researchers need to take a comprehensive look upon the existing possibilities that may function on them.   
欢迎往死拍
作者: 秋尽江南    时间: 2010-7-18 23:43:29

本帖最后由 秋尽江南 于 2010-7-19 01:02 编辑

TOPIC: ARGUMENT53 - Thirteen years ago, researchers studied a group of25 infants who showed signs of mild distress when exposed to unfamiliarstimuli such as an unusual odor or a tape recording of an unknownvoice. They discovered that these infants were more likely than otherinfants to have been conceived in early autumn, a time when theirmothers' production of melatonin-a hormone known to affect some brainfunctions-would naturally increase in response to decreased daylight.In a follow-up study conducted earlier this year, more than half ofthese children-now teenagers-who had shown signs of distress identifiedthemselves as shy. Clearly, increased levels of melatonin before birthcause shyness during infancy and this shyness continues into later life.
WORDS: 279 REVISED: 464       TIME: 00:30:00  1:00:00      DATE: 2010-7-18 19:07:58

     The article is well-organized, but not well-reasoned. By claimingthat the group of infants who showed signs of mild distress was due totheir mothers' production of melatonin and this kind of hormone isresponsible for the later shyness of these children.(这句话的谓语部分…我没有找到…) The argument seemsto be accurate and convincing at first glance. However, a deep analysisreveals many critical fallacies which challenge the strong stand itshows.
     Foremost(好词~), the researchers assert that the mild distress of thegroup of infants comes from the melatonin their mother product, but itis actually not the case(毕竟否定作者提出的理由也是基于猜测,是不是改成it might not be the case更合适一些?). Chances are that their reactions to thephenomenon could be explained by other genetic or environmental factorsinstead of the melatonin. (接下来的例子是按照environmental and genetic的顺序,倒过来和前文呼应是不是更顺畅一些?)For instance, probably when they are broughtto a new environment such as to the laboratory, the unfamiliarsituation causes them to feel distressed rather than what the unusualodor or the unknown voice work on them. Or facts are quite likely thatit is many other hormones not the melatonin that push them to benervous under that circumstance. And melantonin, described in theargument as a hormone known to affect some brain-functions-wouldnaturally increase their curiosity to the unknown stuff. (这一段关于其他原因的分析很详细,展开得相当深入,学习~)Without rulingout these possibilities, the researchers can not attribute the infants'distress to their mothers' production of melatonin.
     A follow-up study shows that most children who had felt mildlydistress identified themselves as shy. However, I have to reject theargument because of the non-relationship between the two words(指代不明确). Todefine the term "shyness", we need to start from both the geneticalfactors and the environmental factors. On account of the former one, itis quite the chance that those children's parents are introvert andtherefore this kind of personality leads to the same shyness in theirnext generation. But the latter factor plays a role as indispensable(好词) asthe first one. Probably their characters are shaped largely by theenvironment where they grow up or because the negative effects they areinfluenced in some certain types of things, which all can best explaintheir shyness.(... are shaped because...感觉不是很顺,or 后面的句子自成一句在语法和表达上都会更容易掌控一些吧?) Hence(学习连接词的用法~), these would have to be ruled out in order for theresearchers’ position to be valid.
     Overall, we can get to know that the researchers try to create arelationship between the infants' signs of mild distress, theirmothers' production of melatonin and the later shyness. However,lacking of thinking about many counter examples that may refute theargument, they hastily reach the conclusion that it is the malatoninthat causes the children's shyness both in their infancy and in theirlater life. So in order to perceive a clearer and more accurate answerof(to?) distress and shyness in the studies, the researchers need to take acomprehensive look upon the existing possibilities that may function onthem.   (结尾总结的相当完整~)

对于错误原因的分析相当深入(这也正是我很欠缺的地方,学习ing...)展现了相当的词汇量(继续学习…)
可能是我读长句的能力有问题,感觉文中的长句过多,有些语法问题也导致理解障碍,或许可以考虑长短句结合一下?
鄙人新手,意见多有不准确之处,欢迎PM赐教交流~
作者: 秋尽江南    时间: 2010-7-18 23:45:37

首先感谢改文的童鞋~辛苦辛苦~
第一次限时、第一次写调查类的argument。
提纲:出借次数最多的书为必是市民最喜欢的读物
          从公立图书馆收集的数据未必能反映人们的阅读习惯
         两次调查间隙未知,市民的阅读习惯可能发生变化。

看到53L 400+的argu,很是受挫啊……

TOPIC: ARGUMENT161 - In a study of reading habits of Leeville citizens conducted by the University of Leeville, most respondents said they preferred literary classics as reading material. However, a follow-up study conducted by the same researchers found that the type of book most frequently checked out of each of the public libraries in Leeville was the mystery novel. Therefore, it can be concluded that the respondents in the first study had misrepresented their reading habits.
WORDS: 345-368          TIME: 00:30:00          DATE: 2010/7/18 22:47:39

In this argument, the author concludes that the first study of reading habits of Leeville citizens misrepresented their reading habits on the basis of the comparison between the results of two studies on the same topic. The conclusion may seem reasonable somehow, however, it is weakened by incomplete comparison and unwarranted assumptions.

The threshold problem with this argument is that the author assumes that the most frequently checked out books in the public libraries are readers' favorites, which might not be the case. Common sense tells that if one put high value on a book, he/she would rather buy it instead of borrowing it from library. It is entirely possible that people would like to buy literary classics instead of borrowing them from libraries. With the unjustified assumption, the conclusion is problematic.

Even assuming that most frequently-checked-out books reflect citizens' preference, data merely collected from public libraries is insufficient. Perhaps there are mainly mystery novels in public libraries, so naturally the most-frequently-checked-out books would be mystery novels. Yet this result is of little to do with the citizens’ reading habits. Or perhaps most Leeville citizens go to private libraries while only a small proportion go to public ones, for that matter, the data in the study cannot represent the preference of the majority citizens. Thus the conclusion based on the one-side data is unsound.

Moreover, though the follow-up study was conducted by the same researchers, we cannot conclude that the second study represents the citizens' reading habits. The argument offers no information about the interval between the two studies, maybe years had passed before the second study was conducted, and during that period, citizens' reading habits might change as well. Without more information about the two studies, the author cannot convince me that the results of the two studies are comparable.

In sum, the argument is unconvincing for its incomplete comparison between the results of the two studies and vague details of them. To better support the argument, it is necessary for the researchers to investigate a wide range of libraries or bookstores and thus describe the reading habits of Leeville citizens precisely. Plus, more detailed information about the way that researchers conduct the studies is in need.
作者: 396857815    时间: 2010-7-19 10:40:58

本帖最后由 396857815 于 2010-7-19 10:59 编辑

TOPIC: ARGUMENT161 - In a study of reading habits of Leeville citizens conducted by the University of Leeville, most respondents said they preferred literary classics as reading material. However, a follow-up study conducted by the same researchers found that the type of book most frequently checked out of each of the public libraries in Leeville was the mystery novel. Therefore, it can be concluded that the respondents in the first study had misrepresented their reading habits.
WORDS: 345-368          TIME: 00:30:00          DATE: 2010/7/18 22:47:39

In this argument, the author concludes that the first study of reading habits of Leeville citizens misrepresented their reading habits on the basis of(换成based on更简洁一些) the comparison between the results of two studies on the same topic. The conclusion may seem reasonable somehow, however, it is weakened by incomplete comparison and unwarranted assumptions(粉色是好词好句).

The threshold problem with this argument is that the author assumes that the most frequently checked out books in the public libraries are readers' favorites, which(这里还是不用内个什么非线性从句比较好吧,直接说it might not be the case会更加有节奏一些 might not be the case. Common sense tells me that if one put high value on a book, he/she would rather buy it instead of borrowing it from library. It is entirely possible that people would like to buy literary classics instead of borrowing them from libraries. With the unjustified assumption, the conclusion is problematic.

Even assuming that most frequently-checked-out books reflect citizens' preference, data merely collected from public libraries is insufficient. Perhaps there are mainly mystery novels in public libraries, so naturally the most-frequently-checked-out books would be mystery novels.(这句稍显Chinglish,尤其是mainly和naturally这两个词的使用,不够地道) Yet this result is of little to do with the citizens’ reading habits. Or perhaps most Leeville citizens go to private libraries while only a small proportion go to public ones, for that matter, the data in the study cannot represent the preference of the majority citizens. Thus the conclusion based on the one-side data is unsound. 后面这两句越读越心水哈~赞一个!还有就是data我见用的不很多,建议换成statistics

Moreover, though the follow-up study was conducted by the same researchers, we cannot conclude that the second study represents the citizens' reading habits. The argument offers no information about the interval between the two studies, maybe years had passed before the second study was conducted, and during that period, citizens' reading habits might change as well. Without more information about the two studies, the author cannot convince me that the results of the two studies are comparable.
这段很流畅,能感觉出来语言功底很棒,赞一个!

In sum, the argument is unconvincing for its incomplete comparison between the results of the two studies and vague details of them. To better support the argument, it is necessary for the researchers to investigate a wide range of libraries or bookstores and thus describe the reading habits of Leeville citizens precisely. Plus, more detailed information about the way that researchers conduct the studies is in need.
语言方面都很不错,但总觉得论证力度不够,纵观全文看来还是主要字数不够多,所以内容部那么充实,总体来说语言很棒,很流畅,基本的点也都抓住了,在加大论证力度就能算是篇很棒的文了,加油!!
作者: 396857815    时间: 2010-7-19 10:41:30

本帖最后由 396857815 于 2010-7-19 11:04 编辑

TOPIC: ARGUMENT209 - The following recommendation was made by the Human Resources Manager to the board of directors of the Fancy Toy Company.

"In the last three quarters of this year, under the leadership of our president, Pat Salvo, our profits have fallen considerably. Thus, we should ask for her resignation in return for a generous severance package. In Pat's place, we should appoint Rosa Winnings. Rosa is currently president of Starlight Jewelry, a company whose profits have increased dramatically over the past several years. Although we will have to pay Rosa twice the salary that Pat has been receiving, it will be well worth it because we can soon expect our profits to increase considerably."
WORDS: 496          TIME: 00:30:00          DATE: 7/19/2010 10:19:33 AM

  In this argument the manager recommends us to replace our current president Pat Salvo with the current president of the Starlight Jewelry Company Rosa Winnings. The manager cites a fact that he or she thought would be the evidence of the reasonable recommendation. However, the mere fact that our company's profit has declined during the last three quarters under the Pat Salvo's leadership while Starlight's profit has increased during the past several years with the current president Rosa Winnings obviously cannot verify the conclusion. With some logical flaws existing the recommendation is not that rational and unambiguous.
  First of all, there is no evidence can prove that it is our president, Pat Salvo, who is responsible for the falling profits. The manager has masked the other alternative possibilities that may also contribute to the profits decline. For example, the rising costs of materials and labor can make our expend increase; the probably whole toy business's depression can make our marketing not as well as before; and so forth. Before these possibilities stated above have been ruled out, the conclusion that it is Pat Salvo, our president, who is responsible for the profits decline is too slapdash and unreliable.
  Secondly, same with the flaws state above, there is also no evidence can prove that it is Rosa Winnings, the current president of Starlight, is responsible for the past several years' increasing profit. There may have other possibilities that can also lead to the profits increasing. For instance, if the whole jewelry business is prospective, may be Rosa's management ability is not good enough; The profit of Starlight can still increase. Or, maybe during the past several year, it is not Rosa who is in charge of Starlight, the increasing profits is totally owing to the predecessor of Starlight but not the current president Rosa.
  Finally, the management has masked another important factor which makes the conclusion totally not rational. The factor is the discrepancy between the toy business and jewelry business. No evidence or information is given to prove that the work experience and management skill in jewelry business can also work in toy business. Moreover, we cannot verify that Rosa's skill and experience in Starlight can also work in Fancy. Maybe the two companies in two different businesses are totally different in every aspects of management. Therefore, even if Rosa is very successful in Starlight, it cannot demonstrate he or she can also be successful in Fancy.
  In conclusion, the manager's recommendation is totally unconvincing with the logical flaws stated above. To strengthen the conclusion, the manager has to prove that the cause of our company's decreasing profits is exactly Pat Salvo. And more information and facts is needed to demonstrate that Rosa is the one who is responsible for the Starlight's increasing profits, not other factors. In addition, to convince to replace Pat with Rosa, there also must be verify that Rosa's experience and skill can also success in our company; Thus, our company can be assured to have an increasing profit considerably.
作者: tyarel    时间: 2010-7-19 21:04:14

本帖最后由 tyarel 于 2010-7-21 10:33 编辑

占楼,限时argument还写到500去了,吓到我了...

TOPIC: ARGUMENT209 - The followingrecommendation was made by the Human Resources Manager to the board ofdirectors of the Fancy Toy Company.

"In the last three quarters of this year, under the leadership of ourpresident, Pat Salvo, our profits have fallen considerably. Thus, we should askfor her resignation in return for a generous severance package. In Pat's place,we should appoint Rosa Winnings. Rosa is currently president of StarlightJewelry, a company whose profits have increased dramatically over the pastseveral years. Although we will have to pay Rosa twice the salary that Pat hasbeen receiving, it will be well worth it because we can soon expect our profitsto increase considerably."
WORDS: 496          TIME: 00:30:00         DATE: 7/19/2010 10:19:33 AM

  In this argument the manager recommends us to replace our current president Pat Salvo with the current president of the Starlight Jewelry Company Rosa Winnings.
The manager cites a fact that he or she thought would be the evidence of the reasonable recommendation. (个人觉得这句话没什么实际意义,纯粹凑字数的话还不如不要) However, the mere fact that our company' s profit has declined during the last three quarters under the Pat Salvo's leadership while Starlight's profit has increased during the past several years with the current president Rosa Winnings obviously cannot verify the conclusion. With some logical flaws existing the recommendation is not that rational and unambiguous.
对首段提点我自己的看法。这篇Argument是建立在这样一个逻辑上的,FTC业绩下降+SJ业绩上升→R的到来可以大幅提升FTC业绩我们应该雇佣R。不管怎么写,关键是把逻辑线抓清楚。所以我觉得以你这样复述的写法的话还有些遗漏,至少从第一段看整个逻辑没有完全出来。

  First of all, there is no evidence can prove that it is our president, Pat Salvo, who is responsible for the falling profits. The manager has masked the other alternative possibilities (alternatives就可以了…) that may also contribute to the profits decline. For example, the rising costs of materials and labor can make our expend increase; the probably whole toy business's depression can make our marketing not as well as (it wasmarketmarket比,不是marketbefore比较) before; and so forth. Before these possibilities statedabove have been ruled out, the conclusion that it is Pat Salvo, our president, who is responsible for the profits decline is too slapdash(这个单词会不会用得太刻意?个人感觉…) and unreliable.(建议还是换个句式,开头结尾都用同一个感觉有点单调)

  Secondly, same with the flaws state above, there is also no evidence can prove that it is Rosa Winnings, the current president of Starlight, (who)is responsible for the past several years' increasing profit. (…这个句式又来了…) There may have(be) other possibilities that can also lead to the profits increasing. For instance, if the whole jewelry business is prospective, may be (maybe) Rosa's management ability is not good enough The profit of Starlight can still increase. Or, maybe during the past several year, it is not Rosa who is in charge of Starlight, (去掉逗号用and连接,逗号不能连接2个完整的句子)the increasing profits is totally owing to the predecessor of Starlight but not the current president (也可以省掉) Rosa.

  Finally, the management has masked another important factor which makes the conclusion totally not rational. The factor is the discrepancy between the toy business and jewelry business. No evidence or information is given to prove that the work experiences and management skills in jewelry business can also work in toy business. Moreover,we cannot verify that Rosa's skill and experience in Starlight can also work in Fancy.(和前一句意思其实是一样的吧?二取一就可以了) Maybe the two companies in two different businesses are totally different in every aspects of management. Therefore,even if Rosa is very successful in Starlight, it cannot demonstrate he or she can also be successful in Fancy.

这里错误是抓到了,但是讲得太简单了,可以具体讲讲2个公司哪些方面不一样。比如说客户群体他们的消费习惯啦,市场份额的问题啦。可以稍微提及一下,你的反驳会更有力。

  In conclusion, the manager's recommendation is totally unconvincing with the logical flaws stated above. To strengthen the conclusion, the manager has to prove that the cause of our company's decreasing profits is exactly Pat Salvo. And more information and facts is needed to demonstrate that Rosa is the one who is responsible for the Starlight's increasing profits, not other factors. In addition, to convince to replace Pat with Rosa, there also must be verify that Rosa's experiences and skills can also success in our company(experiencesskills怎么success?何况success是名词) Thus, our company can be assured to have an increasing profit considerably.

主要逻辑问题抓出来了,结构比较清楚
缺点是文章模版化感觉比较严重。另外在语言的多样性上有待加强。特别是句式反面,用蓝色画出来4个句子,一模一样的句式,其实可以考虑多一些的表达方法,看范文或者改文的时候可以留意下别人的写法丰富自己。还有语句可以精炼下,有的地方表述比较重复
作者: tyarel    时间: 2010-7-19 21:04:50

TOPIC: ARGUMENT60 - The following appeared in the health section of Glenntown's local newspaper.

"Several national medical studies suggest that older people who have pets tend to enjoy better health than those who do not have pets: those who have pets have lower rates of high blood pressure and arthritis. It seems clear that having to care for an animal promotes good health for the older person. Therefore, Glenntown should establish a program to give a small pet such as a dog or cat to all of its citizens who are over the age of 65. This will help to insure that our senior citizens enjoy good health and have fewer medical bills."
WORDS: 430  

Based on the assumption that having pets benefits the older peoples' health due to the study about the relationship between it and rate of high blood pressure and arthritis, the author recommended a program of giving pets in order to insure  senior citizens' health and decrease medical bills. Yet, it isn't the case and because of the illogical inference the suggestion is also unwarranted.

Foremost, the advice depends on the studies about the older people having pets. However, the study just figured out the relationship between but failed to point out it is having pets that caused low rates of high blood pressure and arthritis. It's totally likely that this relationship is just a coincidence. If that is true other factors, like life style or exercises can conduce to the low rate. Thus, if the author cannot rule out the possibilities mentioned, the conclusion is weak obviously.

What's more, even though having pets do help decrease the incidence of blood pressure, no evidence have been offered to illustrate that it really do good to older people's health. All we can obtain from the studies is about the rate of having high blood pressure and arthritis. But what about other common senile diseases, such as heart trouble? Meanwhile it also cannot be denied that pets may lead to skin allergy, which is also not stated. Unfortunately, the author didn't provide any information about that. If having pets will indeed result in heavy skin allergy, how can we promote the program and bring this trouble to the old? So, only after eliminating any likelihood that having pets may do harm to older people can we consider the feasibility of the recommendation.

Last but not least, the author also committed a mistake when supply his advice. The studies are about older people while the program is about those over 65. How can he take it for granted that the objects in the studies are over 65? Maybe the result is correct only when people over 75? Or perhaps the research did not fit people over 85? And what if it is directly opposed to those from 65 to 75? Then it is not impossible that the program will not only be worthless to older people, but may be harmful to their health.

All in all, although the author cited several national medical studies, they accomplished very little towards supporting his suggestion, In my opinion, if more data and analysis about the benefits of having pets to older people's health can be provided, the argument will be strengthened and the advice can be accepted.


谢谢LX了~
作者: 追梦小木耳    时间: 2010-7-19 21:08:28

本帖最后由 追梦小木耳 于 2010-7-19 21:41 编辑

Based on the assumption that having pets benefits the older peoples' health due to the study about the relationship between it and rate of high blood pressure and arthritis, the author recommended a program of giving pets in order to insure  senior citizens' health and decrease medical bills. Yet, it isn't the case and because of the illogical inference the suggestion is also unwarranted.
不知道你看过草木斑竹的一个贴没有~反正我对这种复述题目的开头持保留态度~与其复述一遍,不如指出主要错误


Foremost, the advice depends on the studies about the older people having pets. However, the study just figured out the relationship between
who? but failed to point out it is having pets that caused low rates of high blood pressure and arthritis. It's totally likely that this relationship is just a coincidence.
可改为:the study unreasonably amounted the correlation between having pets and low rates of high blood pressure and arthritis to casual relationship If that is true other factors, like life style or exercises can conduce to the low rate. Thus, if the author cannot rule out the possibilities mentioned, the conclusion is weak obviously.


other possibilities的解释太少,做的假设也太少


What's more, even though having pets do help decrease the incidence of blood pressure, no evidence have been offered to illustrate that it really do good to older people's other aspects of health. All we can obtain from the studies is about the rate of having high blood pressure and arthritis. But what about other common senile diseases, such as heart trouble? Meanwhile it also cannot be denied that pets may lead to skin allergy, which is also not stated. Unfortunately, the author didn't provide any information about that.语义重复,可删去其中之一 If having pets will indeed result in heavy skin allergy, how can we promote the program and bring this trouble to the old? So, only after eliminating any likelihood that having pets may do harm to older people can we consider the feasibility of the recommendation.
这一段论述的很好很充分


Last but not least, the author also committed a mistake when supply his advice. The studies are about older people while the program is about those over 65. How can he take it for granted that the objects in the studies are over 65? Maybe the result is correct only when people over 75? Or perhaps the research did not fit people over 85? And what if it is directly opposed to those from 65 to 75? Then it is not impossible that the program will not only be worthless to older people, but may be harmful to their health.
这个攻击点很勉强~毕竟65也算老年人


All in all, although the author cited several national medical studies, they accomplished very little towards supporting his suggestion. In my opinion, if more data and analysis about the benefits of having pets to older people's health can be provided, the argument will be strengthened and the advice can be accepted.


总的来说,攻击点都找到了,逻辑也比较清晰。
第一个论点阐述的不够详细,再对你所做的假设解释的详细一点会更好更充实
语言不够流畅,还需要加强。
我说一下我的思路:
尽管根据题目养宠物可能有利于老年人的健康,但是由于调查没有说服力,根据此制定的让所有65岁以上的老年人养宠物是不合理的。
(1)
调查没有说服力。可能接受调查的人群仅限于某一个地方,可能不是养宠物使他们身体好。可能是其他原因,比如饮食健康、经常锻炼、遗传因素

(2)
养宠物可以避免高血压和关节炎,但可能有其他副作用。比如产生别的疾病。如皮肤病、过度劳累等

(3)
就算养宠物有助于健康,让所有老年人都养宠物是不合理的,忽略了老年人的个人情况。也许有的老年人不喜欢宠物,对宠物过敏,或者养不起宠物。

作者: 追梦小木耳    时间: 2010-7-19 21:09:39

TOPIC: ARGUMENT7 - The following appeared in a letter to the editor of the Clearview newspaper.

"In the next mayoral election, residents of Clearview should vote for Ann Green, who is a member of the Good Earth Coalition, rather than for Frank Braun, a member of the Clearview town council, because the current members are not protecting our environment. For example, during the past year the number of factories in Clearview has doubled, air pollution levels have increased, and the local hospital has treated 25 percent more patients with respiratory illnesses. If we elect Ann Green, the environmental problems in Clearview will certainly be solved."
WORDS: 358          TIME: 00:30:00          DATE: 2010-7-19 20:05:43

Final words 461 已修改黑体为后来补充

The argument is well presented, but the evidence provided is not sufficient to conclude the essential reason for air pollution, and simply electing a member of the Good Earht Coalition as the mayor may not effectively solve the problem.

In the first place, the main reason of the air pollution in the region is not clearly pointed out according to the argument. The arguer assumes it is the increasing number of factories in Clearview that has caused the air pollution. However, other possible alternatives are totally neglected. Perhaps, the factories built there do not produce polluting gas at all and it is the growing number of cars in the area that has led to increased air pollution. Also, the worse air condition may be caused by other nearby places where air pollution is exceptionally serious, which has affected the air condition of Clearview.

In addition, the arguer fails to demonstrate the air pollution level in Clearview. The rising number of patients with respiratory illnesses in hospital may not the result of local air pollution. Probably, a flu recently spread the area and most people have been infected, leading the 25 percent more patients. Also, it could be caused by a sudden weather change that a storm has attacked the whole area and a proportion of citizens caught a cold, getting worse and developing to serious respiratory illnesses.

Finally, whether Ann Green will be dedicated to solving air problem if she was elected mayor remains a doubt. Even if it was the factory that has caused the air pollution, would Ann Green, a member of Good Earth Coalition, close them? As a mayor, she must consider other matters of the city. Perhaps, closure of the factories would lead to economic problems of the region, making a number of employees be laid off. Considering such possible result, Ann Green may not close the factories simply for better air conditions. On the other hand, no evidence shows that Frank Braun would not be competent for the position. Although, he is not a member of green organizations, he may be better at balancing economy and environment problems as a member town council, who has more experience in politics. In addition, other capabilities should be taken into consideration when electing the mayor. Probably, there exists other problem that is more urgent than air pollution in the region, which calls for a capable mayor rather than a member of some environmental organizations.

To conclude, merely depending on the mayor to solve the environmental problem can hardly take effects. The local government should make more researches to find the air pollution level and its causes. Meanwhile other capabilities of the candidates must be considered in the election of mayor in order to the long-term development of the region.

作者: beanie加油    时间: 2010-7-19 22:34:00

本帖最后由 beanie加油 于 2010-7-21 16:09 编辑

The argument is well presented, but the evidence provided is not sufficient to conclude the essential reason for air pollution, and simply electing a member of the Good Earht Coalition as the mayor may not effectively solve the problem.

In the first place, the main reason of the air pollution in the region is not clearly pointed out according to the argument. The arguer assumes it is the increasing number of factories in Clearview that has caused the air pollution. However, other possible alternatives are totally neglected. Perhaps, the factories built there do not produce polluting gas at all and it is the growing number of cars in the area that has led to increased air pollution. Also, the worse air condition may be caused by other nearby places where air pollution is exceptionally serious, which has affected the air condition of Clearview这个观点很独特!.

In addition, the arguer fails to demonstrate the air pollution level in Clearview. The rising number of patients with respiratory illnesses in hospital may not the result of local air pollution. Probably, a flu recently spread the area and most people have been infected, leading the 25 percent more patients. Also, it could be caused by a sudden weather change that a storm has attacked the whole area and a proportion of citizens caught a cold, getting worse and developing to serious respiratory illnesses.

Finally, whether Ann Green will be dedicated to solving air problem if she was elected mayor remains a doubt. Even if it was the factory that has caused the air pollution, would Ann Green, a member of Good Earth Coalition, close them? As a mayor, she must consider other matters of the city. Perhaps, closure of the factories would lead to economic problems of the region, (causing)making a number of employees be laid off. Considering such possible result, Ann Green may not close the factories simply for better air conditions. On the other hand, no evidence shows that Frank Braun would not be competent for the position. Although, he is not a member of green organizations, he may be better at balancing economy and environment problems as a member town council, who has more experience in politics. In addition, other capabilities should be taken into consideration when electing the mayor. Probably, there exists other problem that is more urgent than air pollution in the region, which calls for a capable mayor rather than a member of some environmental organizations(organization,some当某个讲更好).

To conclude, merely depending on the mayor to solve the environmental problem can hardly take effects. The local government should make more researches to find the air pollution level and its causes. Meanwhile other capabilities of the candidates must be considered in the election of mayor in order to the long-term development of the region.

感觉你的逻辑很清晰,思维很连贯。陈述很到位,这点从整篇文章看下来深有体会。
语言方面也很多样化。你的思维和逻辑是我学习的榜样。继续加油!
作者: beanie加油    时间: 2010-7-19 22:42:07

TOPIC: ARGUMENT20 - The following appeared in a letter to the editor of the Balmer Island Gazette.

"The population of Balmer Island increases to 100,000 duing the summer months. To reduce the number of accidents involving mopeds and pedestrians, the town council of Balmer Island should limit the number of mopeds rented by each of the island's six moped and bicycle rental companies from 50 per day to 30 per day during the summer season. By limiting the number of rentals, the town council is sure to attain the 50 percent reduction in moped accidents that was achieved last year in the neighboring island of Torseau, when Torseau's town council enforced similar limits on moped rentals."
WORDS: 217 REVISED: 470        TIME: 00:30:00          DATE: 2010/7/19 4:41:20

    The article is well-presented, but not thoroughly well-reasoned. By adapting to the similar policy in the island of Torseau of limiting the number of moped rentals, the speaker assures that the town council can attain the 50 percent reduction in moped accidents. However, the argument seems to be compelling at first glance which actually masks many alternative counter examples.
     The speaker demonstrates that through limiting the number of mopeds rental by 20 per day, the number of accidents involving mopeds and pedestrians can be reduced to a half. But if the accident rate is 400 per day concerned with the huge population in Balmer Island, is it accessible to reach the 50 percent reduction by only limiting 20 moped rented? If so, it seems to the speaker that all incidences of accidents are heavily laid on driving mopeds, which is obviously not the case. In reference to the quality of the rented mopeds, maybe it suffers from many vital problems which are responsible for the accidents it causes. Thus, there is no assurance of the reduction in moped accidents. Other reasons contributing to the large number of accidents involve that people in the island may lack of a safety consciousness which directly leads to the high accidents rate, or a general traffic law has not been published yet to constrict pedestrian’s random behavior. All the factors can best explain the number of accidents there. Hence, not excluding the possible alternatives, the speaker can not presume the problems to be resolved.
    The situation in neighboring island of Torseau is probably quite different with that in Balmer Island. Perhaps the population in Torseau is small and hence the accidents rate is relatively low compared with that in Balmer. In addition, the residents there are well-educated to obey the traffic rules. Therefore, when the town council demands to reduce the number of moped rentals, people are more sensible to their duty and subsequently to avoid causing accidents.  Again, the geographical and terrain factors are also the causes of the number of accidents. Chances are that the roads in Torseau Island are very flat to be conducive for people's driving, whereas the Balmer Island is located in a hilly area, where its highways have to built on the hills. Many dangerous curves and sharp turns inevitably obstacle the reduction of accidents rate. Or probably the weather in Balmer is too bad to guarantee the safety of driving mopeds on the road, which is completely contrasted to the mild weather in Torseau. Thus, without ruling out these possible situations that may more or less influence the number of accidents, the speaker can not assume the Balmer Island to be able to achieve the same result of Torseau Island.
     Overall, the speaker tries to carry out the similar policy Torseau Island used to adapt to tackle with the accident rates. However, he neglects to consider the consequence of the seem-to-be reasonable policy due to many different factors. Before any final decisions are made about the reduction of the number of moped rentals, the town council should take a comprehensive view to evaluate possible alternatives and causes for the accidents.
作者: figuechen    时间: 2010-7-20 20:03:09

本帖最后由 figuechen 于 2010-7-20 22:04 编辑

TOPIC: ARGUMENT20 - The following appeared in a letter to the editor of the Balmer Island Gazette.

"The population of Balmer Island increases to 100,000 duing the summer months. To reduce the number of accidents involving mopeds and pedestrians, the town council of Balmer Island should limit the number of mopeds rented by each of the island's six moped and bicycle rental companies from 50 per day to 30 per day during the summer season. By limiting the number of rentals, the town council is sure to attain the 50 percent reduction in moped accidents that was achieved last year in the neighboring island of Torseau, when Torseau's town council enforced similar limits on moped rentals."
WORDS: 217 REVISED: 470        TIME: 00:30:00          DATE: 2010/7/19 4:41:20

    The article is well-presented, but not thoroughly well-reasoned. By adapting to the similar policy in the island of Torseau of limiting the number of moped rentals, the speaker assures that the town council can attain the 50 percent reduction in moped accidents. However, the argument seems to be compelling at first glance which actually masks many alternative counter examples.

[开头很不错,开门见山。语句也很不错,学习了]

     The speaker demonstrates that through limiting the number of mopeds rental by 20 per day, the number of accidents involving mopeds and pedestrians can be reduced to a half. But if the accident rate is 400 per day concerned with the huge population in Balmer Island, is it accessible to reach the 50 percent reduction by only limiting 20 moped rented? [
恩?不是每家公司减少20么?]If so, it seems to the speaker that all incidences of accidents are heavily laid on driving mopeds, which is obviously not the case. [感觉这里有点啰嗦,为了讲清这个道理费了不少劲]In reference to the quality of the rented mopeds, maybe it suffers from many vital problems which are responsible for the accidents it causes. Thus, there is no assurance of the reduction in moped accidents. Other reasons contributing to the large number of accidents involve that people in the island may lack of [不用of了,lack就行]a safety consciousness which directly leads to the high accidents rate, or a general traffic law has not been published yet to constrict pedestrian’s random behavior. All the factors can best explain the number of accidents there. Hence, not excluding the possible alternatives, the speaker can not presume the problems to be resolved.
[这里作者说的是该措施不一定能降低事故率。个人感觉这个错误可以放到最后讲,因为原题中是用了一个类比来说明事故率能降低,那么就应该先反驳这个类比,再说明这点。另外第一点的反例有些勉强,我觉得可以这样说:六家公司只是占了moped出租市场的少数,禁了他们也不能完全禁了moped出租,更不能导致事故率下降。可能我举的这个说法反驳点稍有不同,仅供参考啦]

    The situation in neighboring island of Torseau is probably quite different with that in Balmer Island. [
光这句感觉不够,没点到点上。可以再补一句,因为不同,所以相同的措施在B岛未必有用。]Perhaps the population in Torseau is small and hence the accidents rate is relatively low compared with that in Balmer. In addition, the residents there are well-educated to obey the traffic rules. Therefore, when the town council demands to reduce the number of moped rentals, people are more sensible to their duty and subsequently to avoid causing accidents.  Again, the geographical and terrain factors are also the causes of the number of accidents. Chances are that the roads in Torseau Island are very flat to be conducive for people's driving, whereas the Balmer Island is located in a hilly area, where its highways have to built on the hills. Many dangerous curves and sharp turns inevitably obstacle the reduction of accidents rate. Or probably the weather in Balmer is too bad to guarantee the safety of driving mopeds on the road, which is completely contrasted to the mild weather in Torseau. Thus, without ruling out these possible situations that may more or less influence the number of accidents, the speaker can not assume the Balmer Island to be able to achieve the same result of Torseau Island.
[反驳这个faulty analogy。非常非常到位,例子很详实了。]

     Overall, the speaker tries to carry out the similar policy Torseau Island used to adapt to tackle with the accident rates. However, he neglects to consider the consequence of the seem-to-be reasonable policy due to many different factors. Before any final decisions are made about the reduction of the number of moped rentals, the town council should take a comprehensive view to evaluate possible alternatives and causes for the accidents.


[综述]
总的来说,作者分段清晰,开头和结尾很简洁,没有废话。中间陈述的内容逻辑很清晰,例子也很恰当。

但我总觉得两个反驳点少了一点。特别是作者的两个反驳着眼点有些雷同。其实这篇Argument还有许多从其它角度考虑的反驳点,比如BI在其它月份中的人口很少,没有必要;或者是该措施可能导致负面影响,比如游客出行不便。如果啰嗦一点的话还可以攻击说:限制出租数量未必是邻岛事故率下降的因素。虽然我最近也试图把类似的攻击点归纳到一起,但我还是感觉每一段就攻击一个小点,这样思路和逻辑上都会比较清晰。

一点小意见,仅供参考~
作者: figuechen    时间: 2010-7-20 20:04:06

TOPIC: ARGUMENT40 - The following appeared in a memorandum from the president of Excello Food Markets.

"In 90 towns where Excello has food markets, natural-food stores specializing in organic food products-products containing no chemical preservatives and made with foods grown without pesticides-have opened nearby as competitors. Surveys of our own customers reveal a growing concern about foods grown using pesticides or preserved with chemicals. Recently our market in Sun City participated in a local food tasting fair, and 75 percent of the fair goers who visited the Excello booth requested free samples of organic fruit. Such evidence indicates that to increase our profits, we should begin to stock a full line of organic food products in all our markets."

WORDS: 596          TIME: 00:30:00          DATE: 2010-7-19 20:20:18

    The president suggests that the company should start to stock organic food products in the markets in order to increase the profits. Several reasons are listed to support his claim. However, there are many flaws in the assumption and therefore the argument is unconvincing.
    First of all, the author unfairly concludes that there is a growing trend to buy organic foods for consumers. The author quotes the phenomenon on the food tasting fair, which cannot bolster his claim. Firstly, people who asked for organic fruit might not be interested in buying those foods Perhaps participants requested the fruit simply because the fruits are free, and they want something to eat at that time, while they may not incline to buy the fruit in the result of price, taste, etc. Besides, admitted that there is the need in Sun City, we cannot infer from the argument that there exists the same need all over the country. Perhaps residents in other towns show little interest in organic fruits, and a stock of organic foods in all the markets will be a waste of time and funds. The author too hastily infers that there is a growing need for organic fruit.
    Even assuming that there lies the interest on organic food for customers, the author fails to provide evidence to show that consumers will buy these fruits. There are many factors that determine whether a product will be welcome, which include taste, price, quality, and safety. Perhaps the quality and taste of organic fruit is excellent while the price of it is too high for most families to accept, and therefore the market of organic fruit is rather small; or perhaps the taste and price of the organic fruit is satisfactory while the safety of the fruit remains doubting, and therefore many people will not dare to buy organic foods, leading to little potential for organic fruits. Unless the author provides evidence to eliminate these possibilities, he cannot convince me that there will be a large number of consumers who will have a try on these fruits.
    Finally, even if we assume that many consumers tend to buy organic foods instead of inorganic ones, the author cannot ensure that his measures will bring about the increase of profits. He does not provide information about the profits of organic and inorganic foods, and therefore we do not know whether organic foods are more profitable or not. Perhaps inorganic foods are ten times more profitable than organic ones, and thus they should not turn to organic ones unless there is a much stronger need for organic foods. In addition, he fails to consider other factors that have an influence on the profits. Perhaps it will take much more money to raise these foods, and the company will pay more to the farmers, which might cause a decline in the profits; or perhaps it is easier for organic foods to go bad and therefore it will cost the company more to keep them fresh, resulting in economic losses. Without ruling out these cases, the author cannot confidently conclude that stocking a full line of organic foods will raise their profits.
    In sum, due to the flaws listed above, the claim that Excello Food Markets should initiate to stock organic food products in all their markets so as to increase their profits. To better support his claim, the author had better provide further evidence, such as a more persuasive survey results indicating a strong need for organic fruits and a marketing survey to illustrate that a stock of organic food products will make money.

谢谢楼下了
作者: 追梦小木耳    时间: 2010-7-20 20:42:57

本帖最后由 追梦小木耳 于 2010-7-21 23:41 编辑

不好意思,改晚了

The president suggests that the company should start to stock organic food products in the markets in order to increase the profits. Several reasons are listed to support his claim. However, there are many flaws in the assumption and therefore the argument is unconvincing.
开头不错,简洁,点明要害
    First of all, the author unfairly concludes that there is a growing trend to buy organic foods for consumers. The author quotes the phenomenon on the food tasting fair, which cannot bolster his claim. Firstly, people who asked for organic fruit might not be interested in buying those foods Perhaps participants requested the fruit simply because the fruits are free, and they want something to eat at that time, while they may not incline to buy the fruit in the result of
这个短语啥意思,没查到 price, taste, etc. Besides, admitted that there is the need in Sun City, we cannot infer from the argument that there exists the same need all over the country. Perhaps residents in other towns show little interest in organic fruits, and a stock of organic foods in all the markets will be a waste of time and funds. The author too hastily infers that there is a growing need for organic fruit.
    Even assuming that there lies the interest on organic food for customers, the author fails to provide evidence to show that consumers will buy these fruits. There are many factors that determine whether a product will be
welcome popular, which includeincluding tast
e, price, quality, and safety. Perhaps the quality and taste of organic fruit is excellent while the price of it is too high for most families to accept, and therefore the market of organic fruit is rather small; or perhaps the taste and price of the organic fruit is are
satisfactory while the safety of the fruit remains doubting doubtable, and therefore many people will not dare to buy organic foods, leading to little potential for organic fruits. Unless the author provides evidence to eliminate these possibilities, he cannot convince me that there will be a large number of consumers who will intend to
have a try on these fruits.
    Finally, even if we assume that many consumers tend to buy organic foods instead of inorganic ones, the author cannot ensure that his measures will bring about the increase of profits.
Since He does not provide information about the profits of organic and inorganic foods, and therefore we do not know whether organic foods are more profitable or not. Perhaps inorganic foods are ten times more profitable than organic ones, and thus they should not turn to organic ones unless there is a much stronger need for organic foods. In addition, he fails to consider other factors that have an influence on the profits. Perhaps it will take much more money to raise these foods, and the company will pay more to the farmers, which might cause a decline in the profits; or perhaps it is easier for organic foods to go bad organic foods are easier to go bad and therefore it will cost take the company more to keep them fresh, resulting in economic losses. As a result,
Without ruling out these cases, the author cannot confidently conclude that stocking a full line of organic foods will raise their profits.
    In sum, due to the flaws listed above, t
he claim claim
后缺少谓语和宾语 that Excello Food Markets should initiate to stock organic food products in all their markets so as to increase their profits is unreasonable. To better support his claim, the author had better provide further evidence, such as a more persuasive survey results indicating a strong need for organic fruits and a marketing survey to illustrate that a stock of organic food products will make money.

主要攻击点都说出来了,逻辑无可挑剔。
有些链接词我觉得不太合适,就改了下,可以参考

我写的提纲,和你的角度不太一样,但是内容差不多,供参考
所提供的证据不足以说明有机食物受到居民的欢迎,因此在市场中建立有机食物销售可能不能带来利润
1)调查没有可信度,不能证明有机产品会受欢迎。调查人群不可靠,也许在回答问题的时候受到了引导。担心不表示就会购买有机产品,可能会考虑到有机产品也有坏处
2)去参观要样品不能说明他们也愿意买。也许尝了以后觉得不好吃。只要了有机水果的样品,可能只对水果感兴趣,对其他有机产品不感兴趣。同时价钱也不知道,可能会认为太贵不买
3)即使人们对有机食品感兴趣,储备有机食物也不一定能赚钱。储备会很快坏掉。不知道哪种有机产品最受欢迎



作者: 追梦小木耳    时间: 2010-7-20 20:44:56

大家限时都写到500+了,我才刚300多~
补充完还不到500,还需多练!先感谢楼下指导
TOPIC: ARGUMENT233 - The following appeared in a memo from the vice president of a company that builds shopping malls throughout the country.

"The surface of a section of Route 101, paved two years ago by McAdam Road Builders, is now badly cracked and marred by dangerous potholes. In another part of the state, a section of Route 66, paved by Appian Roadways more than four years ago, is still in good condition. Appian Roadways has recently purchased state-of-the-art paving machinery, and it has hired a new quality-control manager. Because of its superior work and commitment to quality, we should contract with Appian Roadways rather than McAdam Road Builders to construct the access roads for all our new shopping malls."
WORDS: 325  final 498        TIME: 00:30:00          DATE: 2010-7-20 11:27:26

The argument is well presented, but only by comparing the present conditions of the two roads that were constructed separately by the two companies can not convince me that the Appian Roadways is superior to McAdam Road Builders, and therefore should construct the access roads project.

In the first place, the comparison of the present conditions of the two roads is unreasonable, which has failed to consider other possibilities such as the amount of cars travelling on the roads and the climates of the two regions. As the amounts of cars on two roads are not provided, it is entirely possible that there has been larger number of cars running on Route 101during the last two years, causing the cracks and dangerous potholes. While there could have been much fewer cars on Route 66, which may be built in a distant area, and therefore could have kept better condition. Besides, different climates could also affect the road conditions. Perhaps, the place in which Route 101 was built rains a lot every year and the road suffers flood frequently, triggering severe damages. While climate in another part of the state, in which Route 66 was constructed, is exceptionally mild and has little effects on the road conditions. Thus, it seems imprudent to argue that it is the companies' responsibility for the conditions of the roads without accounting for other possible alternatives.

Furthermore, Appian Roadways' new purchase of machinery and the new manager could not demonstrate its superiority as well. Perhaps, the company has received lots of complains about their constructions before, and have to purchase new paving machinery in order to upgrade the quality of their works. Or probably the employees of the company did not work efficiently before and the company has to hire new manager to improve the working efficiency. On the other hand, the facilities of MRB are not mentioned in the argument. Does it have more advanced machinery? Do the workers in the company work more efficiently? If so, the option of AR is unreasonable.

Even if Appian Roadways does have superior work and commitment to quality, no information indicates that it could be competent for the construction of shopping mall’s access roads. The argument only shows its superior construction of state routes but fails to prove that it has any experience in building other types of roads. As it is known, the usage of access roads differs totally from that of state routes. Cars travelling on access roads would be in much lower speed and the roads would own more pedestrians who may require comfortable sidewalks and fabulous scenes. It remains a doubt whether the AR could satisfy these needs.

To conclude, it is not sound to claim that AR is superior to MRB based on the unreliable comparison. To make the argument forceful, more achievements of the two companies should be provided in order to help figure out which one is competent to the construction of the access roads of the shopping malls.
作者: pocketlion    时间: 2010-7-20 21:06:24

本帖最后由 pocketlion 于 2010-7-21 22:11 编辑

占71楼~~


改71楼:

TOPIC: ARGUMENT233 - The following appeared in a memo from the vice president of a company that builds shopping malls throughout the country.


"The surface of a section of Route 101, paved two years ago by McAdam Road Builders, is now badly cracked and marred by dangerous potholes. In another part of the state, a section of Route 66, paved by Appian Roadways more than four years ago, is still in good condition. Appian Roadways has recently purchased state-of-the-art paving machinery, and it has hired a new quality-control manager. Because of its superior work and commitment to quality, we should contract with Appian Roadways rather than McAdam Road Builders to construct the access roads for all our new shopping malls."
WORDS: 325  final 498        TIME: 00:30:00          DATE: 2010-7-20 11:27:26

The argument is well presented, but only by comparing the present conditions of the two roads that were constructed separately by the two companies can not convince me that the Appian Roadways is superior to McAdam Road Builders, and therefore should construct the access roads project.
(去掉project,感觉construct
the .....project
搭配不当)


In the first place, the comparison of the present conditions of the two roads is unreasonable, which has failed to consider other possibilities
(改为aspects such as the amount of cars travelling on the roads and the climates of the two regions. As the amounts of cars on two roads are not provided, it is entirely possible that1 there has been larger number of cars running on Route 101during the last two years, causing(and causes貌似好些) the cracks and dangerous potholes. While there could have been much fewer cars on Route 66,(与前面(1)重复) which may be built in a distant area, and therefore could have kept better condition. Besides, different climates could also affect the road conditions+and leads to the difference between the conditions of the two roads强调导致的不同结果). Perhaps, the place in which Route 101 was built rains a lot every year and the road suffers flood frequently, triggering severe damages. While climate in another part of the state, in which Route 66 was constructed, is exceptionally mild and has little effects on the road conditions. Thus, it seems imprudent to argue that it is the companies' responsibility for the conditions of the roads without accounting for other possible alternatives.

Furthermore, Appian Roadways' new purchase of machinery and the new manager could not demonstrate its superiority as well. Perhaps, the company has received lots of complains
complaints about their constructions before, and have to purchase new paving machinery in order to upgrade the quality of their works. Or probably the employees of the company did not work efficiently before and the company has to hire new manager to improve the working efficiency. On the other hand, the facilities of MRB are not mentioned in the argument. Does it have more advanced machinery? Do the workers in the company work more efficiently? If so, the option of AR is unreasonable.

Even if Appian Roadways does have superior work and commitment to quality, no information indicates that it could be competent for the construction of shopping mall’s access roads. The argument only shows its superior construction of state routes but fails to prove that it has any experience in building other types of roads. As it is known, the usage of access roads differs totally from that of state routes. Cars travelling on access roads would be in much lower speed and the roads would own more pedestrians who may require comfortable sidewalks and fabulous scenes. It remains a doubt whether the AR could satisfy these needs.

(还可以稍加提下以下谬误:1.No information about the performance of the new quality-control manager at Appian is provided.2. Other competent companies might be available for the president to choose from.

To conclude, it is not sound to claim that AR is superior to MRB based on the unreliable comparison. To make the argument forceful, more achievements of the two companies should be provided in order to help figure out which one is competent to the construction of the access roads of the shopping malls.


文章谬误找的较全,论证有力,行文流畅,表达清晰,已经很不错了,至于练习时字数较少,只要稍加练习,形成自己的写作套路和习惯,速度很快会提上来的,作者加油。
作者: pocketlion    时间: 2010-7-20 21:09:42

本帖最后由 pocketlion 于 2010-7-20 21:18 编辑

TOPIC: ARGUMENT82 - The following appeared in a letter to an editor.

"In many countries, wood is the primary fuel used for heating and cooking, but wood smoke can cause respiratory and eye problems, and extensive use of wood causes deforestation, a major environmental problem. In contrast, charcoal, made by partially burning wood in a controlled process, is a fuel that creates less smoke than wood does. Moreover, although charcoal costs slightly more than wood, less charcoal is needed to produce the same amount of heat. Therefore, people who use wood as their primary fuel can, without experiencing economic hardship, switch to charcoal and can thereby improve their health and preserve the environment."

In the letter, the author concludes that people who use wood as their primary fuel can improve their health and preserve the environment without experiencing economic hardship by switching to charcoal. To support it, the author points out that the charcoal creates less smoke than the wood. Meanwhile, the author reasons that the charcoal is more efficient in producing heat than wood. However, the argument suffers from various flaws and therefore it is unwarranted.

To begin with, as no detailed information about the process in which the wood is made into charcoal is provided, we cannot ensure that using charcoal will be more friendly to the environment. Although the charcoal creates less smoke than the wood, the production of charcoal may be harmful to the environment and causes worse environmental problems than using the wood does. If this is the case, the author cannot make any general conclusion that using charcoal can help preserve the environment.

Besides, the author fails to inform us about the cost of producing the charcoal and we do not know how much does charcoal cost more than wood. It is highly possible that it will cost much money to make the wood into charcoal. If so, not all the countries can afford to take the measurement and people who use charcoal as their primary fuel may experience economic hardship if the cost of using the charcoal is high. Therefore, without ruling out the possibility above, the author cannot come to the conclusion that switching to charcoal can improve their health and preserve the environment without experiencing economic hardship.

In addition, the author fails to inform us how much wood will be used to make the charcoal which can produce the same amount of heat. The deforestation will be accelerated if the production of charcoal need more wood to get the charcoal which can produce the same amount of heat. Therefore, we cannot ensure that using charcoal can help preserve the environment.

Finally, only switching to charcoal may not be sufficient to improve the people's health and preserve the environment. The efficiency of the using of wood should be improved and the other measurements such as planting more trees should be taken to protect the environment and improve people's health. Besides, other fuels which may be better than charcoal are ignored by the author. Moreover, not all people may be willing to make the change of using fuel.

In sum, the argument is logically flawed and unconvincing as it stands. To strengthen the conclusion, the author should provide more persuasive evidence about the advantages of using charcoal. Besides, the author should rule out the possibility that people cannot afford to make the wood into charcoal .To better assess the conclusion, we should know more information about the cost of producing charcoal and the influence on the environment when using charcoal instead of wood.


请使劲拍之~~~
作者: tyarel    时间: 2010-7-22 12:41:24

本帖最后由 tyarel 于 2010-7-22 23:23 编辑

占楼73

TOPIC: ARGUMENT82 - The following appeared in a letter to an editor.

"In many countries, wood is the primary fuel used for heating and cooking, but wood smoke can cause respiratory and eye problems, and extensive use of wood causes deforestation, a major environmental problem. In contrast, charcoal, made by partially burning wood in a controlled process, is a fuel that creates less smoke than wood does. Moreover, although charcoal costs slightly more than wood, less charcoal is needed to produce the same amount of heat. Therefore, people who use wood as their primary fuel can, without experiencing economic hardship, switch to charcoal and can thereby improve their health and preserve the environment."

In the letter, the author concludes that people who use wood as their primary fuel can improve their health and preserve the environment without experiencing economic hardship by switching to charcoal. To support it, the author points out that the charcoal creates less smoke than the wood. Meanwhile, the author reasons that the charcoal is more efficient in producing heat than wood. However, the argument suffers from various flaws and therefore it is unwarranted.
对首段的一点小建议。模版化的痕迹太重了。现在我个人觉得首段可以有2种处理方法:淡化原文内容,着重指出逻辑错误以引导下文;或者不提及错误,但是清楚复述出原文推理的逻辑链。关键就是要从首段就能读出以下信息文章的逻辑链我读懂了,接下来我要从哪几个逻辑上反驳。否则感觉这个首段就完全没起到作用了。当然这个我自己也在练习。。。


To begin with, as no detailed information about the process in which the wood is made into charcoal is provided, we cannot ensure that using charcoal will be more friendly(friendlier) to the environment. Although the charcoal creates less smoke than the wood, the production of charcoal may be harmful to the environment and causes worse environmental problems than using the wood does. If this is the case, the author cannot make any general conclusion that using charcoal can help preserve the environment.
这点反驳的太简单了。不够深入,为什么木炭可能对环境不好?木炭可能引起什么问题?这一段几乎没有什么实质性的反驳内容。比如说,制作木炭同样要花费木材,那是不是提供同等热量花费的木炭,转化为原材料花费的木材更多?制作木炭的时候要通过不充分燃烧,会产生CO等气体是不是对环境更不利?这些都可以提及

Besides, the author fails to inform us about the cost of producing the charcoal and we do not know how much does charcoal cost more than wood. It is highly possible that it will cost much money to make the wood into charcoal. If so, not all the countries can afford to take the measurement and people who use charcoal as their primary fuel may experience economic hardship if the cost of using the charcoal is high. (
表达上有点重复了,可省) Therefore, without ruling out the possibility above, the author cannot come to the conclusion that switching to charcoal can improve their health and preserve the environment without experiencing economic hardship.(提高健康和保护环境不是重点,因此结论句这么写不合适,应该突出的是经济方面,所以就不要提及healthenvironment)

In addition, the author fails to inform us how much wood will be used to make the charcoal which can produce the same amount of heat. The deforestation will be accelerated if the production of charcoal need more wood to get the charcoal which can produce the same amount of heat. Therefore, we cannot ensure that using charcoal can help preserve the environment.
完全可以并入第一段,作为质疑保护环境的一个方面。

Finally, only switching to charcoal may not be sufficient to improve the people's health and preserve the environment. The efficiency of the using of wood should be improved and the other measurements such as planting more trees should be taken to protect the environment and improve people's health. Besides, other fuels which may be better than charcoal are ignored by the author. Moreover, not all people may be willing to make the change of using fuel.
反驳得有点牵强也散乱,感觉没有核心的说理来串出这段的观点,只是堆了些可能性再这里。其实我觉得这个反驳点就在于换木炭的可行性。1.作者没有证明除了这个办法就没有更好的办法了,比如就像换别的fuel,gas2.作者没有考虑别的可能的困难。比如说谁来制作木炭?还有意愿的问题?木炭的供应是否跟得上或者说是否充足?

In sum, the argument is logically flawed and unconvincing as it stands. To strengthen the conclusion, the author should provide more persuasive evidence about the advantages of using charcoal. Besides, the author should rule out the possibility that people cannot afford to make the wood into charcoal.
 To better assess the conclusion, we should know more information about the cost of producing charcoal and the influence on the environment when using charcoal instead of wood.
最后一段的3句话可以在句式上再推敲一下,做点变化,the author should; the author should; we should显得比较单调,也可以索性句子合并合并


主要逻辑问题抓出来了,文章语句词汇上没有什么大错
主要问题是,模板化感觉过重;反驳不够深入,对于他因分析例举不够导致文章感觉没有什么说服力,同时也使得文章显得没有内容;结构上稍有问题,2,4两段针对的是同一个逻辑错误,应该并在一起论述;最后就是语言上变化性不足,句式显得有点平淡

以上就是个人想法,希望对你有用
作者: tyarel    时间: 2010-7-22 12:43:46

TOPIC: ARGUMENT185 - The following appeared in a letter from the owner of the Sunnyside Towers apartment building to its manager.

"One month ago, all the showerheads on the first five floors of Sunnyside Towers were modified to restrict the water flow to approximately 1/3 of its original force. Although actual readings of water usage before and after the adjustment are not yet available, the change will obviously result in a considerable savings for Sunnyside Corporation, since the corporation must pay for water each month. Except for a few complaints about low water pressure, no problems with showers have been reported since the adjustment. Clearly, restricting water flow throughout all the 20 floors of Sunnyside Towers will increase our profits further."
WORDS: 325 -- 532         TIME: 00:30:00          DATE: 2010/7/22 11:46:03

Associating the modification implemented and the fact that Sunnyside Corporation (SC), the author assumed that the restraint would result in a considerable savings and then on the assumption that no other big problems would be brought besides some complains, he recommended that restricting water flow would be spread to all the 20 floors. However, that is not the case and since he made a wrong inference on the result restricting flow may cause, didn't consider the complains would damage the firm and the unexpected reactions of other residents to the plan, the suggestion is not so tempting as it appeared at the first glance.

Foremost, the author made an experiment on the first floors of Sunnyside Towers (ST) to make sure the influence the adjustment may bring to residents and SC. But it is too hasty to conclude that the plan will certainly help SC decrease costs. On one hand, since no readings of water usage is obtained no evidence illustrating that the usage may be remained. It is highly possible that the restriction will force dwellers find some methods to meet their need of water, like increase the time of using water. On the other hand, the price of water is also not mentioned. Maybe the fluctuation of price of water per unit is the true reason to the decreasing costs. Without ruling out these possibilities, the author cannot persuade me that the modification result in a considerable savings to SC.

What's more, the author admitted that some bad impressions will be left on the consumers of SC because of the adjustment. People will complain about the low water pressure and the author failed to figure out the measures to handle this problem. It is likely that residents may consider changing the company providing their water or maybe this incident will also worse others’ impression on our company, which may decrease our potential consumers and our profits at the same time. And then the bad image will be spread further and cause more lost of consumers. Without finding a method to rebuild our image and tackle the chain reaction, it is not wise to adopt this plan at the risk of losing consumers.

Last but not least, the owner also didn't take the difference between first 5 floors and all the 20 floors into consideration. Even if the restraint and the impact did not incur the dislike of residents on first 5 floors, it can be necessarily that other inhabitants will not deprecate that since decreasing amount of water may be more obvious to them and conduce to more inconvenience to their livesA, which can lead to the same possible results mentioned above and may bring more damage to SC. Unless we can make sure that all people in ST will not reject the adjustment, the modification cannot be casually carried out.

In conclusion, to strengthen the argument and make the recommendation more adoptable, the owner must provide that the plan can truly help SC save money and offer enough methods to deal with the problems caused as well. Besides, he also need to do the survey to ensure that it can be accepted by residents on all 20 floors. Thus, the project may be more reasonable and feasible.

有个小小疑问,我写完后想想,会不会最后一段反驳论证比较单薄?另外关于except complains, no problems have been shown也可以进行反驳?如果反驳的话单独一段好还是和complain那段放在一起好?

希望lx改的时候能给我点意见,谢谢
作者: figuechen    时间: 2010-7-24 17:26:19

本帖最后由 figuechen 于 2010-7-25 17:25 编辑

改75

TOPIC: ARGUMENT185 - The following appeared in a letter from the owner of the Sunnyside Towers apartment building to its manager.

"One month ago, all the showerheads on the first five floors of Sunnyside Towers were modified to restrict the water flow to approximately 1/3 of its original force. Although actual readings of water usage before and after the adjustment are not yet available, the change will obviously result in a considerable savings for Sunnyside Corporation, since the corporation must pay for water each month. Except for a few complaints about low water pressure, no problems with showers have been reported since the adjustment. Clearly, restricting water flow throughout all the 20 floors of Sunnyside Towers will increase our profits further."
WORDS: 325 -- 532         TIME: 00:30:00          DATE: 2010/7/22 11:46:03

Associating the modification implemented and the fact that Sunnyside Corporation (SC), the author assumed that the restraint would result in a considerable savings and then on the assumption that no other big problems would be brought besides some complains, he recommended that restricting water flow would be spread to all the 20 floors. However, that is not the case and since he made a wrong inference on the result restricting flow may cause, didn't consider the
complains [complaints?] would damage the firm and the unexpected reactions of other residents to the plan, the suggestion is not so tempting as it appeared at the first glance.

[开头先简要概括了题目,然后点出了三个要攻击的点。]

Foremost, the author made an experiment on the first floors of Sunnyside Towers (ST) to make sure the influence the adjustment may bring to residents and SC. But it is too hasty to conclude that the plan will certainly help SC decrease costs. On one hand, since no readings of water usage is obtained no evidence illustrating that the usage may be remained. It is highly possible that the restriction will force dwellers find some methods to meet their need of water, like increase the time of using water. On the other hand, the price of water is also not mentioned. Maybe the fluctuation of price of water per unit is the true reason to the decreasing costs. Without ruling out these possibilities, the author cannot persuade me that the modification result in a considerable savings to SC.

[这段反驳该措施会节省花费的做法,一方面可能流量不变,另一方面可能价格会变。]

What's more, the author admitted that some bad impressions will be left on the consumers of SC because of the adjustment. People will complain about the low water pressure and the author failed to figure out the measures to handle this problem. It is likely that residents may consider changing the company providing their water or maybe
this incident will also worse [
这里有语法错误,谓语动词呢?]others’ impression on our company, which may decrease our potential consumers and our profits at the same time. And then the bad image will be spread further and cause more lost of consumers. Without finding a method to rebuild our image and tackle the chain reaction, it is not wise to adopt this plan at the risk of losing consumers.
[这段说该公司没有应对投诉的办法。个人认为这段的攻击性不是太强,可以考虑说没有投诉不等于没有问题。毕竟原文已经说了投诉很少,再说投诉会造成问题好像有点不太合乎常理。]

Last but not least, the owner also didn't take the difference between first 5 floors and all the 20 floors into consideration. Even if the restraint and the impact did not incur the dislike of residents on first 5 floors, it can be necessarily that other inhabitants will not deprecate that since decreasing amount of water may be more obvious to them and conduce to more inconvenience to their lives, which can lead to the same possible results mentioned above and may bring more damage to SC. Unless we can make sure that all people in ST will not reject the adjustment, the modification cannot be casually carried out.

[这段说5层能实施的未必20层能实施,主要写了居民会反对。我觉得这个理由不是太好,因为刚才已经说过投诉和居民反对的问题了。可以考虑说20层太高了,减少那么多水压水就打不上去了。]

In conclusion, to strengthen the argument and make the recommendation more adoptable, the owner must provide that the plan can truly help SC save money and offer enough methods to deal with the problems caused as well. Besides, he also
need
[needs] to do the survey to ensure that it can be accepted by residents on all 20 floors. Thus, the project may be more reasonable and feasible.

全文的批驳点基本都到位了,语言可能还需要进一步地锤炼。在批驳错误的时候,反例有时不够有力,如果能在这方面加强的话可能会更好。

另外no complaint的问题可以批驳,在本文中应该还是一个比较重要的批驳点,可以考虑代替原文中的批驳点,毕竟前者更有力一些。
作者: figuechen    时间: 2010-7-24 17:26:50

TOPIC: ARGUMENT106 - The following editorial appeared in a newspaper in the country of Solaria.

"The Eliot Valley region was primarily agricultural twenty years ago. In the past twenty years, however, many computer-chip manufacturers have opened factories there. A recent study found that water pollution in the region was worse than in any other region in the country. Moreover, the computer-chip factories, which use large quantities of water to manufacture the chips, are probably responsible for the low levels of water in the region's lakes and reservoirs. Therefore, if the region's computer-chip makers had limits placed on the amount of water they could use, water quality would improve."
WORDS: 532          TIME: 00:30:00          DATE: 2010-7-24 17:22:16

    The author suggested in the editorial that computer-chip makers in the region should have limits on the amount of water they could use in order to improve the water quality. The argument which seems to be compelling at first glance actually masks many alternative counter examples and therefore unconvincing it stands.
    Initially, the author might be assigning a false clause to the water pollution. The author fails to provide evidence to indicate that computer-chip manufacturers are indeed the reason for water pollution. There might be factors other than computer-chip makers that lead to the serious pollution. For instance, some oil companies might have established their branch factories in Eliot Valley ten years ago and brought huge pollution to the water system there; or perhaps the environmental situation in the whole country was worse than twenty years ago. Eliot Valley might be doing better than most of the regions in the country, which illustrates that computer-chip companies might not cause any pollution. Without further evidence to prove that it is computer-chip manufacturers that result in the water pollution, the author cannot confidently draw the claim that the water pollution was owing to the computer-chip manufacturers.
    In addition, it is presumptuous for the author to judge that computer-chip makers are attributable to the low levels of water in the region according to the mere fact that these factories consume large amount of water. Firstly, the computer-chip factories might not use the water in the region. It is possible that they are using water from other regions. Moreover, other reasons might also explain the changes of water levels. Perhaps the climate this year is unusual and brings about the low levels of water; or perhaps over-consumption of underground water is the true reason for low levels of water. Unless the author provide more evidence to eliminate these possibilities, otherwise it is hard for me to accept the claim that low levels of water is due to the water consumption of computer-chip factories.
    What's more, the claim that the water quality would rise if the restrictions were taken rests on the unsubstantiated assumption and thereby is open to doubt. One point is that the limits against the quantity of water used might not have relationship with the quality of the water. Perhaps those factories do not release the polluted water into the river. The second point is that common sense tells me that it is easier to pollute water than to recover it. The water quality would not improve unless certain measures designed to eliminate the pollutants in the river are taken. As long as the water is continuously used, the quality of the water will get only worse. Without taking these possibilities into consideration, the author cannot convince me that the measures taken will surely improve the water quality.
    In sum, there is nowhere more ridiculous than the advice that the limits against water usage of computer-chip makers should be adopted to improve the water quality. To better support his claim, the author has to provide extra evidence to make sure that computer-chip makers are the decisive factors in determining the level and quality of water, together with the evidence that the restrictions will work.

恳请楼下狠拍,先谢~
作者: tyarel    时间: 2010-7-26 12:22:55

本帖最后由 tyarel 于 2010-7-26 21:49 编辑

TOPIC: ARGUMENT106 - The following editorial appeared in a newspaper in the country of Solaria.

"The Eliot Valley region was primarily agricultural twenty years ago. In the past twenty years, however, many computer-chip manufacturers have opened factories there. A recent study found that water pollution in the region was worse than in any other region in the country. Moreover, the computer-chip factories, which use large quantities of water to manufacture the chips, are probably responsible for the low levels of water in the region's lakes and reservoirs. Therefore, if the region's computer-chip makers had limits placed on the amount of water they could use, water quality would improve."
WORDS: 532          TIME: 00:30:00          DATE: 2010-7-24 17:22:16

    The author suggested in the editorial that computer-chip makers in the region should have limits on the amount of water they could use in order to improve the water quality. The argument which seems to be compelling at first glance actually masks many alternative counter examples and therefore unconvincing it stands.

首段内桶简洁,但是还是建议修改一下,从首段看不出原文的主要逻辑链或者下文可能的反驳点,没有起到统领全文的作用,相反还在一些比较模板化的字句上浪费太多字数。The argument which seems to be compelling at first glance actually masks many alternative counter examples and therefore unconvincing it stands. 从这句中完全看不到有用的信息,建议修改下,更有内容

Initially, the author might be assigning a false clause to the water pollution. The author fails to provide evidence to indicate that computer-chip manufacturers are indeed the reason for water pollution. There might be factors other than computer-chip makers that lead to the serious pollution. For instance, some oil companies might have established their branch factories in Eliot Valley ten years ago and brought huge pollution to the water system there; or perhaps the environmental situation in the whole country was worse than twenty years ago. Eliot Valley might be doing better than most of the regions in the country, which illustrates that computer-chip companies might not cause any pollution[1]. Without further evidence to prove that it is computer-chip manufacturers that result in the water pollution, the author cannot confidently draw the claim that the water pollution was owing to the computer-chip manufacturers. (前后有点重复了,可以精简一下句子)
[1]这个例子的使用有欠妥当。EV的污染状况有没有好转与电脑公司是否是主要污染之间并没有主要关系。并不能说电脑公司入住之后污染情况好转了,就认为公司不是污染源了,这里的逻辑还需注意。

    In addition, it is presumptuous for the author to judge that computer-chip makers are attributable to the low levels of water in the region according to the mere fact that these factories consume large amount of water. Firstly, the computer-chip factories might not use the water in the region. It is possible that they are using water from other regions. Moreover, other reasons might also explain the changes of water levels. Perhaps the climate this year is unusual and brings about the low levels of water; or perhaps over-consumption of underground water is the true reason for low levels of water. Unless the author provides more evidence to eliminate these possibilities, otherwise it is hard for me to accept the claim that low levels of water is due to the water consumption of computer-chip factories. (otherwise
的使用好象没有必要,“除非否则”,这个表达有点中式,事实上只需要unless就可以了。另外可以考虑用conclusion, statement什么的代替claim,保持文字的多样性)

    What's more, the claim that the water quality would rise if the restrictions were taken rests on the unsubstantiated assumption and thereby is open to doubt. One point is that the limits against the quantity of water used might not have relationship with the quality of the water. Perhaps those factories do not release the polluted water into the river. The second point is that common sense tells me[2] that it is easier to pollute water than to recover it. The water quality would not improve unless certain measures designed to eliminate the pollutants in the river are taken. As long as the water is continuously used, the quality of the water will get only worse. (
说得太绝对了,不建议) Without taking these possibilities into consideration, the author cannot convince me that the measures taken will surely improve the water quality.
[2] 个人感觉argu里还是不要出现common sense这类过于主观的表达了,毕竟考的是逻辑不是常识,有时候逻辑正确甚至是可以推翻常识的,这么写太不严谨了

    In sum, there is nowhere more ridiculous(
个人觉得这个也用得不太合适,语气太重了,不过这纯粹是个人语感,可斟酌) than the advice that the limits against water usage of computer-chip makers should be adopted to improve the water quality. To better support his claim, the author has to provide extra evidence to make sure that computer-chip makers are the decisive factors in determining the level and quality of water, together with the evidence that the restrictions will work.

文章语句很顺畅,没有什么大的语法问题,感觉个别用词上还需斟酌一下。
结构很清楚,主要逻辑漏洞都抓到了。建议把一二两个反驳段顺序换一下,因为无论从原文行文顺序来看还是从逻辑链来看,author都是从reportCCM导致low level+pollutionCCM导致pollution,所以以这个顺序行文显得你的逻辑更清晰
首段写的太过模板化了,换个文章换个结论全都都可以套进去,不太建议,首段一来起不到提纲挈领的作用,二来信息量太小。建议看看这个帖子https://bbs.gter.net/thread-1073291-1-1.html
以上是个人意见,希望有用~
作者: tyarel    时间: 2010-7-26 12:23:10

TOPIC: ARGUMENT45 - The following appeared as an editorial in a wildlife journal.

"Arctic deer live on islands in Canada's arctic region. They search for food by moving over ice from island to island during the course of a year. Their habitat is limited to areas warm enough to sustain the plants on which they feed, and cold enough, at least some of the year, for the ice to cover the sea separating the islands, allowing the deer to travel over it. Unfortunately, according to reports from local hunters, the deer populations are declining. Since these reports coincide with recent global warming trends that have caused the sea ice to melt, we can conclude that the decline in arctic deer populations is the result of deer being unable to follow their age-old migration patterns across the frozen sea."
WORDS: 482         TIME: 00:30:00          DATE: 2010/7/26 12:03:19

Based on the global warming trend, supposing that deer cannot follow their migration patterns and associating the report, the author conclude that the ice melting caused by high temperature is the major contributor to the decline in arctic deer populations. However, that is not the case and the conclusion is not as plausible as it seems at first glance.

Admittedly, warming is the tendency of global climate nowadays. However, there still exist some questions that whether it will prevent arctic deer following their age-old patterns. No evidence has been provided that the ice in Canada’s arctic region melts actually. If it is true ice still remains, how can it be obtained that the patterns disappear? Without further researches about the situation of deer's immigrate patterns, it cannot persuade me that arctic deer cannot follow their old-age patterns.

Even the patterns are changed, the report cited by the author also fail to offer enough information to illustrate whether deer populations is decline indeed. Since it is not easy to estimate deer populations, the result of report is not convincing. Even scientists, with a lot of facilities may neglect a large number of arctic deer, let alone hunters just estimating the numbers on their own. Moreover, the change of habitat or immigrate patterns may also lead to the fallacious conclusion. Hence, the result of reports is quite questionable because it cannot be proved the report offered by hunters must be correct and thus the according conclusion that population of arctic deer declines is weak.

Last but not least, even those mentioned are all like the author's hypothesis, the editorial still didn't illustrate why not following patterns is the major reason induce  the decline. Many other factors are not considered in the argument. It's possible that the global warming has more significant impact on the environment that may cause the decrease of deer's food. Those plants may not suitable to the warming weather and extinct in that area. And it's natural that the population will decline since food is not enough to deer. Meanwhile, the author also not take human's influence into account. Perhaps, the population of Canadian is rise and many Canadian has to move to the area which was the habitat of deer. So in order to live there, people build roads or other buildings which decrease the area these arctic deer can live and cause the decrease the number of these deer. Without ruling out all these potential possibilities, the author's attributing decline in populations to climate and change of immigration patterns is weak.

All in all, the author should supply more research on whether the tendency of global temperature change really is really influential to arctic deer's migration patterns and more information demonstrating that number of deer is decreasing as well. Besides, more analysis is needed to show that the change of those mentioned is the only reason to the decline.

谢谢lx~
作者: 清水风铃_nono    时间: 2010-7-26 14:06:24

本帖最后由 清水风铃_nono 于 2010-7-26 14:46 编辑

Based on the global warming trend, supposing that deer cannot follow their migration patterns and associating the report, the author conclude that the ice melting caused by high temperature is the major contributor to the decline in arctic deer populations. However, that is not the case and the conclusion is not as plausible as it seems at first glance.

Admittedly, warming is the tendency of global climate nowadays. However, there still exist some questions that whether it will prevent arctic deer following their age-old patterns. No evidence has been provided that the ice in Canada’s arctic region melts actually. If it is true ice still remains, how can it be obtained that the patterns disappear? Without further researches about the situation of deer's immigrate patterns, it cannot persuade me that arctic deer cannot follow their old-age patterns.(
段意 :最近有ice melt 不代表这里的icemelt了,没问题。需要注意的是,这里的逻辑错误实际上有两点,一个是“在此之后,因此之故”,也就是鹿的数量减少,同时有报道说冰化了,作者就将其等同为因为冰化了,所以鹿减少,这是逻辑谬误。另外一点是本文提到的这一点,这里的冰不一定化了。相比而言,个人感觉攻击前者的强度更大。)

Even the patterns are changed, the report cited by the author also fail to offer enough information to illustrate whether deer populations is decline indeed. Since it is not easy to estimate deer populations, the result of report is not convincing. Even scientists,
(去掉逗号) with a lot of plenty offacilities may neglect a large number of arctic deer, let alone hunters just estimating the numbers on their own. Moreover, the change of habitat or immigrate patterns may also lead to the fallacious conclusion. Hence, the result of reports is quite questionable because it cannot be proved that,形式主语的that不能省略) the report offered by hunters must be correct and thus the according conclusion that population of arctic deer declines is weak.(段意:谁说鹿的数量真的少了

Last but not least, even those mentioned are all like the author's hypothesis, the editorial still didn't illustrate why not following patterns is the major reason induce  the decline. Many other factors are not considered in the argument.(这个点找的是正确的,只是有一点,作者的逻辑错误在于错误把鹿的数量下降和冰化了这两个事实总结为鹿的数量下降是因为迁徙受阻,而前文从未提过迁徙的事儿。这个逻辑错误是“Not Follow”明确这一点,再列举他因才会更有力。) It's possible that the global warming has more significant impact on the environment that may cause the decrease of deer's food. Those plants may not suitable to the warming weather and extinct in that area. And it's natural that the population will decline since food is not enough to deer. Meanwhile, the author also not take human's influence into account. Perhaps, the population of Canadian is rise and many Canadian has to move to the area which was the habitat of deer. So in order to live there, people build roads or other buildings which decrease the area these arctic deer can live and cause the decrease the number of these deer. Without ruling out all these potential possibilities, the author's attributing decline in populations to climate and change of immigration patterns is weak.

All in all, the author should supply more research on whether the tendency of global temperature change really is really influential to arctic deer's migration patterns and more information demonstrating that number of deer is decreasing as well. Besides, more analysis is needed to show that the change of those mentioned is the only reason to the decline.


总体来说,语言没有什么大问题。行文很流畅,没有什么阻碍理解的点。
文章的逻辑点的位置都找得比较准确,只是正文第一段和第三段的说理还可以逻辑性再强一些。建议板油阅读一下ETS官方的argu逻辑点(如果你有作文大讲堂的话在285页)另外也可以参考板内“argu也可以这样写”系列,本人受益匪浅~argu最重要的是逻辑,既然已经过了语言关,那就加强逻辑~~再接再厉~
作者: 清水风铃_nono    时间: 2010-7-26 14:08:01

ARGUMENT9 - The following appeared in a memorandum from a dean at Omega University.

"Fifteen years ago, Omega University implemented a new procedure that encouraged students to evaluate the teaching effectiveness of all their professors. Since that time, Omega professors have begun to assign higher grades in their classes, and overall student grade averages at Omega have risen by thirty percent. Potential employers apparently believe the grades at Omega are inflated; this would explain why Omega graduates have not been as successful at getting jobs as have graduates from nearby Alpha University. To enable its graduates to secure better jobs, Omega University should now terminate student evaluation of professors."
WORDS 528(改后) 时间:1h

提纲:
1.        两个学校不能比较
2.        教师评价体系不一定没有效果
3.        提高就业率需要从学生入手,加强与就业相关的

The arguer is attempted to find the reason why graduates of Omega University have not been as smooth as graduates from Alpha University. The argument assert that grades inflation the only reason, which lake of concerns about other factors that lead to the difference between the two Universities. Plus, the evaluation system is criticized.

The most apparent mistake in this argument is that the employers believe that the grades inflation forms the difference between the two schools, without concerning other factors that may affect the rate of employment. For example, if students of Alpha University are smart before they are admitted, having more competitive abilities of learning, which is desirable to employers that looks for future employees. Plus, the facilities like library and laboratories can be much more convenient to Alpha University students then to Omega student, whom do not have easy access to various information and research experience. Nevertheless, it might be true that Alpha University provides programs that are hot currently in the society like business management and accounting, while Omega University only have anthropology program, or other programs that requires further study before graduate. In a word, the two schools are different; more observation should be conducted before comparison.

Even if the conditions of the two schools are comparable, the argument does not provide enough information to demonstrate that it is the evaluation system is invalid. It is clear that grades improvement occurs after the implement of evaluating procedure, but not possibly as a result of it. On the contrary, student evaluation is an effective way for students to speak their voices and helps to improve the class. It can be true that professors do enhance their methods of teaching, and students do acquire much more knowledge in class thus qualifies higher grades. In this way, inflation, if any, are so modest that they will be wiped out by effective improvement of the quality of education. Moreover, note that the evaluation procedure was implemented 15 years ago. Other conditions various during the one and a half decades. For instance, it may be that a significant change of grading system, or that the society take college education more seriously, and student studies assiduously and persevering that lead to the level up of scores.

Provided that students of Omega perform well at school, it does not indicate that they have the abilities to take jobs. Instead of sticking their minds on the evaluation system, staffs of Omega should concern other aspects to improve the employment of graduates, such as to offer pre-occupation trainings for graduates students. Performing a career is different from learning at school. Communication skills, team work and leadership is highly required, but are not learnt in regular class. In some career, employers may prefer the one with nice presentation skills to the one too shy to talk to others when both of them have satisfying grades. It is the duty of school to help graduate students to be compatible with their career.

In summary, the given argument is not persuasive to terminate student evaluation. Omega should deep analysis the reason of lack of employment and helps more students to set out for their career.
作者: b6pp    时间: 2010-7-27 11:33:44

本帖最后由 b6pp 于 2010-7-27 17:30 编辑

有问题的部分用下划线标出,修改意见用【】表示。
ARGUMENT9 - The following appeared in a memorandum from a dean at Omega University.
"Fifteen years ago, Omega University implemented a new procedure that encouraged students to evaluate the teaching effectiveness of all their professors. Since that time, Omega professors have begun to assign higher grades in their classes, and overall student grade averages at Omega have risen by thirty percent. Potential employers apparently believe the grades at Omega are inflated; this would explain why Omega graduates have not been as successful at getting jobs as have graduates from nearby Alpha University. To enable its graduates to secure better jobs, Omega University should now terminate student evaluation of professors."
WORDS 528(改后) 时间:1h

提纲:
1.两个学校不能比较
2.教师评价体系不一定没有效果
3.提高就业率需要从学生入手,加强与就业相关的

The arguer is【删除is attempted to find the reason why graduates of Omega University have not been as smooth as graduates from Alpha University. The argument assert that grades inflation【添加seems to be the only reason, which lakelacks of【删除of concerns about other factors that 【添加wouldlead to the differencedifferences between the two Universities. Plus, the evaluation system isshould be criticized.

The most apparent mistake in this argument is that the employers believe that the grades inflation forms the difference between the two schools, without concerning other factors that may affect the rate of employment. For example, if students of Alpha University are smart before they are admitted, having more competitive abilities of learning, which is desirable to employers that looks for future employees.
【这句话只有if从句,没主句。可改为:It is quite possible that students of Alpha University are smart enough and have more competitive abilities of learning, which is desirable to employers that are looking for future employees. Plus, the facilities like library and laboratories canmight be much more convenient to Alpha University students thenthan to Omega student, whomwho do not have easy access to various information and research experience. Nevertheless, it might be true that Alpha University provides programs that are hot currently in the societyin current societylike business management and accounting, while Omega University only have anthropology program, or other programs that requires further study before graduatehaving a job. In a word【注意:in a word后面只能跟一个单词,而不能跟一句话,例如,This movie is, in a word, boring.。可改为In sum, the two schools are different; more observationobservations should be conducted before comparison.
【以上一段最后一句分析不到位,只说明了两校可能存在不同,但未说明“因为这种不同所以不能机械照搬经验而废除评价”】

Even if the conditions of the two schools are comparable, the argument does not provide enough information to demonstrate that it is the evaluation system is invalidOmega’s the evaluation system does not work successfully. It is clear that grades improvement occurs after the implement of evaluating procedure, but not possibly
as a result of it.
【这句话把逻辑搞混乱了。到底分数提高是不是评价体系的作用?建议删掉。】On the contrary, student evaluation is an effective way for students to speak their voices and【添加thenhelps to improve the class. It can be true that professors do enhance their methods of teaching, and students do acquire much more knowledge in class thus qualifiesachievinghigher grades. In this way, inflation, if any, are so modest that they will be wiped out by effective improvement of the quality of education. Moreover, note that the evaluation procedure was implemented 15 years ago. Other conditions various during the one and a half decades. For instance, it maythere maybe that【删除that a significant change of grading system, or that【删除thatthe society take college education more seriously,【删除逗号】and student studiesstudents study so assiduously and perseveringly that lead to the level up of scoresthe scores became higher than before.
【以上一段大体意思是对的,但是思路不够清楚。建议思路:论者没有提供有力证据证明学生的实际水平没有比以前有所提高,或是教师的确在给一些能力不高的学生打高分。因为评价体系的实施本身就是希望教师重视学生的意见,从而提高教学水平,最终提高学生的能力,而分数的提高是一个很重要的参数。而论断武断地将整体提升的分数归结为教师不真诚的行为,对学生和教师都非常不公平。

Provided that students of Omega perform well at school, it does not indicate that they have the abilities to take jobs. Instead ofBesidessticking their minds on the evaluation system, staffs of Omega should concern other aspects to improve the employment of graduates, such as to offer pre-occupation trainings for graduates students. Performing a career is different from learning at school. Communication skills, team work and leadership isarehighly required, but are not cannot belearnt in regular class. In some careerFor some industry, employers may prefer the one with nice presentation skills to the onewho istoo shy to talk to others when both of them have satisfying grades. It is the duty of schoolschoolsto help graduate students to be compatible with their career.
【以上一段论点应是“即使废除评价体系,那么就业状况也未必能够好转,因为尚有其它因素”。建议将这个意思表达出来。】

In summary, the given argument is not persuasive to terminate student evaluation. Omega should deepdeeplyanalysisanalyzethe reason of lack of employment and helps more students to set out for their career.
总评:
1.语法错误比较多,例如主谓不一致,句式杂糅等;还有词不达意的现象。建议加强训练。
2.每一段的主题句不够到位,没有说清楚你想批驳啥。
3.分析思路不清楚,往往停留于罗列事实,却没有推理。比如你说“可能还有其他的因素,such as。。。”,但是没说清楚这些因素到底怎样导致题目的观点站不住脚。

不揣浅陋,请多包涵。来信请至 yanghaoship@gmail.com


作者: b6pp    时间: 2010-7-27 11:34:30

51The following appeared in a medical newsletter.
"Doctors have long suspected that secondary infections may keep some patients from healing quickly after severe muscle strain. This hypothesis has now been proved by preliminary results of a study of two groups of patients. The first group of patients, all being treated for muscle injuries by Dr. Newland, a doctor who specializes in sports medicine, took antibiotics regularly throughout their treatment. Their recuperation time was, on average, 40 percent quicker than typically expected. Patients in the second group, all being treated by Dr. Alton, a general physician, were given sugar pills, although the patients believed they were taking antibiotics. Their average recuperation time was not significantly reduced. Therefore, all patients who are diagnosed with muscle strain would be well advised to take antibiotics as part of their treatment."


In this argument, the arguer advocates that antibiotics would be an effective part during the treatment of muscle strain since secondary infections may prevent the patients’ healing quickly. In order to justify this conclusion, the arguer points out that according to a study of two groups of patients, less time is needed for patients who use antibiotics. Although this explanation seems promising at first glance, it is in fact ill-conceived and would be attacked on numerous grounds, as is shown blew.

To begin with, the arguer assumes that muscle strain will certainly cause a secondary infection which will prevent the patients from healing quickly, but no evidence is offered by the arguer about that. In fact, it is quite possible that muscle strain patients will not suffer from a secondary infection because as is known to us all, many factors (such as proper air temperature, dangerous water source and unhealthy living habits, etc.) are needed to lead to an infection. However, the arguer unfairly assumes that all these factors are available just after the muscle strain that it is enough to cause an infection. Maybe some patients are very careful about their illness so they avoid the infection successfully; but this possibility is not considered by the arguer.

In the second place, there are several problems about the study mentioned in the argument so the conclusion that antibiotic is helpful to muscle strain patients is unreliable. Firstly, the conditions of the two groups of patients are not offered. That is to say, we do not have a specific knowledge about these patients’ ages, heath conditions and some other key details. It is very likely that patients in first group are younger and healthier than the second group and then their capability to recover healthy is certainly easier than the second group. Therefore, the first group’s quick healing should not be an effect of antibiotics. Secondly, as is mentioned in the argument, the doctor of first group specializes in sports medicine while the second group’s doctor is a general physician. Obviously, the first doctor has more experience upon muscle strain--one of the sport medicine problems--then his treatment might be more effective and efficient than the second doctor. The arguer fails to consider the possibility that it is the different methods and experience of doctors that make the different results. Thirdly, details of the sugar pills, which may have a passive effect on treatment muscle strain, are not given. Perhaps the sugar pills’ ingredients make against patients’ healing and then first group can heal more quickly.

In addition, even if antibiotics can treat the infection successfully, the arguer ignores the passive respect of antibiotics so his assertion that all the patients should be advised to take antibiotics is dangerous. It is quite possible some patients is sensitive to antibiotics and their health condition would be worse when treated with antibiotics.

In sum, it is unwise to come to the conclusion that all the patients should be advised to take antibiotics to treat muscle strain quickly. In order to draw a better conclusion, more details are needed. Specifically, the arguer should offer more evidence about information of the (1) two groups of patients and their doctors; (2) the passive effect of sugar pill; (3) whether secondary infections will happen necessarily. Under this circumstance, the argument would be more convincing.
多谢!
作者: guo0693    时间: 2010-8-1 10:35:14

本帖最后由 guo0693 于 2010-8-1 11:23 编辑

占位改83楼~
51The following appeared in a medical newsletter.
"Doctors have long suspected that secondary infections may keep some patients from healing quickly after severe muscle strain. This hypothesis has now been proved by preliminary results of a study of two groups of patients. The first group of patients, all being treated for muscle injuries by Dr. Newland, a doctor who specializes in sports medicine, took antibiotics regularly throughout their treatment. Their recuperation time was, on average, 40 percent quicker than typically expected. Patients in the second group, all being treated by Dr. Alton, a general physician, were given sugar pills, although the patients believed they were taking antibiotics. Their average recuperation time was not significantly reduced. Therefore, all patients who are diagnosed with muscle strain would be well advised to take antibiotics as part of their treatment."


In this argument, the arguer advocates that antibiotics would be an effective part during the treatment of muscle strain since secondary infections may prevent the patients’ healing quickly. In order to justify this conclusion, the arguer points out that according to a study of two groups of patients, less time is needed for patients who use antibiotics. Although this explanation seems promising at first glance, it is in fact ill-conceived and would be attacked on numerous grounds, as is shown blew.
个人认为,忽略了最重要的一个结论,可以说整个Topic就是为了引出该结论的,那就是Therefore, all patients who are diagnosed with muscle strain would be well advised to take antibiotics as part of their treatment.单从开头看来我觉得作者是想攻击study->secondary infection->antibiotics…effective;

To begin with, the arguer assumes that muscle strain will certainly cause a secondary infection which will prevent the patients from healing quickly
, but no evidence is offered by the arguer about that. In fact, it is quite possible that muscle strain patients will not suffer from a secondary infection because as is known to us all, many factors (such as proper air temperature, dangerous water source and unhealthy living habits, etc.) are needed to lead to an infection. However, the arguer unfairly assumes that all these factors are available just after the muscle strain that it is enough to cause an infection. Maybe some patients are very careful about their illness so they avoid the infection successfully;(,) but this possibility is not considered by the arguer.hasn’t been taken into consideration?

In the second place, there are several problems about the study mentioned in the argument so the conclusion that antibiotic is helpful to muscle strain patients is unreliable(so
的断句第一次我没看懂,so 改成which result in会不会更好点?). Firstly, the conditions of the two groups of patients are not offered. That is to say, we do not have a specific knowledge about these patients’ ages, heathhealthy? conditions and some other key details(details/key factors 感觉key details有点冗余). It is very likely that patients in first group are younger and healthier than the second group and then their capability to recover healthy is certainly easier than the second group.capability is easier…?这句有问题 Therefore, the first group’s quick healing should not be an effect of antibiotics. Secondly, as is mentioned in the argument, the doctor of first group specializes in sports medicine while the second group’s doctor is a general physician. Obviously, the first doctor has more experience upon muscle strain--one of the sport medicine problems--then his treatment might be more effective and efficient than the second doctor. 这个论点有点weak,如果我是读者我会反驳为什么外科手术的就不能是个专家?我觉得应该argue两个人的领域不同背景不同已经够了,不要把经验什么的问题考虑进来,有点强词夺理了The arguer fails to consider the possibility that it is the different methods and experience of doctors that make the different results. Thirdly, details of the sugar pills, which may have a passive effect on treatment muscle strain, are not given.我来找茬的话,医生有那么的不专业吗?把效果不好的药给病人吃,治疗还是整治呢? Perhaps the sugar pills’ ingredients make against patients’ healing and then first group can heal more quickly.

In addition, even if antibiotics can treat the infection successfully, the arguer ignores the passive respect of antibiotics so his assertion that all the patients should be advised to take antibiotics is dangerous. It is quite possible some patients is sensitive to antibiotics and their health condition would be worse when treated with antibiotics.
想法很好,需要展开 用例子证明抗生素的消极作用 后面那个假设有点weakIt is quite possible some patients is sensitive to antibiotics and their health condition would be worse when treated with antibiotics特例不具有普遍性,除非有调差显示大多数人对抗生素敏感

In sum, it is unwise to come to the conclusion that all the patients should be advised to take antibiotics to treat muscle strain quickly. In order to draw a better conclusion, more details are needed. Specifically, the arguer should offer more evidence about information of the (1) background of/detailed condition of two groups of patients and their doctors; (2) the passive effect of sugar pill;
没有副作用怎么办?糖丸有副作用吗? (3) whether secondary infections will happen necessarily. Under this circumstance, the argument would be more convincing.
以上只是我的个人观点 可能会有失偏颇 毕竟我也不是牛人,有错误的地方请原谅呵呵~
作者: guo0693    时间: 2010-8-1 10:36:04

本帖最后由 guo0693 于 2010-8-1 11:08 编辑

多谢楼下的了~第一篇ARG, 还剩8天。。。
TOPIC: ARGUMENT7 - The following appeared in a letter to the editor of the Clearview newspaper.

"In the next mayoral election, residents of Clearview should vote for Ann Green, who is a member of the Good Earth Coalition, rather than for Frank Braun, a member of the Clearview town council, because the current members are not protecting our environment. For example, during the past year the number of factories in Clearview has doubled, air pollution levels have increased, and the local hospital has treated 25 percent more patients with respiratory illnesses. If we elect Ann Green, the environmental problems in Clearview will certainly be solved."
WORDS: 531          TIME: 01:50:46          DATE: 2010-8-1 10:25:14

The arguer claim that Ann Green, who is a member of the Good Earth Coalition, should be voted in the next mayoral election rather than Frank Braun, who is a member of the Clearview town council, for the reason that Clearview town council's current members are not protecting their environment. To support that reason the arguer list several evidences: the increasing number of factories, increased air pollution and  25 percent more local patients with respiratory illness. Thus he make a prediction that the environmental problems will be solved if Ann Green were elected in order to mislead our residents to make their choice. However, the arguer's logic do suffer several fatal fallacies.

To begin with, the three grounds of arguments lend no support to the claim that members in Frank Braun are not protecting local environment. Putting aside the possibility that some "green" factories without contamination are introduced by the council to accelerate the development of local economy, how can the arguer assure that no actions are taken by the council? As we all know that environment protection is a long run problem which can not be done within a short period of time. When it comes to the air pollution, my doubt remains the same. Besides, the increasing number of patients with respiratory illness have little to do with the pollution, unless the arguer rule out the other possibilities that causes such phenomenon. As far as I know, there are more than one causation that may result in respiratory illness such as smoking, eating duty foods, bacteria infection and so forth. Thus, the 25 percent more patients poorly account for the deterioration of environment which is one premise of that the members overlook environmental problems.

Even if the council takes no actions in protecting local environment, the arguer still fail to convince us that Frank Braun will keep abreast of the council's policy. What if he disagree with those policy that may aggravate environmental problems but have less power to realize his idea?  Even a reactionary gang has some guys with good willings. We can not deny one person just according to where he or she stands. Most times the minorities have no choice but to comply. Thus, the council's policy may not represent for Frank Braun's attitude towards environmental problems.

Last but not the least, how could the arguer affirm that Ann Green could bring us a better solution if Frank Braun couldn't? The membership of Good Earth Coalition does not means that Ann is capable to deal with some important issues out of the realm of environment. To manage a town is not just simple as to lead an organization, let alone we know nothing but a membership of the Good Earth Coalition about Ann. I will remain my right to question Ann Green's qualification unless the arguer provides enough information to support that Ann Green is better than any one else for this position.

In conclusion, to convince us, more detailed information should be provided rather than just list some weak even irrevelant facts with a unconvinced logic. Otherwise, I will doubt the motivation of the arguer's claim: is there any personal interest laying behind this opinion?
作者: guo0693    时间: 2010-8-1 16:00:42     标题: 这个帖子怎么这么冷清。、。

本帖最后由 guo0693 于 2010-8-1 18:31 编辑

哎 小顶一下 别沉了。。。希望有个版主给置顶。。。
作者: zmy1029    时间: 2010-8-1 20:22:48

题目:ARGUMENT161 - In a study of reading habits of Leeville citizens conducted by the University of Leeville, most respondents said they preferred literary classics as reading material. However, a follow-up study conducted by the same researchers found that the type of book most frequently checked out of each of the public libraries in Leeville was the mystery novel. Therefore, it can be concluded that the respondents in the first study had misrepresented their reading habits.

In the argument, by make a comparison between literary classic as the result of a study of reading habits conducted in University of Leeville and mystery novel as the result of another study of the public libraries in Leeville, the argument concludes the respondents in the first study misrepresented their reading habits due to the distinct results of two studies. However, the argument lacks a series of information that can be the possible factors lead to different result in two studies.

To begin with, the author fails to take the time between two studies into account. It is possible that it is a long period of time that has changed the structure of population of Leeville senior people in the first study who prefer to
literary classics rather than mystery novel responded by young people in the later study .Or consider the popular season that literary classics time in the first study and it is the popular with mystery novel during the second study time. Without ruling out these possibilities above, the difference between two studies is reasonable.

Besides, the respondents it also important to the result. The author provides no evidence about the respondents that whether they are a group of people who can representative of citizens of Leeville as a whole. Perhaps in the first study respondents are a group of students in the university whose
major is literaure.Assuming that respondents in the latter study do represente the reading habitat of Leeville citizens. In this case, it is just the researcher’s fault not the misrepresentation of respondents in the first study.

In addition, the author overlooks the distinct between libraries. It is likely that the amount of books especially classic literary of library in university is larger than that in public libraries. In this case, citizens prefer to go to university to borrow books. Or perhaps the price of classic literary is much cheaper than mystery novel so that citizen could afford them.

Finally, the definition of mystery novel and literary classics is unclear. It is entire possible that a book could regard as mystery novel and literary classic at the same time.Odyseey, for instance, which could regard as classic literary due to the story described happened in ancient Greek, while the story is make by author not reality so that it could be mystery novel too.Libaray is replete with such books that between the line of classics literary and mystery novel. Hence, the author could not conclude that respondents misrepresented in the first study.

In sum, the author should make the conclusion before consider the several factors like time, respondents, libraies and definition of books may differentiate the results of two studies.

想用多举例展开的方法写,没怎么用模板,求指教
作者: zmy1029    时间: 2010-8-1 20:24:07

本帖最后由 zmy1029 于 2010-8-1 21:02 编辑

TOPIC: ARGUMENT7 - The following appeared in a letter to the editor of the Clearview newspaper.

"In the next mayoral election, residents of Clearview should vote for Ann Green, who is a member of the Good Earth Coalition, rather than for Frank Braun, a member of the Clearview town council, because the current members are not protecting our environment. For example, during the past year the number of factories in Clearview has doubled, air pollution levels have increased, and the local hospital has treated 25 percent more patients with respiratory illnesses. If we elect Ann Green, the environmental problems in Clearview will certainly be solved."
WORDS: 531          TIME: 01:50:46          DATE: 2010-8-1 10:25:14

The arguer claim that Ann Green, who is a member of the Good Earth Coalition, should be voted in the next mayoral election rather than Frank Braun, who is a member of the Clearview town council, for the reason that Clearview town council's current members are not protecting their environment. To support that reason the arguer list several evidences: the increasing number of factories, increased air pollution and  25 percent more local patients with respiratory illness. Thus he make
s
a prediction that the environmental problems will be solved if Ann Green were elected in order to mislead our residents to make their choice. However, the arguer's logic do suffer several fatal fallacies.

To begin with, the three grounds of arguments lend no support to the claim that members in Frank Braun are not protecting
did not protect local environment. Putting aside the possibility that some "green" factories without contamination are introduced by the council to accelerate the development of local economy, how can the arguer assure that no actions are taken by the council? As we all know that environment protection is a long run problem which can not be done within a short period of time. When it comes to the air pollution, my doubt remains the same. Besides, the increasing number of patients with respiratory illness have little to do with the pollution, unless the arguer rule out the other possibilities that causes such phenomenon. As far as I know, there are more than one causation that may result in respiratory illness such as smoking, eating duty foods, bacteria infection and so forth. Thus, the 25 percent more patients poorly account for the deterioration of environment which is one premise of that the members overlook environmental problems.
应该说一下作为政府没有控制污染的论据不成立,点一下主题,这样逻辑比较清晰.

Even if the council takes no actions in protecting local environment, the arguer still fail to convince us that Frank Braun will keep abreast of the council's policy. What if he disagreed with those that policy that may aggravate environmental problems but have less power to realize his idea?  Even a reactionary gang has some guys with good willings. We can not deny one person just according to where he or she stands. Most times the minorities have no choice but to comply. Thus, the council's policy may not represent for Frank Braun's attitude towards environmental problems.
段与段之间要有过渡句照应一下

Last but not the least, how could the arguer affirm that Ann Green could bring us a better solution if but Frank Braun couldn't? The membership of Good Earth Coalition does not means that Ann is capable to deal with some important issues out of the realm of environment. To manage a town is not just simple as to lead an organization, let alone we know nothing but a membership of the Good Earth Coalition about Ann. I will remain my right to question Ann Green's qualification unless the arguer provides enough information to support that Ann Green is better than any one else for this position.

In conclusion, to convince us, more detailed information should be provided rather than just list some weak even irrevelant facts with a unconvinced logic. Otherwise, I will doubt the motivation of the arguer's claim: is there any personal interest laying behind this opinion?
结尾用问句不太好吧, 应该委婉的提出下建议, 建议再考虑下造成污染的其他方面情况,以及考虑能够胜任市长的其他人选,不然没有什么实质性的内容.

你的正文段的写法不错,没怎么用模板,逻辑也挺清晰的
反对政府没有控制污染不一定是K的责任 – A不一定能胜任

但是没有主要过渡句,加上过度句逻辑就更清晰了,虽然说不要用太多模板,但是北美的结构框架还是值得我们学习的,结构清晰,再充分展开论证,离高分就不远了.
还有你要控制时间啊,时间是个大问题,这篇文章不错,但在三十分钟能写出来吗?

以上个人意见,仅供参考,大家互相学习,加油~~

我跟你考试时间差不多, 没几天了, issue准备的差不多了吧?
作者: guo0693    时间: 2010-8-1 21:47:44

呵呵 谢谢指点~最后那个问句我只是表达一种可能的推论而已,看了些文章说结尾再重复一遍arguer的错误累赘,说ETS要我们的insight opinion,所以提出一种假设的质疑而已。Issue还早呢~从时间上就可以看出来我的阿狗都还是处女座呢。。。Issue更惨 第一篇用了3个多小时。。。还有7天 悲剧啊~ 88# zmy1029
作者: Maydaysj    时间: 2010-8-2 14:46:48

题目:ARGUMENT161 - In a study of reading habits of Leeville citizens conducted by the University of Leeville, most respondents said they preferred literary classics as reading material. However, a follow-up study conducted by the same researchers found that the type of book most frequently checked out of each of the public libraries in Leeville was the mystery novel. Therefore, it can be concluded that the respondents in the first study had misrepresented their reading habits.

In the argument, by make a comparison between literary classic as the result of a study of reading habits conducted in University of Leeville and mystery novel as the result of another study of the public libraries in Leeville,
提取主干后觉得题目并不是比较literary classic& mystery novel的,换个说法概括调查结果吧,另外觉得这半句有点冗长,建议简练些 the argument concludes the respondents in the first study misrepresented their reading habits due to the distinct results of two studies.欣赏这句总结简练准确 However, the argument lacks a series of information that can be the possible factorswhich may缺的话好像语法不太对 lead to different resultS in two studies.

To begin with, the author fails to take the time between two studies into account. It is possible that it is a long period of time that has changed the structure of population of Leeville senior people in the first study who prefer toliterary classics rather than mystery novel responded by young people in the later study .Or consider the popular season that literary classics time in the first study and it is the popular with mystery novel during the second study time.(
这句语法有问题,句子结构不清楚) Without ruling out these possibilities above, the difference between two studies is reasonable.

Besides, the respondents it
拼写注意哦--is also important to the result. 建议具体概括说明respondents的哪方面The author provides no evidence about the respondents that whether they are a group of people who can representative of换成动词 citizens of Leeville as a whole. Perhaps in the first study respondents are a group of students in the university whose major is literaure拼错了. Assuming that respondents in the latter study do represente拼错了 the reading habitat of Leeville citizens.语法 In this case, it is just the researcher’s fault not the misrepresentation of respondents in the first study.感觉这段还是说respondents的结构的,是不是可以跟上段第一点整合下,将上段第二点加些别的展开成另一段,重整下段落要点会更有逻辑

In addition, the author overlooks the distinct between libraries. It is likely that the amount of books especially classic literary of library in university(--literary classic books in university libraries) is larger than that in public libraries. In this case, citizens prefer to go to university to borrow books.
(这句想表达什么点,不清楚,另有些university library=public library,举这个不太恰当) Or perhaps the price of classic literary is much cheaper than mystery novel so that citizen could afford them. (这个可以展开写呐,看文学书的可能都去买书而不借书,有收藏价值,悬疑小说一时之快,借而不买。。。。)

Finally, the definition of mystery novel and literary classics is unclear. It is entire possible that a book could
be regard as mystery novel and literary classic at the same time. Odyseey, (拼写)for instance, which (去掉,不然没谓语了could regard as classic literary due to the story described happened in ancient Greek, while the story is makemade by author not reality so that it could be mystery novel too.(逗号前后都不通顺)Libaray is replete with such books that between the line of classics literary and mystery novel. Hence, the author could not conclude that respondents misrepresented in the first study.

In sum, the author should make the conclusion before consider
ing the several factors like time, respondents, libraies and definition of books may differentiate the results of two studies.时序说反了

小建议:
1.希望多注意下语法,意思才能表达的更清楚
2.段落要点组合要考虑下逻辑联系,抓住重要的攻击点药深入展开说明
3.先放到word里检查下拼写

PS:再接再厉!加油!!


87# zmy1029
作者: Maydaysj    时间: 2010-8-2 14:49:22

TOPIC: ARGUMENT89 - The following appeared in a newspaper published in the state of Celera.

"Speed limits on our state's highways should be eliminated in order to increase our state's prosperity. Because greater speed means more efficient travel, commercial deliveries will be faster, increasing business profits. Elimination of speed limits will also make driving more attractive to motorists, so that more people will use the highways, providing more highway toll revenues for the state. At the same time, safety on our highways will not be affected: daytime speed limits were eliminated last year in the western states of our country, and no significant increase in the number of accidents in these states has been reported."
DATE: 2010/8/2 9:41:09 自己修改稿多指教!谢谢~~

The author suggests an elimination of speed limits on highways of Celera to increase its prosperity. However, all demonstrations about the economic benefits of high speed and safety guarantee are full of flaws.
First of all, high speed elimination does not necessarily lead to commercial benefit and revenue increase. Greater speed can make commercial delivers faster only on the condition that the traffic is in good order. If there is no speed limit, it is quiet possible that some highway drivers want to overtake others which may cause the traffic condition into chaos. If it's the case, high speed elimination will not increase but decrease the travel efficiency and thus give a bad effect on business profit. Furthermore, vehicles for commercial deliver on Celera’s highways may not belong to Celera and therefore cannot make contributions to the state’s financial increase whatever their business benefit blossom. As for the point that high speed attracts more motorists and thus increases toll revenue, however, increasing number of motorists also means heavier burden on the maintenance. Suppose that another 100 motorists can get only $200 toll revenue while will cost $500 for maintenance, then could we say it’s worthwhile? Without weight the cost and revenue of traffic flow increase, it is unconvincing to conclude that great speed will do increase economic benefits.
Second, the author cites an example of the western stats to claims that high speed has little to do with safety. However, the testimony that "no significant increase “is vogue. Whether there is a great increase in traffic accidents or not in fact is unknown. Does it possible that researchers didn’t conduct a precise calculation survey and thus gave this inaccurate statistic result? Or does it possible that the proposers or officers supporting speed elimination conceal the fact deliberately? In a nutshell, the author should provide more convincing information to support that high speed has little thing to do with safety. In addition, even we concede the point mentioned above, it does not mean the same will do in state of Celera. There may be some differences between them in states population, condition of highway, and climate ect. Maybe the western states are in peripheral areas where there are little population and thus less vehicles on the highway which alone can guarantee low ratio of accidents. And if the climate in western states is stable, while state of Celera are full of hazards weather like tornadoes and rainstorm, then the traffic accidents will be undoubtly more in the latter. If so, eliminating speed limit will add insult to injury there. And since the western states only adopt speed limits elimination on daytime, it is inappropriate to extend it to Celera to night when the traffic accidents are more likely to happen.
Finally, even elimination of speed limit can boost commercial benefit and not affect traffic safety, it dose not lead to prosperity of Celera state. Toll revenue makes up of only negligible proportion of the almost every state revenue let alone commercial vehicles may not belong to Celera. To really increase Celera’s prosperity, what should be done is adjusting measures to local conditions, for examples, exploiting local resources, developing local state industry, and strengthening exchanges and cooperation with outside through the full use of highways.
To sum up, before hastily eliminating speed limits, the local officers should careful weight the benefit and cost to ensure economic propensity and traffic safety.
作者: 雨馨    时间: 2010-8-3 00:18:22

本帖最后由 雨馨 于 2010-8-3 20:44 编辑

TOPIC: ARGUMENT89 - The following appeared in a newspaper published in the state of Celera.

"Speed limits on our state's highways should be eliminated in order to increase our state's prosperity. Because greater speed means more efficient travel, commercial deliveries will be faster, increasing business profits. Elimination of speed limits will also make driving more attractive to motorists, so that more people will use the highways, providing more highway toll revenues for the state. At the same time, safety on our highways will not be affected: daytime speed limits were eliminated last year in the western states of our country, and no significant increase in the number of accidents in these states has been reported."
DATE: 2010/8/2 9:41:09 自己修改稿多指教!谢谢~~

The author suggests an elimination of speed limits on highways of Celera to increase its prosperity. However, all [the] demonstrations about the economic benefits of high speed and safety guarantee are full of flaws. [这个however表示强烈转折,但其实句子中的转折没有那么充分]

First of all, high speed elimination does not necessarily lead to commercial benefit and revenue increase. Greater speed can make commercial delivers faster only on the condition that the traffic is in good order. If there is no speed limit, it is quiet [quite] possible that some highway drivers want to overtake others which may cause the traffic condition into chaos. If it's the case, high speed elimination will not increase but decrease [increase和decrease不能这么用啦……not enhance but reduce] the travel efficiency and thus give a bad effect on business profit. Furthermore, vehicles for commercial deliver on Celera’s highways may not belong to Celera and therefore cannot make contributions to the state’s financial increase whatever their business benefit blossom. As for the point that high speed attracts more motorists and thus increases toll revenue, however, increasing number of motorists also means heavier burden on the maintenance. Suppose that another 100 motorists can get only $200 toll revenue while will cost $500 for maintenance, then could we say it’s worthwhile? [这个例子可能不符常识,建议去掉] Without weight the cost and revenue of traffic flow increase, it is unconvincing to conclude that great speed will do increase economic benefits.

Second, the author cites an example of the western stats to claims that high speed has little to do with safety. However, the testimony that "no significant increase “is vogue. Whether there is a great increase in traffic accidents or not in fact is unknown. [这个论据不是很充分,因为对政府而言还是可以统计的,这个攻击文章中的既定事实可能不是很强力] Does it possible that researchers didn’t conduct a precise calculation survey and thus gave this inaccurate statistic result? Or does it possible that the proposers or officers supporting speed elimination conceal the fact deliberately? In a nutshell, the author should provide more convincing information to support that high speed has little thing to do with safety. In addition, even we concede the point mentioned above, it does not mean the same will do in state of Celera. There may be some differences between them in states population, condition of highway, and climate ect [etc]. Maybe the western states are in peripheral areas where there are little population and thus less  [fewer] vehicles on the highway which alone can guarantee low ratio of accidents. And if the climate in western states is stable, while state of Celera are full of hazards weather like tornadoes and rainstorm, then the traffic accidents will be undoubtly [undoubtedly] more in the latter. If so, eliminating speed limit will add insult to injury there. And since the western states only adopt speed limits elimination on daytime, it is inappropriate to extend it to Celera to night when the traffic accidents are more likely to happen.
[一大段中有三个小点可能较为混乱,伤其十指可能不如断其一指]

Finally, even elimination of speed limit can boost commercial benefit and not affect traffic safety, it dose [does] not [necessarily] lead to prosperity of Celera state. Toll revenue makes up of only negligible proportion of the almost every state revenue let alone commercial vehicles may not belong to Celera. To really increase Celera’s prosperity, what should be done is adjusting measures to local conditions, for examples, exploiting local resources, developing local state industry, and strengthening exchanges and cooperation with outside through the full use of highways.
[这点感觉想得很对,但是其实题目还有运输增快带来的经济效能,这里既然要驳就要包括,不然觉得有点以偏概全——虽然第一段说明过运输增快也可能不能带来经济效能,但这一段还是要包括比较好。]

To sum up, before hastily eliminating speed limits, the local officers should careful weight the benefit and cost to ensure economic propensity and traffic safety.
[个人比较喜欢这个结尾]

总体而言,对语言的把握可能还不够到位,自己已经修改过的稿子还有不少问题,建议用word和这个检查一下:
http://www.whitesmoke.com/free-online-checker
然后上考场的时候,也要注意避免很绕的句子和吃不准的表达

另外,感觉你思路是
1、不一定能带来经济效益
2、不一定没车祸
3、还有其他更好的方式带动经济效益
私以为,1和2换下位置,对逻辑和表达感觉更容易

还有就是,总体感觉,点很多,改批的似乎都没漏下,但详略不得当,并且相关的几点放在一个段落里,又没有顺畅的过渡。
个人意见是把握主要矛盾,攻击得彻底,但只要攻击几个重要错误,并且各占一段,对于我们这样没有语言表达优势的foreinger比较容易掌握。

个人意见,要批判吸收呀⋯⋯
作者: zmy1029    时间: 2010-8-3 00:35:41

本帖最后由 zmy1029 于 2010-8-3 10:47 编辑

改95L
The following appeared in a newsletter from a national astronomy association.

"Various sources are predicting higher-than-average temperatures across the country next winter, including in Sun City, the traditional location of our yearly winter conference. Higher winter temperatures are sure to result in higher-than-usual tourism in Sun City, a location already known for its attractive beaches and good weather. Hotels will have fewer rooms available, transportation will be more difficult to reserve, and public places such as parks and restaurants will be more crowded. These conditions are likely to significantly reduce attendance at the conference. We should therefore move our conference to a city less popular with winter tourists."

--------------------------------------------

The argument states that as Sun City, the famous tourist attraction and the traditional location of the association's yearly conference, would be warmer next winter, it would be much crowded during the association's yearly conference and thus reducing the attendance. Therefore, the conference should be moved to another city. That is, however, not so reasonable as it may seem to be.


A city known for its attractive beaches and good weather, according to the common sense, would be popular in summer but probably not so popular in winter because summer is the right season for swimming and sun bathing. Would there be more people in a warm winter than in summer that could occupy all the public areas a tourist attraction should have (
句子不通)? Without answering this question with specific data, the higher temperature alone could not sufficiently support the idea that the city would be too crowded during the conference. 攻击点找对了,理由有点牵强吧, 据我所知, 滨海城市冬天游客的确多, 夏天太热,比如说海南岛. 你要说这点的话要吧理由说清楚,这段就都是一句话的意思,其他的全是模板. 可以说可能这个城市纬度怎样, 夏天和冬天气温差不多, 而且没有证据证明以前冬天游客就比夏天多……. 还有一点可以攻击的是温度升高没说升多少, 什一两度不会有很大的影响……

Even if the city is overloaded at that time, there is no evidence supporting that the condition would significantly reduce attendance at the conference according to the argument. People attend conferences for the content, not the environment of the location. Perhaps the members would attend the national astronomy association's yearly meeting in regardless of the crowd and the inconvenience. Since the logical chain between the condition in Sun City and the attendance of the conference is unclear, the soundness of the whole passage is weakened.

Moreover, there are other concepts that can avoid absence and keep the traditional location at the same time. For example, the association can make reservation for both the hotel rooms and the vehicles a long time before the conference and thus avoiding finding for the empty room or the transportation in the rush hour. Nevertheless, they can hold the meeting in the location far away from the beaches and the scenery spots as it would alleviate the possible situation. Failed to analyze these possible solutions, the conclusion of the argument is apparently not well-supported.



To sum up, the argument is not well grounded because lacking of enough supportive materials to prove that whether there is a causal relationship among the rising temperature, the congestion degree, the attendance and the suggestion. After all, it would be a pity to move from the traditional conference location just because the temperature there is higher-than-average.


语法没有大错误 说说你的逻辑问题吧
你在攻击的时候逻辑链不太清楚,东说一下西说一下,有的理由还有些强词夺理
作者的逻辑链是这样的
全国气温升高 sun city气温升高 游客增加 房间减少 出席率减少 应该转移

你的顺序是 1.人不一定多 2.出席不一定少 3.出席率减少不一定转移
很明显有几个重点你没攻击到
第一个逻辑链 常见的问题  以多推少  你没有攻击
第三个逻辑链 可以这样说  游客增加 房间交通不一定拥挤,因为不知道sun city原来的情况,也许根本就没有供不应求的现象…….
还有一个可攻击重点就是攻击后果  转移了会怎样, 打破了传统不好, 换了地点参加者不熟悉,说不定反而会影响出勤率……..

建议你可以看看那个argument242题库分析,argument版的精华帖, argument也是需要展开的,不是说一个理由再加模板,全是空话, 不要被北美范文误导了

以上个人意见,仅供参考,加油哦O(_)O
作者: zmy1029    时间: 2010-8-3 00:37:06

题目:ARGUMENT51 - The following appeared in a medical newsletter.

"Doctors have long suspected that secondary infections may keep some patients from healing quickly after severe muscle strain. This hypothesis has now been proved by preliminary results of a study of two groups of patients. The first group of patients, all being treated for muscle injuries by Dr. Newland, a doctor who specializes in sports medicine, took antibiotics regularly throughout their treatment. Their recuperation time was, on average, 40 percent quicker than typically expected. Patients in the second group, all being treated by Dr. Alton, a general physician, were given sugar pills, although the patients believed they were taking antibiotics. Their average recuperation time was not significantly reduced. Therefore, all patients who are diagnosed with muscle strain would be well advised to take antibiotics as part of their treatment."

In the argument, the author cites a study that two group of patients were treated for muscle injuries by taking antibiotics and sugar pills, the results is antibiotics group recuperated quicker than the other group without taking that medicine. In that case, the author infers that antibiotics have effect on treating secondary infections which prevent healing quickly after severe muscle strain. Hence, the author concludes antibiotics could make muscle strain patients heal quickly, therefore he or she recommends that antibiotics should be taken by all patients who are diagnosed with muscle train. However, the argument suffers from a series of poor assumptions, which render it unpersuasive.

To begin with, the author cities the study as the evidence to prove taking antibiotic is an effective means of treating secondary infection. However, the author provides no evidence that whether patients who involved the study have suffered secondary infection after muscle strain. If not, the result that antibiotics could shorten recuperation time prove nothing about the hypothesis that long recuperation time of muscle strain patients causes by secondary infection.

Besides, even assuming all the patients have secondary infections, the study still appears to be weakened by its questionable method and statistics. As we know, for an experiment to be accurate, it must be controlled, with a balance between the experimental and the control group, which means all the factors that potentially affect the results, should remain constant during the experiment. In the above study, unfortunately, three factors may impact on the results. First of all, there is no information about the patients involved in the study such as ages, backgrounds, physical condition and the severity of injury. It is possible that patients are younger, stronger, or injured more serious in the first group than those in the other group. Secondly, the author fails to consider the fact that doctors play a significant role throughout the treatment. In generally speaking, Dr. Newland who specializes in sports medicine have more experiment and professional skill in treating muscle than a general physician Dr. Alton. In this case, it is entirely possible the first group recuperated faster than the other group without antibiotics due to the treatment from sports medicine specialist. Thirdly, the sugar pills given to the second group is open to doubt. Perhaps sugar pills have side effect on secondary so that made the second group recuperated slower. In short, without ruling out all other possible factors may influence the result, the study cannot be a evidence to prove the hypotheses.

Finally, even if the author can substantiate all of the foregoing assumptions, the author’s assertion that all the patients diagnosed with muscle strain should take antibiotics is unwarranted. Experience tells me, a part of people are allergic to antibiotics due to their physical constitution and genetic predisposition or they are in a special period such as pregnancy. Taking antibiotics mistakenly will cause uncomfortable like dizziness and shock. For that matter, the author’s conclusion is unjustifiable.

In sum, the argument, while it seems logical at first, has several flaws as discussed above. The author should provide better evidence about the information of patients, doctors and sugar pills to ensure the reliable of the study as evidence of the assumption. Furthermore, before making recommendation to patients, the author should make sure they do not have an allergy to antibiotics to guarantee their health.

这是自己先改过的,我很纠结第三段要不要分开,都是说的实验的问题, 但太长了
给点建议吧
作者: 雨馨    时间: 2010-8-3 00:46:31

我又来了~改哪篇啊?92L的呢
zmy1029 发表于 2010-8-3 00:35


在此,有劳了~

The following appeared in a newsletter from a national astronomy association.

"Various sources are predicting higher-than-average temperatures across the country next winter, including in Sun City, the traditional location of our yearly winter conference. Higher winter temperatures are sure to result in higher-than-usual tourism in Sun City, a location already known for its attractive beaches and good weather. Hotels will have fewer rooms available, transportation will be more difficult to reserve, and public places such as parks and restaurants will be more crowded. These conditions are likely to significantly reduce attendance at the conference. We should therefore move our conference to a city less popular with winter tourists."

--------------------------------------------

The argument states that as Sun City, the famous tourist attraction and the traditional location of the association's yearly conference, would be warmer next winter, it would be much crowded during the association's yearly conference and thus reducing the attendance. Therefore, the conference should be moved to another city. That is, however, not so reasonable as it may seem to be.

A city known for its attractive beaches and good weather, according to the common sense, would be popular in summer but probably not so popular in winter because summer is the right season for swimming and sun bathing. Would there be more people in a warm winter than in summer that could occupy all the public areas a tourist attraction should have? Without answering this question with specific data, the higher temperature alone could not sufficiently support the idea that the city would be too crowded during the conference.

Even if the city is overloaded at that time, there is no evidence supporting that the condition would significantly reduce attendance at the conference according to the argument. People attend conferences for the content, not the environment of the location. Perhaps the members would attend the national astronomy association's yearly meeting in regardless of the crowd and the inconvenience. Since the logical chain between the condition in Sun City and the attendance of the conference is unclear, the soundness of the whole passage is weakened.

Moreover, there are other concepts that can avoid absence and keep the traditional location at the same time. For example, the association can make reservation for both the hotel rooms and the vehicles a long time before the conference and thus avoiding finding for the empty room or the transportation in the rush hour. Nevertheless, they can hold the meeting in the location far away from the beaches and the scenery spots as it would alleviate the possible situation. Failed to analyze these possible solutions, the conclusion of the argument is apparently not well-supported.

To sum up, the argument is not well grounded because lack of enough supportive materials to prove that there is a causal relationship among the rising temperature, the congestion degree, the attendance and the suggestion. After all, it would be a pity to move from the traditional conference location just because the temperature there is higher-than-average.
作者: ljz1990    时间: 2010-8-3 11:42:32

本帖最后由 ljz1990 于 2010-8-4 00:29 编辑

占楼改94


"Doctors have long suspected that secondary infections may keep some patients from healing quickly after severe muscle strain. This hypothesis has now been proved by preliminary results of a study of two groups of patients. The first group of patients, all being treated for muscle injuries by Dr. Newland, a doctor who specializes in sports medicine, took antibiotics regularly throughout their treatment. Their recuperation time was, on average, 40 percent quicker than typically expected. Patients in the second group, all being treated by Dr. Alton, a general physician, were given sugar pills, although the patients believed they were taking antibiotics. Their average recuperation time was not significantly reduced. Therefore, all patients who are diagnosed with muscle strain would be well advised to take antibiotics as part of their treatment."

In the argument, the author cites a study that two group of patients were treated for muscle injuries by taking antibiotics and sugar pills, the results is antibiotics group recuperated quicker than the other group without taking that medicine. In that case, the author infers that antibiotics have effect on treating secondary infections which prevent healing quickly after severe muscle strain. Hence, the author concludes antibiotics could make muscle strain patients heal quickly, therefore he or she recommends that antibiotics should be taken by all patients who are diagnosed with muscle train. However, the argument suffers from a series of poor assumptions, which render it unpersuasive.(开头太长了而且有点混乱,可以简洁一点,大概把思路理清就可以马上反驳了The author draw a conclusion that all muscle strain patient should take antibiotics partly as their treatment according to a study, in which one group took antibiotics and treated by Dr. Newland while the other group take sugar pills and treated by Dr. Alton. In addition the study also prove a hypothesis about secondary infections and severe muscle strain. 你看这样好一点么)

To begin with, the author cities the study as the evidence to prove taking antibiotic is an effective means of treating secondary infection. (中心句不明显,把第一第二句话互换一下吧)However, the author provides no evidence that whether patients who involved the study have suffered secondary infection after muscle strain. If not, the result that antibiotics could shorten recuperation time prove nothing about the hypothesis that long recuperation time of muscle strain patients causes by secondary infection.(额,我觉得分析好像有点不够,显得有点单薄。)

Besides, even assuming all the patients have secondary infections, the study still appears to be weakened by its questionable method and statistics. As we know, for an experiment to be accurate, it must be controlled,(逗号去掉) with a balance(平衡???没看懂哦,可能是我也不知道,如果是正确用法请告知,呵呵) between the experimental and the control group, which means(in which会不会好些?) all the factors that potentially affect the results,(逗号不要) should remain constant(constant??可以这样用吗,这里的对比只有两个组,用the same会不会好些?) during the experiment. In the above study, unfortunately, three factors (影响因素其实还挺多,说是三个有点绝对,other factors 就够了)may impact on(on 不要) the results. First of all, there is no information about the patients involved in the study such as ages, backgrounds(什么background是职业吗,说background太宽泛), physical condition and the severity of injury. It is possible that patients are younger, stronger, or injured more serious(更严重?怎么会好的更快) in the first group than those in the other group.(分析不够充分哦,可以再稍微一两句说明一下,因为更年轻……所以好的更快) Secondly, the author fails to consider the fact that doctors play a significant role throughout the treatment. In generally speaking, Dr. Newland who specializes in sports medicine have more experiment and professional skill in treating muscle than a general physician Dr. Alton. In this case, it is entirely possible the first group recuperated faster than the other group without antibiotics due to the treatment from sports medicine specialist. Thirdly, the sugar pills given to the second group is open to doubt. Perhaps sugar pills have side effect on secondary so that made the second group recuperated slower. In short, without ruling out all other possible factors may influence the result, the study cannot be a evidence to prove the hypotheses.(恩,我觉得你第二段可以分开来说,好看一些,也可以展开的更充分。其实要说起来,第一个点也是关于这个实验的严谨性的问题,重点就在攻击这个实验,最后一点只是顺带说一下,分开来应该没问题,个人意见)

Finally, even if the author can substantiate all of the foregoing assumptions, the authors assertion that all the patients diagnosed with muscle strain should take antibiotics is unwarranted. Experience tells me(common sense好些,经验毕竟是你个人的经验), a part of people are allergic to antibiotics due to their physical constitution and genetic predisposition or they are in a special period such as pregnancy. Taking antibiotics mistakenly will cause uncomfortable like dizziness and shock(shock是怎么来的?). For that matter, the authors conclusion is unjustifiable.(是不是还要加进扭伤的程度,每个人的扭伤程度不一样,是否抗生素对每种扭伤都有效)

In sum, the argument, while it seems logical at first, has several flaws as discussed above. The author should provide better evidence about the information of patients, doctors and sugar pills to ensure the reliable of the study as evidence of the assumption. Furthermore, before making recommendation to patients, the author should make sure they do not have an allergy to antibiotics to guarantee their health.

条理还算蛮清晰,但是某些地方说理不够充分。语言虽然有些地方比较别扭整体来说还流畅吧,或许多练练就好,额我是菜鸟就不乱给你说了。还有一个,我在想是不是大家都这样写,虽然没做过这个题目,但是觉得很熟悉,如果要我写,估计也八九不离十,然后语言还差点。不知道94楼大虾有咩高见啊。。。。
作者: ljz1990    时间: 2010-8-3 13:42:16

42.The following appeared in a proposal from the economic minister of the country of Paraterra.
"In order to strengthen its lagging economy, last year the government of the nearby country of Bellegea began an advertising campaign to promote ecologically sound tourism (ecotourism). This year the number of foreign visitors arriving at Bellegea's main airport doubled, and per capita income in Bellegea increased by ten percent. To provide more income for the population of Paraterra and also preserve the natural environment of our tiny country, we too should begin to promote ecotourism. To ensure that our advertising campaign is successful, we should hire the current director of Bellegea's National Tourism Office as a consultant for the campaign."
临近城市Bellegea的政府为促进其落后的经济,去年开展了一项对生态旅游的广告推广。今年到达Bellegea主要机场的外国游客数量翻了一番,人均收入增加了10%。为增加Paraterra居民的收入并保护我们有限国土的天然环境,我们也应该推广生态旅游。为保证我们广告策略的成功,我们应该雇佣Bellegea现任国家旅游局的主任来担任广告的顾问。

The proposal above recommends that, in order to raise people's income and preserve the natural environment, Paraterra(P) should  promote ecotourism and the advertising campaign of neighbour country Bellegea (B), and hire the current director of B's National Tourism Office as consultant as well. The reasoning given seems at first glance sound, while it quite not the case when looked in details.

1.        机场人数增加 不等于 旅游业兴旺 他们不一定是来旅游的 会议或者是转机(这个或者另一个附近机场航空线路改变或者低票价) 就算是来旅游的 是生态游吗 音乐会足球赛呢不一定是因为这个广告推广 而且double这个数字需要质疑

First of all, the foreign visitors’ number doubled is not equal to the growth in ecotourism. For one, the number doubled fails to give comprehensible information. If the foreign visitors' number is very small in the past, then the raise could be not that significant. For another, the foreigners arrive at B airport for other purposes but not ecotourism. It could be well possible they are arriving just for transfer or business or academic conferences, rock music festivals, football or tennis tournament?


2.        人均收入增长不代表B的经济有了好转,首先是10%如果是通货膨胀人均收入本来就是增长的。就算经济有了好转,其中旅游业占到的份额又有多少,农业工业,

Furthermore, the author concludes too harshly that the 10% income raise per capita result in the advertise campaign. Consider the possible situation that B's lagging economy happens due to its inflation, not only the income per capita but also the commodity price increases, and the latter is greater in increasing rate than the former. Then the purchasing capacity per capita is in fact decreasing, which serves no good to the people's better life even though their income rises.

Moreover, the author fails to take the other factors cause the income raise. It could be well possible that the income raise is due to agriculture and industrial   development while ecotourism only contribute little in the income raise. In addition, even if they come to ecotourism, is it due to the advertising campaign? The reason these foreign people decide to visit B could well be the recommend of their friends who returning from B but not the advertising campaign.

3.        承认这个广告成功推动了旅游业拉动经济增长,在B地适用在P适用吗,旅游景点,交通,风俗。对比一下。可行性。而且要雇用旅游局主任,这个广告的成功(如果是的话),不是,或者不止是他一个人的功劳,而且他对P的情况不了解,合适吗能胜任吗?负面作

Another important factor must be considered that P's situation is quite different from B's. The author provide no evidence that p have well enough travelling resource. P is a tiny country as the author states, it could possibly happen that P didn't have that much places satisfying the visiting condition, or the visiting resort in P had already be fully even over exploited? Whether the relevant facilities and services around are easy to access? Besides, the author have to be well aware of the traffic information, the local traditions and customs to assess the feasibility, the cost and side effects contained of carrying out the advertising campaign in a whole.

In addition, whether to hire the current director of Bellegea's National Tourism Office as a consultant for the campaign awaits further consideration. Since the success in B's campaign, if it is, does not or not only owe to the director only but the group as a whole, possibly still, he could even have no idea about the P's current situation, it hard to say that B's director is qualified for the work.

4.        P还想保护环境,但是生态旅游能保护环境吗,恰恰相反吧

Last but not least, the author fails to convince me that ecotourism could preserve the natural environment. If the advertising campaign attracts a great many people to participate in ecotourism, the natural would environment inevitably bear much heavy burden. Even with good management, the waste might be throw about, the animals habits might be disturbed by people activities, which is harmful even critical for the environment preserving.

P 要推动经济增长保护环境是个很好的想法,借鉴B地的做法也是好的。但是应该有更周全的考虑,权衡各方因素,弄清B p两地的不同,或者多参考,而不是只盯在B上面。
It is a good idea to provide more income and preserve the natural environment as well as to consult B's situation, while P's minister need more comprehensive consideration, get all the factors weighted, and be clear about the difference between P and B. If not, the conclusion the author is too inadequate too be sound.


第一篇,没有用AWP。写的挺乱的,因为可以写的点太多了不知道详细写哪些,按什么顺序写,这还是删改了一些的。求指导。。。
作者: Maydaysj    时间: 2010-8-3 16:56:50

多谢,有劳了!! 92# 雨馨
作者: 雨馨    时间: 2010-8-4 15:57:29

本帖最后由 雨馨 于 2010-8-5 00:14 编辑

我又来了(顺便征人互改作文⋯⋯下周五就考了⋯⋯TAT)
抱走97L

42. The following appeared in a proposal from the economic minister of the country of Paraterra.
"In order to strengthen its lagging economy, last year the government of the nearby country of Bellegea began an advertising campaign to promote ecologically sound tourism (ecotourism). This year the number of foreign visitors arriving at Bellegea's main airport doubled, and per capita income in Bellegea increased by ten percent. To provide more income for the population of Paraterra and also preserve the natural environment of our tiny country, we too should begin to promote ecotourism. To ensure that our advertising campaign is successful, we should hire the current director of Bellegea's National Tourism Office as a consultant for the campaign."
临近城市Bellegea的政府为促进其落后的经济,去年开展了一项对生态旅游的广告推广。今年到达Bellegea主要机场的外国游客数量翻了一番,人均收入增加了10%。为增加Paraterra居民的收入并保护我们有限国土的天然环境,我们也应该推广生态旅游。为保证我们广告策略的成功,我们应该雇佣Bellegea现任国家旅游局的主任来担任广告的顾问。

The proposal above recommends that, in order to raise people's income and preserve the natural environment, Paraterra(P) should  promote ecotourism and the advertising campaign of [as 才对,用of的话,是P为B去做嫁衣了] neighbor [neighbour] country Bellegea (B), and hire the current director of B's National Tourism Office as [a] consultant as well. The reasoning [reasons] given [by the argument] seems at first glance sound [seem sound at first glance 更顺一些] while it [is] quite not the case when looked in details. [这里有一个比较别扭的地方,即given和looked的主体不一样,会造成误解,所以要分开指名或换个句式]

1.        机场人数增加 不等于 旅游业兴旺 他们不一定是来旅游的 会议或者是转机(这个或者另一个附近机场航空线路改变或者低票价) 就算是来旅游的 是生态游吗 音乐会足球赛呢不一定是因为这个广告推广 而且double这个数字需要质疑

First of all, the foreign visitors’ number doubled is not equal to the growth in ecotourism. For one, the number doubled fails to give comprehensible information. [这个有点过于绝对,你能说人家不一定、不有效,不能说人家不可理解吧] If the foreign visitors' number is very small in the past, then the raise could be not that significant. [这句话我明白是说如果人少,double一下也不能说明问题,但是这句话变成英文非常模糊,感觉没说清楚,不如举具体的数字和例子] For another, the foreigners arrive at B airport for other purposes but not ecotourism. It could be well possible they are arriving just for transfer or business or academic conferences, rock music festivals, football or tennis tournament? [这个当然可能,但是注意原文的意思是,这个上涨和推广生态旅游的时间是吻合的,所以有逻辑关系。应该举例说明同时有其他可能导致人数上升。]
[For one和For another我觉得比较别扭……如果你是看范文看下来的当然也可]

2.        人均收入增长不代表B的经济有了好转,首先是10%如果是通货膨胀人均收入本来就是增长的。就算经济有了好转,其中旅游业占到的份额又有多少,农业工业,
[这两个观点距离其实很大啊,为什么放在一起?而且文中没有说经济好转,只说人均收入增加。重点应该是人均收入增加并不一定是因为推广生态旅游。]

Furthermore, the author concludes too harshly that the 10% income raise per capita result in [comes from 不然逻辑关系颠倒了] the advertise campaign. Consider the possible situation that B's lagging economy happens due to its inflation, not only the income per capita but also the commodity price increases, and the latter is greater in increasing rate than the former. Then the purchasing capacity per capita is in fact decreasing, which serves no good to the people's better life even though their income rises. [问题跟上文是一样的,要体现他因的性质,你说通胀,也要说可能当时正好有通胀,不说明就说10%来自于通胀是很奇怪的]

Moreover, the author fails to take the other factors [that may] cause the income raise [into consideration]. It could be well possible that the income raise is due to agriculture and industrial   development while ecotourism only contribute little in the income raise. [同样的问题,不赘述了] In addition, even if they come to ecotourism, is it due to the advertising campaign? The reason these foreign people decide to visit B could well be the recommend of their friends who returning from B but not the advertising campaign.

3.        承认这个广告成功推动了旅游业拉动经济增长,在B地适用在P适用吗,旅游景点,交通,风俗。对比一下。可行性。而且要雇用旅游局主任,这个广告的成功(如果是的话),不是,或者不止是他一个人的功劳,而且他对P的情况不了解,合适吗能胜任吗?负面作

Another important factor must be considered that P's situation is quite different from B's. The author provides no evidence that p [P] have well enough travelling resource. P is a tiny country as the author states, it could possibly happen that P didn't have that much places satisfying the visiting condition, or the visiting resort in P had already be fully even over exploited? Whether the relevant facilities and services around are easy to access? Besides, the author have to be well aware of the traffic information, the local traditions and customs to assess the feasibility, the cost and side effects contained of carrying out the advertising campaign in a whole.

In addition, whether to hire the current director of Bellegea's National Tourism Office as a consultant for the campaign awaits further consideration. Since the success in B's campaign, if it is, does not or not only owe to the director only but the group as a whole, possibly still, he could even have no idea about the P's current situation, it hard to say that B's director is qualified for the work. [这一句好绕,建议拆开]

4.        P还想保护环境,但是生态旅游能保护环境吗,恰恰相反吧

Last but not least, the author fails to convince me that ecotourism could preserve the natural environment. If the advertising campaign attracts a great many people to participate in ecotourism, the natural would environment inevitably bear much heavy burden. Even with good management, the waste might be throw about, the animals habits might be disturbed by people activities, which is harmful even critical for the environment preserving.
[生态旅游本来就是要保护环境的……这一点可能很难驳倒]

P 要推动经济增长保护环境是个很好的想法,借鉴B地的做法也是好的。但是应该有更周全的考虑,权衡各方因素,弄清B p两地的不同,或者多参考,而不是只盯在B上面。
It is a good idea to provide more income and preserve the natural environment as well as to consult B's situation, while P's minister need more comprehensive consideration, get all the factors weighted, and be clear about the difference between P and B. If not, the conclusion the author is too inadequate too be sound.


第一篇,没有用AWP。写的挺乱的,因为可以写的点太多了不知道详细写哪些,按什么顺序写,这还是删改了一些的。求指导。。。

总体而言,我觉得有比较明显的问题是:
在语言方面,感觉特别绕,而且很多表达不是错,而是诡异(当然也有可能我水平不够)建议不要生硬地套用好句子之类的⋯⋯
在论证方面,感觉你还不是特别理解他因的论证方法,不是说可能有其他解释就可以了,而是要进一步说明其他情况也非常可能发生……(我表达不好,建议多看看范文体会)
在结构方面,还是有什么都攻击,但都不是特别有力的感觉。其实Argument不需要面面俱到,在你的6个段落中,选3个重要且能有力攻击的比较好。

在整体思路方面,攻击两个主要论据,然后攻击不能照搬照抄我觉得是可以的。

个人意见啦,大家多交流咯^^
作者: 雨馨    时间: 2010-8-4 19:08:17

TOPIC: ARGUMENT203 - The following appeared in a newspaper feature story.

"At the small, nonprofit hospital in the town of Saluda, the average length of a patient's stay is two days; at the large, for-profit hospital in the nearby city of Megaville, the average patient stay is six days. Also, the cure rate among patients in the Saluda hospital is about twice that of the Megaville hospital. The Saluda hospital has more employees per patient than the hospital in Megaville, and there are few complaints about service at the local hospital. Such data indicate that treatment in smaller, nonprofit hospitals is more economical and of better quality than treatment in larger, for-profit hospitals."
WORDS: 538          TIME: 00:24:06          DATE: 2010-8-4 15:46:06

The argument listed several facts to show the small, nonprofit hospital in Saluda is better than the large, for-profit hospital in Megaville thus concluding smaller, nonprofit hospitals are better than larger, for-profit hospitals. Ignoring different conditions between Saluda hospital and Megaville hospital and taking the two isolated examples to represent all the hospitals, the argument is not well-grounded.

A large city hospital as Megaville hospital is, would face much severe diseases than Saluda hospital, the small, nonprofit hospital according to our common sense because when we catch small diseases, we choose local hospital and when we get into serious diseases, we turn to large hospitals. Therefore, there is no wonder that the average length in Megaville hospital is three times longer than that in Saluda hospital. Take an extreme example, if one hospital cures cancer in 6 days while another hospital cures bad cold in 2 days, is the latter better than the former? Similarly, it might explain the relatively lower cure rate in Megaville hospital as well. Imagine if Saluda hospital is full of local people with headache and throat congestion while Megaville hospital is full of people from a large area with heart attack and brain disorder, how can we blame Megaville hospital for its low cure rate? Without further statistics of the diseases in the two hospitals, it is too hasty to judge the treatments basing on those facts.

Megaville hospital may face not only the more serious diseases but also the more patients for its larger scale. It is entirely possible that Megaville hospital has more employees, especially the employees of higher degrees, than Saluda hospital but it has to treat many more patients than Saluda hospital that lower its employees-per-patient data. And as Megaville might have a better employee team, it does not necessarily link to its medical performance. Nevertheless, the number of complaints may largely affected by the number of patients. See if there was 100 patients in Saluda hospital and 10 of them complained about service while there was 1000 patients in Megaville hospital and 20 of them complained about service, Megaville would be consider to have better service than Saluda. Therefore, the argument should provide more data of the number of the patients in these hospitals to support its conclusion better.

However, even if Saluda hospital does give better treatments than Megaville hospital, it is too justify saying smaller, nonprofit hospitals are of better quality than larger, for-profit hospitals. There was no evidence, according to the argument, sufficiently proves that Saluda hospital is a typical small, nonprofit hospital and Megaville hospital is a typical large, for-profit hospital. What if Saluda hospital is the best small, nonprofit hospital while Megaville hospital is the worst large, for-profit hospital in the country? Much more examples of both small and large hospitals are needed to judge the two types of hospitals as two groups.

To sum up, the argument might be partly true but quite limited. Beyond the isolated data, much more background materials are needed to judge Saluda hospital and Megaville hospital while even more evidences are needed to judge small, nonprofit hospitals and large, for-profit hospitals. Making conclusion basing on just a few direct comparisons is not so reasonable after all.




欢迎光临 寄托家园留学论坛 (https://bbs.gter.net/) Powered by Discuz! X2